Sumario´ - labMA/UFRJmcabral/livros/livro-calculo/... · 2019. 6. 12. · Sumario´ 1 Enunciados...

106

Transcript of Sumario´ - labMA/UFRJmcabral/livros/livro-calculo/... · 2019. 6. 12. · Sumario´ 1 Enunciados...

Page 1: Sumario´ - labMA/UFRJmcabral/livros/livro-calculo/... · 2019. 6. 12. · Sumario´ 1 Enunciados dos Exercícios 1.1 Exercícios de Limite. 1 1.1.1 Exercícios de Fixação. 1 1.1.2

Curso de Cál ulo de Uma Variável(Somente a Lista de Exer í ios)Segunda Edição V2.1Outubro de 2011Mar o Aurélio Palumbo CabralPhD Indiana University EUAProfessor do Instituto de Matemáti aUniversidade Federal do Rio de Janeiro

Departamento de Matemáti a Apli adaInstituto de Matemáti aUniversidade Federal do Rio de JaneiroRio de Janeiro - BrasilCópias são autorizadas e bem vindas: divulgue nosso trabalho! Consulte o sítiowww.labma.ufrj.br/~m abral/livros ou entre em ontato om o autor emmap abral(at)ufrj(dot)br.

Page 2: Sumario´ - labMA/UFRJmcabral/livros/livro-calculo/... · 2019. 6. 12. · Sumario´ 1 Enunciados dos Exercícios 1.1 Exercícios de Limite. 1 1.1.1 Exercícios de Fixação. 1 1.1.2

ii

Page 3: Sumario´ - labMA/UFRJmcabral/livros/livro-calculo/... · 2019. 6. 12. · Sumario´ 1 Enunciados dos Exercícios 1.1 Exercícios de Limite. 1 1.1.1 Exercícios de Fixação. 1 1.1.2

Sumario

1 Enun iados dos Exer í ios 11.1 Exer í ios de Limite . . . . . . . . . . . . . . . . . . . . . . . . . . . . . . 11.1.1 Exer í ios de Fixação . . . . . . . . . . . . . . . . . . . . . . . . . 11.1.2 Problemas . . . . . . . . . . . . . . . . . . . . . . . . . . . . . . . 41.1.3 Extras . . . . . . . . . . . . . . . . . . . . . . . . . . . . . . . . . 61.1.4 Desaos . . . . . . . . . . . . . . . . . . . . . . . . . . . . . . . . 71.2 Exer í ios de Continuidade . . . . . . . . . . . . . . . . . . . . . . . . . . . 81.2.1 Exer í ios de Fixação . . . . . . . . . . . . . . . . . . . . . . . . . 81.2.2 Problemas . . . . . . . . . . . . . . . . . . . . . . . . . . . . . . . 91.2.3 Extras . . . . . . . . . . . . . . . . . . . . . . . . . . . . . . . . . 101.2.4 Desaos . . . . . . . . . . . . . . . . . . . . . . . . . . . . . . . . 111.3 Exer í ios de Derivada . . . . . . . . . . . . . . . . . . . . . . . . . . . . 121.3.1 Exer í ios de Fixação . . . . . . . . . . . . . . . . . . . . . . . . . 121.3.2 Problemas . . . . . . . . . . . . . . . . . . . . . . . . . . . . . . . 151.3.3 Extras . . . . . . . . . . . . . . . . . . . . . . . . . . . . . . . . . 171.3.4 Desaos . . . . . . . . . . . . . . . . . . . . . . . . . . . . . . . . 191.4 Exer í ios de Apli ação de Derivada . . . . . . . . . . . . . . . . . . . . . 201.4.1 Exer í ios de Fixação . . . . . . . . . . . . . . . . . . . . . . . . . 201.4.2 Problemas . . . . . . . . . . . . . . . . . . . . . . . . . . . . . . . 231.4.3 Extras . . . . . . . . . . . . . . . . . . . . . . . . . . . . . . . . . 271.4.4 ⋆Problemas (Taxas Rela ionadas) . . . . . . . . . . . . . . . . . . . . 311.4.5 ⋆Problemas (Derivação Implí ita) . . . . . . . . . . . . . . . . . . . . 341.4.6 Desaos . . . . . . . . . . . . . . . . . . . . . . . . . . . . . . . . 351.5 Exer í ios de Integral . . . . . . . . . . . . . . . . . . . . . . . . . . . . . 371.5.1 Exer í ios de Fixação . . . . . . . . . . . . . . . . . . . . . . . . . 371.5.2 Problemas . . . . . . . . . . . . . . . . . . . . . . . . . . . . . . . 391.5.3 Extras . . . . . . . . . . . . . . . . . . . . . . . . . . . . . . . . . 401.5.4 ⋆Problemas (Integração e Substituição Trigonométri a) . . . . . . . . . 421.5.5 ⋆Problemas (Integração de Funções Ra ionais) . . . . . . . . . . . . . 431.5.6 Desaos . . . . . . . . . . . . . . . . . . . . . . . . . . . . . . . . 431.6 Exer í ios de Apli ações da Integral . . . . . . . . . . . . . . . . . . . . . . 471.6.1 Exer í ios de Fixação . . . . . . . . . . . . . . . . . . . . . . . . . 471.6.2 Problemas . . . . . . . . . . . . . . . . . . . . . . . . . . . . . . . 481.6.3 Extras . . . . . . . . . . . . . . . . . . . . . . . . . . . . . . . . . 501.6.4 ⋆Problemas (Comprimento de Curvas no Plano) . . . . . . . . . . . . . 521.6.5 ⋆Problemas (Área de Superfí ie de Sólido de Revolução) . . . . . . . . 521.6.6 Desaos . . . . . . . . . . . . . . . . . . . . . . . . . . . . . . . . 52iii

Page 4: Sumario´ - labMA/UFRJmcabral/livros/livro-calculo/... · 2019. 6. 12. · Sumario´ 1 Enunciados dos Exercícios 1.1 Exercícios de Limite. 1 1.1.1 Exercícios de Fixação. 1 1.1.2

iv SUMÁRIO2 Respostas dos Exer í ios 552.1 Limite . . . . . . . . . . . . . . . . . . . . . . . . . . . . . . . . . . . . . 552.1.1 Exer í ios de Fixação . . . . . . . . . . . . . . . . . . . . . . . . . 552.1.2 Problemas . . . . . . . . . . . . . . . . . . . . . . . . . . . . . . . 582.1.3 Extras . . . . . . . . . . . . . . . . . . . . . . . . . . . . . . . . . 602.1.4 Desaos . . . . . . . . . . . . . . . . . . . . . . . . . . . . . . . . 612.2 Continuidade . . . . . . . . . . . . . . . . . . . . . . . . . . . . . . . . . . 622.2.1 Exer í ios de Fixação . . . . . . . . . . . . . . . . . . . . . . . . . 622.2.2 Problemas . . . . . . . . . . . . . . . . . . . . . . . . . . . . . . . 632.2.3 Extras . . . . . . . . . . . . . . . . . . . . . . . . . . . . . . . . . 642.2.4 Desaos . . . . . . . . . . . . . . . . . . . . . . . . . . . . . . . . 642.3 Derivada . . . . . . . . . . . . . . . . . . . . . . . . . . . . . . . . . . . . 652.3.1 Exer í ios de Fixação . . . . . . . . . . . . . . . . . . . . . . . . . 652.3.2 Problemas . . . . . . . . . . . . . . . . . . . . . . . . . . . . . . . 662.3.3 Extras . . . . . . . . . . . . . . . . . . . . . . . . . . . . . . . . . 682.3.4 Desaos . . . . . . . . . . . . . . . . . . . . . . . . . . . . . . . . 692.4 Apli ação de Derivada . . . . . . . . . . . . . . . . . . . . . . . . . . . . 702.4.1 Exer í ios de Fixação . . . . . . . . . . . . . . . . . . . . . . . . . 702.4.2 Problemas . . . . . . . . . . . . . . . . . . . . . . . . . . . . . . . 712.4.3 Extras . . . . . . . . . . . . . . . . . . . . . . . . . . . . . . . . . 772.4.4 ⋆Problemas (Taxas Rela ionadas) . . . . . . . . . . . . . . . . . . . . 832.4.5 ⋆Problemas (Derivação Implí ita) . . . . . . . . . . . . . . . . . . . . 852.4.6 Desaos . . . . . . . . . . . . . . . . . . . . . . . . . . . . . . . . 862.5 Integral . . . . . . . . . . . . . . . . . . . . . . . . . . . . . . . . . . . . . 872.5.1 Exer í ios de Fixação . . . . . . . . . . . . . . . . . . . . . . . . . 872.5.2 Problemas . . . . . . . . . . . . . . . . . . . . . . . . . . . . . . . 892.5.3 Extras . . . . . . . . . . . . . . . . . . . . . . . . . . . . . . . . . 912.5.4 ⋆Problemas (Integração e Substituição Trigonométri a) . . . . . . . . . 922.5.5 ⋆Problemas (Integração de Funções Ra ionais) . . . . . . . . . . . . . 942.5.6 Desaos . . . . . . . . . . . . . . . . . . . . . . . . . . . . . . . . 942.6 Apli ações da Integral . . . . . . . . . . . . . . . . . . . . . . . . . . . . . 962.6.1 Exer í ios de Fixação . . . . . . . . . . . . . . . . . . . . . . . . . 962.6.2 Problemas . . . . . . . . . . . . . . . . . . . . . . . . . . . . . . . 962.6.3 Extras . . . . . . . . . . . . . . . . . . . . . . . . . . . . . . . . . 992.6.4 ⋆Problemas (Comprimento de Curvas no Plano) . . . . . . . . . . . . . 1012.6.5 ⋆Problemas (Área de Superfí ie de Sólido de Revolução) . . . . . . . . 1012.6.6 Desaos . . . . . . . . . . . . . . . . . . . . . . . . . . . . . . . . 101

Page 5: Sumario´ - labMA/UFRJmcabral/livros/livro-calculo/... · 2019. 6. 12. · Sumario´ 1 Enunciados dos Exercícios 1.1 Exercícios de Limite. 1 1.1.1 Exercícios de Fixação. 1 1.1.2

Capıtulo 1

Enunciados dos Exercıcios

1.1 Exer í ios de Limite1.1.1 Exer í ios de FixaçãoExer í io 1.Considere o grá o de y = f(x) esboçada no grá o abaixo. Determine oslimites abaixo. Caso algum não exista, determine os limites laterais.(a) limx→a

f(x); (b) limx→b

f(x); ( ) limx→c

f(x).x

y

a b c

5

6

3

1

Exer í io 2.Determine se é Verdadeiro ou Falso. Se for falso dê um ontraexemplo ou orrija. Se for verdadeiro justique.(a) x ∈ R; |x− 3| ≤ 2 = [1, 5].(b) x ∈ R; |x+ 2| < 1 = (1, 3).( ) √x2 = x para todo x ∈ R.(d) se g(x) =

4; x 6= 2;

π; x = 2, então lim

x→2g(x) = g(2) = π.Exer í io 3.Determine se é Verdadeiro ou Falso. Se for falso dê um ontraexemplo ou orrija. Se for verdadeiro justique.(a) Se lim

x→3+f(x) = 5, então e lim

x→3f(x) = 5.(b) Se lim

x→2f(x) = 4, então e lim

x→2−f(x) = −4.009.set.2011 1

Page 6: Sumario´ - labMA/UFRJmcabral/livros/livro-calculo/... · 2019. 6. 12. · Sumario´ 1 Enunciados dos Exercícios 1.1 Exercícios de Limite. 1 1.1.1 Exercícios de Fixação. 1 1.1.2

2 CAPÍTULO 1. ENUNCIADOS DOS EXERCÍCIOS( ) Se limx→2

f(x) = 4, então f(2) = 4.(d) Existe uma função f tal que limx→3+

f(x) 6= limx→3−

f(x) = limx→3

f(x).Exer í io 4.Considere a função f dada por f(x) =

5; x ≤ 1

7; 1 < x ≤ 2

9; x > 2

. Determine limx→k

f(x)ou, aso não exista, os limites laterais para:(a) k = 1; (b) k = 0.9999; ( ) k = 1.0001;(d) k = 2; (e) k = 1.9999; (f) k = 2.0001.Exer í io 5.Aplique a denição do módulo para esboçar o o grá o de:(a) cosx

| cos(x)| ; (b) √|x|.Exer í io 6.Partindo de grá o de funções simples (±x2, ±1/x, ±1/x2, √x, sen(x), |x|,log(x), ex), utilizando translações verti ais e/ou horizontais e/ou reexões, esbo e o grá ode: (a) y = 1 +

√x (b) y = 2 + sen(x); ( ) y = log(x− 1) + 2;(d) y =

−1

(x+ 2)3; (e) y = |(x+ 1)(x+ 2)|; (f) y = |ex − 2|.Exer í io 7.Determine os limites:(a) lim

x→2

x− 2

(2− x)(3− x); (b) lim

x→0

x4 + x

x3 + 2x; ( ) lim

x→3

x− 3

x2 − 4.Exer í io 8.Dena lim

x→+∞f(x) = +∞ seguindo mutatis mutandis1 as denições dadas notexto.Exer í io 9. Faça o estudo de sinal do numerador e denominador para determinar os valoresde x que satisfazem as desigualdades:(a) 3− x2

x2 − 1≥ 0; (b) x3 − 1

x(x2 − 4)≤ 0.Exer í io 10. Faça o estudo de sinal e o esboço do grá o dos polinmios abaixo.(a) p(x) = (x− 2)(x+ 3)(1− x); (b) q(x) = (x− 2)2(x+ 1);( ) r(x) = (3− x)(x− 2)2(x− 5).Exer í io 11.Determine os limites:(a) lim

x→0−

1

x; (b) lim

x→0−

1

x2; ( ) lim

x→0−

x

|x| ; (d) limx→0

x3

|x| ;(e) limx→2

x2 + 1

x− 2; (f) lim

x→0−

(

x+1

x

)

; (g) limx→3+

x

x2 − 9.1latim para modique o que tem que ser modi ado

Page 7: Sumario´ - labMA/UFRJmcabral/livros/livro-calculo/... · 2019. 6. 12. · Sumario´ 1 Enunciados dos Exercícios 1.1 Exercícios de Limite. 1 1.1.1 Exercícios de Fixação. 1 1.1.2

1.1. EXERCÍCIOS DE LIMITE 3Exer í io 12.Determine os limites:(a) limx→+∞

x√x2 + 1

; (b) limx→+∞

(

x+1

x

); ( ) limx→+∞

1 + 6x

x− 2;(d) lim

x→−∞

2x− x2

3x+ 5; (e) lim

x→+∞

2x3 − 4

5x+ 3; (f) lim

x→−∞

7x3 − 15x2

13x;(g) lim

x→−∞

3x5 + x− 1

x5 − 7;(h) lim

x→+∞

3x3 + 2x4 + 5x5 − 1

4x5 − 3x4 − 2x2 + x+ 3; (i) lim

x→+∞

5x10 − 3x7 + 9x6 − 12x2 − x+ 1

x9 − 7x2 − 21.Exer í io 13.Complete as la unas om pode/não pode:(a) A assíntota verti al do grá o de y = f(x) inter eptar o grá o de f .(b) A assíntota horizontal do grá o de y = g(x) inter eptar o grá o de g.Exer í io 14.Determine se é Verdadeiro ou Falso. Se for falso dê um ontraexemplo ou orrija. Se for verdadeiro justique. Se lim

x→1q(x) = 0, então(a) lim

x→1

3

q(x)= +∞; (b) lim

x→1

q(x)

f(x)= 0; ( ) lim

x→1

q(x)

−x2= 0.Exer í io 15.Qual das Figuras abaixo pode representar o grá o de uma função g tal que:(i) lim

x→∞g(x) = 1 (ii) lim

x→−∞g(x) = −1(iii) lim

x→1+g(x) = +∞ (iv) lim

x→1−g(x) = −∞.

-1

1

1 x

y

-1

1

1 x

y

-1

1

1 x

y

-1

1

1 x

y

-1

1

1 x

y

-1 1 x

y

-1 1 x

y

-1 1 x

y

-1

1

1 x

y

-1

1

1 x

y

-1

1

1 x

y

-1

1

1 x

y

-1

1

1 x

y

-1

1

1 x

y

-1

1

1 x

y

-1

1

1 x

y

-1

1

1 x

y

-1

1

1 x

y

(a) (b) ( ) (d)Exer í io 16. Faça um esboço de um grá o de uma função f tal que limx→1−

f(x) = 2,

f(1) = 1 e, além disso (um grá o para ada item):(a) limx→1+

f(x) = −2, (b) limx→1+

f(x) não exista, ( ) limx→1+

f(x) = +∞,Exer í io 17.Determine os limites:(a) limx→0

|x| sen(1/x); (b) limh→0

sen(3h)

h; ( ) lim

x→+∞(1 + 1/x)5x;(d) lim

x→π/2+tan(x); (e) lim

x→0+(1− 2x)1/x.Exer í io 18.Estude o Teorema 4 da p.33 (Sanduí he) e responda:(a) É verdade que se 1 ≤ g(x) ≤ 2 então lim

x→3/2g(x) existe e é um número entre 1 e 2?(b) Explique, utilizando o Teorema do Sanduí he, omo al ular lim

x→+∞

cos(√x2 + 1)

x2.

Page 8: Sumario´ - labMA/UFRJmcabral/livros/livro-calculo/... · 2019. 6. 12. · Sumario´ 1 Enunciados dos Exercícios 1.1 Exercícios de Limite. 1 1.1.1 Exercícios de Fixação. 1 1.1.2

4 CAPÍTULO 1. ENUNCIADOS DOS EXERCÍCIOS1.1.2 ProblemasProblema 1.Esbo e o grá o das seguintes funções:(a) f(x) = −√9− x2; |x| ≤ 3

|x| − 3; |x| > 3.(b) f(x) = √x− 1; x ≥ 1;

log(x) + 1; x < 1.Problema 2.Considere a função IZ ( hamada de função ara terísti a ou indi adora do onjunto Z) denida por IZ(x) = 0; x 6∈ Z

1; x ∈ Z.Esbo e o grá o e determine (se existir):(a) lim

x→3/4IZ(x); (b) lim

x→−3IZ(x); ( ) lim

x→+∞IZ(x).Problema 3.Cal ule os limites abaixo (quando eles existirem) justi ando seus passos (semutilizar a regra de L'Hospital) Limites om raízes:(a) lim

h→0

√1 + h−

√1− h

h(b) lim

x→4

|x| − 4√x− 2

; ( ) limh→−1

√h2 + 3− 2

h + 1;Problema 4.Determine os limites e, aso não exista, os limites laterais ( aso existam).(a) lim

x→−3sen

(

7

x+ 3

); (b) limx→2

log |x− 2|;( ) limx→2

|x− 2|(x+ 1)

x− 2; (d) lim

x→−5

x+ 3

x+ 5.Problema 5.Cal ule os limites abaixo (quando eles existirem) justi ando seus passos (semutilizar a regra de L'Hospital):(a) lim

x→2−

x

x2 − 4; (b) lim

x→1+

x+ 3

1− x( ) lim

x→0

(

1

x− 1

x2

)

; (d) limx→2−

|x− 2|x2 − 5x+ 6

;(e) limx→−2

x+ 2

|x| − 2; (f) lim

a→2

(a− 2)(a2 − 4)

a3 − 5a2 + 8a− 4; (g) lim

x→2

x2 − 3x+ 2

x2 − 3x+ 5;(h) lim

x→1

x3 − x

x2 − 3x+ 2; (i) lim

x→2

x2 + 3x− 1

x2 + 2x− 1; (j) lim

x→1

x+ 1− 2x

x− 1;(k) lim

x→−1

x2 + 2x+ 1

x+ 1(l) lim

x→−1

x3 + 1

x+ 1; (m) lim

x→1

2x2 − 3x+ 1

x− 1;Problema 6.Cal ule os limites abaixo (quando eles existirem) justi ando seus passos (semutilizar a regra de L'Hospital) Limites no innito:(a) lim

x→−∞

√x2 + 1

x+ 1; (b) lim

y→+∞

7− 2y√

5− 2y + 9y2; ( ) lim

x→+∞

√10x4 + 3x3 + 2x+ 5

5x2 − 10x− 100;(d) lim

x→+∞

√x2 + 1√x+ 1

; (e) limy→−∞

5− 3y3√

8− y + 10y4; (f) lim

x→−∞sen

(√16x6 − x+ 1

2x3 − x2 + 20

).Problema 7.Considere a, b ∈ R e c > 0. Determine os limites:(a) limx→0

(1+ax)b/x; (b) limx→−∞

(√cx2 + a−

√cx2 + b

); ( ) limx→+∞

(√cx2 + ax− bx

);

Page 9: Sumario´ - labMA/UFRJmcabral/livros/livro-calculo/... · 2019. 6. 12. · Sumario´ 1 Enunciados dos Exercícios 1.1 Exercícios de Limite. 1 1.1.1 Exercícios de Fixação. 1 1.1.2

1.1. EXERCÍCIOS DE LIMITE 5(d) limx→+∞

(√cx2 + ax−

√cx2 + bx

); (e) limx→−∞

(√cx2 + ax−

√cx2 + bx

).Problema 8.Considere os polinmiosp(x) = axm + x2 − 3x + 1, q(x) = bxm + 2x5 − 4, r(x) = cx2m + 3x7 + 2 om m > 10,a, b 6= 0 e c > 0. Determine os limites:(a) lim

x→+∞

p(x)

r(x)(b) lim

x→+∞

q(x)

p(x)( ) lim

x→+∞

r(x)

x2p(x)(d) limx→+∞

xmp(x)

r(x)(e) lim

x→+∞

r(x)

p(x)(f) lim

x→+∞

r(x)

xq(x)Problema 9.Determine os limites laterais quando x → 0 para:(a) h(x) = 1

1 + e1/x; (b) h(x) = 1

x− 1

|x| .Problema 10. Sabendo que o quadro de sinais de f(x) é dado pela tabela abaixo e quelim

x→+∞f(x) = 4 e lim

x→−∞f(x) = +∞, esbo e o grá o de f(x) e determine TODAS asassíntotas verti ais e horizontais.

−3 −2 3 40 ±∞ 0 ±∞

f(x) + − − − +Problema 11.Esbo e o grá o de ada uma das funções abaixo seguindo o roteiro abaixo.(i) Faça um estudo do sinal da função (onde ela é zero, positiva e negativa).(ii) Determine assíntotas horizontais e verti ais.(iii) Baseado em (i) e (ii) esbo e o grá o.(a) y =x2 − 1

x− 1; (b) y =

1

x2 − 1; ( ) y =

x

x2 + 1(d) y =x2 − 1

x(x− 2); (e) y =

3x2 − 3

4− x2;Problema 12.Considere a função h(x) =

x; x ∈ Q

−x; x 6∈ Q.Esbo e o grá o e determine (seexistir):(a) lim

x→πh(x); (b) lim

x→1h(x); ( ) lim

x→0+

h(x)

x2; (d) lim

x→+∞

h(x)

x2; (e) lim

x→0

h(x)

x.Problema 13. (a) Suponha que h satisfaz √

x

x3 + x≤ h(x) ≤ x

x2 + 1. Determine lim

x→+∞h(x).(b) Suponha que f(x) satisfaz |f(x)− 3| ≤ 2|x− 5|4. Cal ule lim

x→5f(x).Problema 14.Cal ule os limites abaixo (quando eles existirem) justi ando seus passos (semutilizar a regra de L'Hospital): Limites trigonométri os e exponen iais.(a) lim

x→0

(tan(3x))2 + sen(11x2)

x sen(5x); (b) lim

x→+∞3x2sen

(

1

x2

); ( ) limx→0

cosx− cos3 x

3x2;

Page 10: Sumario´ - labMA/UFRJmcabral/livros/livro-calculo/... · 2019. 6. 12. · Sumario´ 1 Enunciados dos Exercícios 1.1 Exercícios de Limite. 1 1.1.1 Exercícios de Fixação. 1 1.1.2

6 CAPÍTULO 1. ENUNCIADOS DOS EXERCÍCIOS(d) limh→0+

sen(√h) tan(2

√h)

5h; (e) lim

x→1sen

(

7x+ 1

sen(πx/2)− 1

)

(ex−1 − 1);(f) limh→0+

(1− 5h3)2/h3 ; (g) lim

x→π

sen x

x− π; (h) lim

x→0

sen x

|x| .1.1.3 ExtrasExtra 1.Partindo de grá o de funções simples (±x2,±1/x,±1/x2,

√x, sen(x), |x|), utili-zando translações verti ais e/ou horizontais e/ou reexões, esbo e o grá o de:(a) y = | sen(x)| − 1; (b) y = ||x| − 1|; ( ) y = |x+ 2| − 1.Extra 2. Faça um esboço de um grá o de uma função f tal que, simultaneamente:

limx→−∞

f(x) = 4, limx→+∞

f(x) = −∞, limx→1−

f(x) = −∞, f(1) = 1, limx→1+

f(x) = −2.Extra 3.Determine limx→0

sengr(x)

x, onde sengr é a função seno do ângulo x medido em graus.Note que para a função seno utilizada em ál ulo, o ângulo é medido em radianos.Extra 4.Esbo e o grá o de:(a) y = x+ |x|; (b) x− ⌊x⌋.Extra 5.Determine os limites:(a) lim

x→1

|x| − 1

|x− 1| ; (b) limx→1

x3 + 1

(x− 1)2; ( ) lim

x→2

x2 + 2x

x3 − x; (d) lim

x→πcos

(

1

x− π

)

(x−π).Extra 6.Determine os limites:(a) limx→+∞

(√x4 + x− x2

); (b) limx→+∞

2x+ |x|x+ 1

; ( ) limx→−∞

2x+ |x|x+ 1

;(d) limx→−∞

x+ 1

x+ |x|+ 1.Extra 7.Considere a ∈ R. Determine os limites:(a) lim

x→+∞

(√x+ a−

√x); (b) lim

x→+∞

(√x2 + a− x

).Extra 8.Esbo e o grá o das seguintes funções:(a) f(x) = 1; x ∈ Q;

2; x 6∈ Q; (b) g(x) = x; x ∈ Q;

x2; x 6∈ Q;

Page 11: Sumario´ - labMA/UFRJmcabral/livros/livro-calculo/... · 2019. 6. 12. · Sumario´ 1 Enunciados dos Exercícios 1.1 Exercícios de Limite. 1 1.1.1 Exercícios de Fixação. 1 1.1.2

1.1. EXERCÍCIOS DE LIMITE 71.1.4 DesaosDesao 1.A função parte inteira de x, denotada por ⌊x⌋ é denida na p. 12.(a) Cal ule, se existir: limx→∞

x

1

x

. (b) Cal ule, se existir: limx→−∞

x

1

x

.( ) Esbo e o grá o de f(x) = x

1

x

⌋. (d) Cal ule, se existir: limx→0

x

1

x

.Desao 2.Determine:(a) limx→+∞

(ex + x)1/x. (b) limx→+∞

(1 + x)α/ logx, om α 6= 0.Desao 3. Como al ular assíntotas oblíquas e generalizações?Dividindo os polinmios e separando em quo iente e resto.Assim, x2 − 3x+ 2

x− 1= q(x) +

r

x− 1. Para x grande, x2 − 3x+ 2

x− 1≈ q(x), sua assíntotaoblíqua. Plote uns grá os para ver omo de fato se pare em. O mesmo o orre quando adiferença entre os graus do numerador e denominador é maior que 1.Desao 4.Determine lim

x→0+

1

x sen(1/x). Tente esboçar o grá o perto do zero desta função.Utilize algum software para isso.Desao 5. (Cari atura de sen(1/x) do livro do Spivak de Cál ulo) Esbo e o grá o da função

f que satisfaz:(i) f(1/n) = (−1)(n+1),(ii) f é linear entre [1/(n+ 1), 1/n] (segmento de reta),(iii) f(x) = 1 para x > 1,(iv) f(−x) = f(x).Desao 6.Prove que a área do ír ulo de raio r é πr2 seguindo o seguinte roteiro:(a) Mostre que a área do polígono de n-lados ins rito no ír ulo é n

2r2 sen(2π/n).(b) Mostre que a área do polígono de n-lados ir uns rito no ír ulo é nr2 tan(π/n).( ) Faça n → +∞ e on lua o argumento.Desao 7. Sejam f e g duas funções tais que |f(x)| ≤ M para todo x ∈ R e lim

x→1g(x) = 0.Mostre que

limx→1

f(x)g(x) = 0.

Desao 8.Objetivo desta atividade é aproximar a função fatorial. É fá il ver que ()

n! =

(

1

2

)(

2

3

)2(3

4

)3(4

5

)4

· · ·(

n− 1

n

)n−1

nn.

Page 12: Sumario´ - labMA/UFRJmcabral/livros/livro-calculo/... · 2019. 6. 12. · Sumario´ 1 Enunciados dos Exercícios 1.1 Exercícios de Limite. 1 1.1.1 Exercícios de Fixação. 1 1.1.2

8 CAPÍTULO 1. ENUNCIADOS DOS EXERCÍCIOSLogo n! = nn

n−1∏

j=1

(

j

j + 1

)j

= nn/

n−1∏

j=1

(1 + 1/j)j. Já sabemos que o termo (1 + 1/j)jtende para e quando j tende para innito. Portanto n! ≈ nn/en−1 = e(n/e)n (vide [Fe).Utilizando esta aproximação, determine os limites, quando n vai para innito, de:(a) n!

n; (b) n!

n5; ( ) n!

en; (d) n!

nn/2; (e) n!

nn.Obs: Podemos denir fatorial de não-inteiros (e até mesmo de omplexos) om a funçãogama de Euler.Obs: Utilizando outro aminho (vide [C p.361364 ou [Sp p.483) obtemos a fórmulade Stirling2: n! = √

2πn(n/e)neθn om |θ| ≤ 1/12.Desao 9. Dena o número e por e =∞∑

n=0

1

n!e prove que e 6∈ Q (é irra ional) seguindo oroteiro abaixo.(a) Suponha por absurdo existem p, q ∈ N tais que e = p/q. Mostre que

p(q − 1)!−q∑

n=0

q!

n!=

+∞∑

n=q+1

q!

n!.Di a: Multiplique e por q!.(b) Mostre que o lado esquerdo da igualdade em (a) é um inteiro.( ) Mostre que o lado direito da igualdade em (a) é igual a um número entre 0 e 1.Di a: Simplique o fatorial e ompare om a PG de razão 1/2.(d) Con lua a prova mostrando que (b) + ( ) ontradiz (a).

1.2 Exer í ios de Continuidade1.2.1 Exer í ios de FixaçãoExer í io 1.Determine se é Verdadeiro (provando a armativa) ou Falso (dando um ontra-exemplo):(a) Se limx→a

f(x) existe, então f é ontínua em a;(b) Se f é ontínua em a, então limx→a−

f(x) existe.( ) Se f é des ontínua em a, então limx→a−

f(x) 6= limx→a+

f(x).Exer í io 2.(a) Determine se f esboçada no grá o abaixo é ontínua ou não nos pontos A,B,C,D.(b) Explique, aso não seja ontínua, qual (quais) ondições são violadas.( ) Determine os pontos de des ontinuidade removível2James Stirling: ⋆ 05/1692, Garden, Es ó ia † 05/12/1770, Edinburgh, Es ó ia.212.set.2011

Page 13: Sumario´ - labMA/UFRJmcabral/livros/livro-calculo/... · 2019. 6. 12. · Sumario´ 1 Enunciados dos Exercícios 1.1 Exercícios de Limite. 1 1.1.1 Exercícios de Fixação. 1 1.1.2

1.2. EXERCÍCIOS DE CONTINUIDADE 9x

y

A B C DExer í io 3.Considere as funções abaixo:(I) f(x) = x; x < 0;

0; x ≥ 0;(II) g(x) = x; x < 0;

1; x ≥ 0;(III) h(x) = 5; x ≥ −2;

4; x < −2;Determine se são ontínuas em:(a) R; (b) (−2, 0); ( ) [−2, 0].Exer í io 4.Esbo e o grá o de uma função ontínua ujos pontos de des ontinuidade(úni os pontos onde a função não é ontínua) são:(a) 1, 2, 3; (b) N = 1, 2, . . ..Exer í io 5.Determine um valor para k ∈ R, se for possível, de modo que a função seja ontínua em R.(a) f(x) =

1

x; x 6= 0;

k; x = 0;(b) f(x) =

1

x2; x 6= 0;

k; x = 0;( ) f(x) =

x sen(

1x

)

; x 6= 0;

k; x = 0;Exer í io 6. Seja f uma função ontínua em [1, 4] tal que f(1) = 2, f(2) = 3, f(3) =−1 e f(4) = 2. Determine se é Verdadeiro (provando a armativa) ou Falso (dando um ontraexemplo):(a) f não tem raiz em [1, 2]; (b) f tem pelo menos duas raízes em [1, 4];( ) f tem exatamente uma raiz em [2, 3].Exer í io 7.Determine se é Verdadeiro (provando a armativa) ou Falso (dando um ontra-exemplo):(a) a função que representa o número de habitantes de uma idade em função do tempoé ontínua em todos os pontos;(b) a função que representa a altura de uma pessoa em função do tempo é ontínua emtodos os pontos;Exer í io 8.Estude o Teorema 10 da p.53 (TVI) e determine se é Verdadeiro (provando aarmativa) ou Falso (dando um ontraexemplo):(a) se f é ontínua om f(0) > 0 e f(1) > 0, então f(x) > 0 para todo x ∈ [0, 1];(b) Se f(1) < 0 < f(2), então f possui raiz em [0, 1].Exer í io 9.Estude o Teorema 10 da p.53 (TVI). Considere f : [−3,−1] → R ontínua omf(−3) = 5 e f(−1) = 2. Determine se esta orreto ou orrija.

Page 14: Sumario´ - labMA/UFRJmcabral/livros/livro-calculo/... · 2019. 6. 12. · Sumario´ 1 Enunciados dos Exercícios 1.1 Exercícios de Limite. 1 1.1.1 Exercícios de Fixação. 1 1.1.2

10 CAPÍTULO 1. ENUNCIADOS DOS EXERCÍCIOS(a) se K ∈ [−3,−1], então existe c ∈ [2, 5] tal que f(c) = K;(b) se K ∈ [3, 4], então existe c ∈ [−3,−1] tal que f(c) = K;( ) se K ∈ [0, 3], então existe c ∈ [−3,−1] tal que f(c) = K;Exer í io 10.Estude o Lema 3 da p.52 e o Teorema 7 da p.52. Supondo que f é ontínua,prove, fazendo referên ia somente ao Lema 3 e o Teorema 7, que h(x) = 5[f(x)]3

x2 + 1é ontínua.

1.2.2 ProblemasProblema 1.Determine o onjunto dos pontos de des ontinuidade (úni os pontos onde afunção não é ontínua) de:(a) f(x) =

1

sen(x); x 6= kπ; k ∈ Z

1; x = kπ;(b) g(x) = 1

2 + cos(x);( ) h(x) = x− ⌊x⌋; (d) j(x) = x; x ∈ Q;

x3; x 6∈ Q.Problema 2.Determine se f(x) =

|x+ 2|; x < 0;

3; x = 0;

3− x; x > 0.

é ontínua e al ule limx→−∞

f(x).Problema 3.(a) Seja f(x) = x4 − 2x3 + x2 + 7 sen(x). Mostre que existe a ∈ R tal que f(a) = 10;(b) Mostre que existe pelo menos um b > 0 tal que log(b) = e−b;( ) Considere f ontínua em [0, 1] om 0 ≤ f(x) ≤ 1. Mostre que existe c ∈ [0, 1] talque f(c) = c;(d) Suponha que f é ontínua em [0, 2] om f(1) = −3 e f(x) 6= 0 para todo x ∈ [0, 2].Prove que f(x) < 0 para todo x ∈ [0, 2].Problema 4.Determine um valor para a ∈ R, se for possível, de modo que a função seja ontínua em R.(a) f(x) =

(x− 2)2(x+ a)

x2 − 4 x+ 4; x 6= 2

7; x = 2.(b) f(x) =

2x+ 5 se x < −1,

a se x = −1,

x2 − 3 se x > −1.( ) f(x) =

x

|x| ; |x| ≥ 1

ax; |x| < 1.(d) f(x) =

sen(

1x

)

; x 6= 0;

a; x = 0;(e) f(x) = e1/x; x > 0

a; x ≤ 0.(f) f(x) =

sen(6x)

sen(8x); x 6= 0;

a; x = 0..Problema 5.Determine a, b ∈ R, se for possível, de modo que f seja ontínua em R.

Page 15: Sumario´ - labMA/UFRJmcabral/livros/livro-calculo/... · 2019. 6. 12. · Sumario´ 1 Enunciados dos Exercícios 1.1 Exercícios de Limite. 1 1.1.1 Exercícios de Fixação. 1 1.1.2

1.2. EXERCÍCIOS DE CONTINUIDADE 11f(x) =

ax+ b; |x| ≤ 2;

|x− 1|; |x| > 2.Problema 6. Suponha que f : R → R é ontínua e f(x) ∈ Q para todo x ∈ R. Prove que

f(x) é onstante para todo x ∈ R.1.2.3 ExtrasExtra 1.Determine o onjunto dos pontos de des ontinuidade (úni os pontos onde a funçãonão é ontínua) de f(x) =

1, x ∈ Q,

1 + |x|, x 6∈ Q.Extra 2.Determine um valor para a ∈ R, se for possível, de modo que a função seja ontínuaem R.(a) f(x) =

x3 − 4x2 + 5x− 2

x3 − 2x2 + xse x 6= 1,

a se x = 1.(b) f(x) =

x2 + 2 se x < 0,

a se x = 0,√x+ 4 se x > 0.( ) f(x) =

x3 + 1

x− 1; x 6= 1

a; x = 1. (d) f(x) = e1/x; x < 0

a; x ≥ 0.(e) f(x) = 2x+ a; x ≤ 1;

x2/a; x > 1. (f) f(x) =

a x; x < 0;

1; x ≥ 0;Extra 3.Determine se é Verdadeiro (provando a armativa) ou Falso (dando um ontrae-xemplo):(a) se f é ontínua om f(0) = 2 e f(3) = 5, então f(x) > 0 para todo x ∈ [0, 3].(b) se g é ontínua om g(1) = g(3) = −10 e g(2) = 10, então g possui exatamenteduas raízes no intervalo [1, 3];Extra 4. (Apli ação do TVI)(a) Mostre que existe pelo menos um x0 ∈ R tal que x0 + 2 sen(x0) = 1.(b) Mostre que todo polinmio de grau impar possui pelo menos uma raiz.( ) Mostre que a equação sen(π sen(x)) = sen(x) possui pelo menos uma solução em[π/6, π/2].(d) Considere h(x) = sen(x) + 1 − 2

π|x|. Prove que existem x0, x1 ∈ R distintos taisque h(x0) = h(x1) = 0.Extra 5.Determine a, b ∈ R, se for possível, de modo que f seja ontínua em R.

f(x) =

−2x; x ≥ 4;

ax+ b; 1 < x < 4;

x; x ≤ 1.

Page 16: Sumario´ - labMA/UFRJmcabral/livros/livro-calculo/... · 2019. 6. 12. · Sumario´ 1 Enunciados dos Exercícios 1.1 Exercícios de Limite. 1 1.1.1 Exercícios de Fixação. 1 1.1.2

12 CAPÍTULO 1. ENUNCIADOS DOS EXERCÍCIOSExtra 6. Prove que:(a) cosh2(x)− senh2(x) = 1;(b) senh(a+ b) = senh a cosh b+ senh b cosh a;( ) cosh(a+ b) = cosh a cosh b+ senh a senh b.1.2.4 DesaosDesao 1.Um montanhista ini ia a subida do Pi o das Agulhas Negras do abrigo Rebouçasas 8h da manhã e atinge o pi o as 15h deste dia. Ele dorme no pi o e retorna na manhãseguinte as 8h, hegando de volta ao abrigo Rebouças as 15h do mesmo dia.Mostre que ele passou por um ponto do per urso na mesma hora (em dias distintos)durante a subida e durante a des ida.Desao 2.Esbo e o grá o e determine os pontos de des ontinuidade de:(a) f(x) igual ao segundo dígito da expansão de imal de x.(b) f(x) igual ao número de 7's da expansão de imal de x se este número é nito e zero aso ontrário.( ) f(x) = 0 se x ∈ R − Q, f(p/q) = 1/q se p/q é fração irredutível om q > 0 ef(0) = 0; Di a: esbo e o grá o para q = 2, 3, . . .(d) f(x) = 0 se 1 não apare e na expansão de imal de x e f(x) = n se 1 apare e naenésima posição.Desao 3.En ontre uma função f que seja des ontínua nos seguintes pontos, mas ontínuaem todos os outros:(a) 1, 1

2, 13, 14, . . .; (b) 0, 1, 1

2, 13, 14, . . .Desao 4. Suponha que lim

x→1log x = 0. Prove que log(x) é ontínua para x > 0.Desao 5. Prove (veja outra prova no Desao da p.19), utilizando as séries da exponen ial(p.58) e do seno e osseno (p.58), a relação de Euler:

eiθ = cos θ + i sen θ.

Desao 6. Utilizando a relação de Euler eiθ = cos θ + i sen θ e a denição de senh e coshdadas na p.60, prove que:senh(ix) = i sen(x) e cosh(ix) = cos(x).Tome x = iθ e prove quecos(iθ) = cosh(θ) e sen(iθ) = i senh(θ).

Page 17: Sumario´ - labMA/UFRJmcabral/livros/livro-calculo/... · 2019. 6. 12. · Sumario´ 1 Enunciados dos Exercícios 1.1 Exercícios de Limite. 1 1.1.1 Exercícios de Fixação. 1 1.1.2

1.3. EXERCÍCIOS DE DERIVADA 13Desao 7.Dizemos que J é um intervalo em R se J é igual a [a, b] ou (c, d) ou [a, d) ou(c, b] om a, b ∈ R e c, d ∈ R ∪ −∞,+∞.Prove que se f é ontínua em um intervalo I então a imagem f(I) é um intervalo.Dizemos que função ontínua leva intervalo em intervalo.Desao 8.Adapte a denição de lim

x→cf(x) = L e dena:(a) lim

x→c+f(x) = −∞; (b) lim

x→−∞f(x) = +∞; ( ) lim

x→c−f(x) = L.

1.3 Exer í ios de Derivada1.3.1 Exer í ios de FixaçãoExer í io 1.Determine a equação da reta tangente ao grá o de f(x) no ponto x = −2sabendo que f(−2) = 3 e f ′(−2) = 3.Exer í io 2.Determine se é Verdadeiro ou Falso. Se for falso dê um ontraexemplo ou orrija.(a) Se f é ontínua em x = 3, então f é derivável em x = 3.(b) Se f(2) = g(2), então f ′(2) = g′(2).( ) Se f ′(1) > 0, então f(1) > 0.Exer í io 3.Considere o grá o de f abaixo.(a) se f ′(x1) = 2 determine f ′(x2) e f ′(x3).(b) Coloque em ordem res ente f ′(x2), f′(x4), f

′(x5), f′(x6).

x

y

x1 x2 x3 x4 x5 x6

f(x)

Exer í io 4.Dado o grá o de f abaixo, faça o grá o exato de f ′.207.out.2011

Page 18: Sumario´ - labMA/UFRJmcabral/livros/livro-calculo/... · 2019. 6. 12. · Sumario´ 1 Enunciados dos Exercícios 1.1 Exercícios de Limite. 1 1.1.1 Exercícios de Fixação. 1 1.1.2

14 CAPÍTULO 1. ENUNCIADOS DOS EXERCÍCIOSx

y

f(x)

4 12

3

−2Exer í io 5. Se f e g são funções diferen iáveis tais que f(2) = 3, f ′(2) = −1, g(2) = −5,g′(2) = 2, determine o valor de h′(2) se:(a) h(x) = f(x)g(x); (b) h(x) = f(x)

g(x).Exer í io 6.Considere f e g duas funções ujos grá os estão na gura abaixo. As retasque apare em são tangentes ao grá o. Determine o valor de h′(1) se:(a) h(x) = f(x) · g(x); (b) h(x) = 5f(x)− 3g(x).

-

6

x

y

1

1

f(x)

-

6

x

y

3

2

1

g(x)

Exer í io 7. Se um balonista joga um sa o de areia de um balão a 500m de altura então osa o de areia estará numa altura (em metros) h(t) = 500−16t2 após t segundos. Determine:(a) sua velo idade em t = 2;(b) em qual instante t o sa o atingirá o solo;( ) om qual velo idade o sa o atingirá o solo;(d) om qual a eleração o sa o atingirá o solo.Exer í io 8.Cal ule a derivada em relação a x das funções:(a) ex log x; (b) cosx

x+ 5( ) cos(x3 + 1);(d) eπ + log(π2 + 1). (e) log(1 + sen x); (f) |x− 2|;Exer í io 9.Cal ule:

Page 19: Sumario´ - labMA/UFRJmcabral/livros/livro-calculo/... · 2019. 6. 12. · Sumario´ 1 Enunciados dos Exercícios 1.1 Exercícios de Limite. 1 1.1.1 Exercícios de Fixação. 1 1.1.2

1.3. EXERCÍCIOS DE DERIVADA 15(a) d

dr

(

4

3πr3); (b) d

dk(3k2 − k−1); ( ) du

dtse u = t log t;(d) dv

dsse v = sπ; (e) dy

dxse y = (

√3)x; (f) d

dt(log π).Exer í io 10.Estude o Teorema do Valor Médio (Corolário 5 da p.84) e responda. Suponhaque f é derivável em R e −4 ≤ f ′(x) ≤ 3 para todo x ∈ R. Prove que:(a) −16 ≤ f(5)− f(1) ≤ 12; (b) −4h ≤ f(h)− f(0) ≤ 3h para todo h > 0.Exer í io 11.Um objeto ai do alto de um edifí io de 100m e atinge o solo em 5 segundos.Aplique o Teorema do Valor Médio (TVM) e prove que em algum instante o objeto estava om velo idade (em módulo) igual a 20m/s.Exer í io 12. Suponha que f ′′(x) = 0 para todo x ∈ R. Sabendo que f ′(−3) = 0 e

f(5) = π, aplique uma onsequên ia do Teorema do Valor Médio (TVM) duas vezes para on luir que f(x) = π para todo x ∈ R.Exer í io 13.Considere f e g duas funções ujos grá os estão na gura abaixo. As retasque apare em são tangentes ao grá o.(a) Se h(x) = f(g(x)), determine h′(2).(b) Se k(y) = g−1(y), determine k′(3).-

6

x

y

QQQQQQQQQQQ

QQ

QQQ

3

2f(x)

-

6

x

y

2

2

3

g(x)

Exer í io 14.Considere o grá o abaixo.x

y

Se o grá o representa f(x) determine maiores intervalos (indique no grá o) onde:(a) f ′ é positiva e negativa; (b) f é injetiva (possui inversa).

Page 20: Sumario´ - labMA/UFRJmcabral/livros/livro-calculo/... · 2019. 6. 12. · Sumario´ 1 Enunciados dos Exercícios 1.1 Exercícios de Limite. 1 1.1.1 Exercícios de Fixação. 1 1.1.2

16 CAPÍTULO 1. ENUNCIADOS DOS EXERCÍCIOSSe o grá o representa f ′(x) determine maiores intervalos (indique no grá o) onde:( ) f é res ente e de res ente; (d) f é injetiva (possui inversa).Exer í io 15.Prove que. para a > 0,cos(arcsen(x/a)) =

1

a

√a2 − x2.

1.3.2 ProblemasProblema 1.Cal ule, pela denição (utilizando limite), a derivada de:(a) f(x) = 1

x2; (b) f(x) = 1√

x; ( ) f(x) = |x|(x− 1); (d) f(x) = |x|x.Problema 2.Determine a, b ∈ R tais que a função abaixo tenha derivada em todos os pontos.

f(x) =

x2; x < 1;

ax+ b; x ≥ 1.Problema 3. Suponha que |f(x)| ≤ |x|k om k > 1. Cal ule pela denição f ′(0).Di a: Veja o Exemplo 58 da p.72.Problema 4.Para ada uma das funções abaixo, determine onde possui derivada e al ule aderivada nestes pontos.(a) g(x) = 3; x < 2;

−4; x ≥ 2;(b) f(x) = |ex − 1|; ( ) h(x) = |(3− x)(x+ 1)|.Problema 5.Em ada um dos itens abaixo, s(t) representa a posição de uma partí ula semovendo em linha reta no instante t. Determine:(i) A velo idade e a eleração da partí ula no instante t = 0.(ii) Os instantes em que a partí ula está parada.(a) s(t) = t2 − 1

t2 + 1; (b) s(t) = sen t.Problema 6.Considere a função f(x) = 2x3 − 2x2 + 5. Determine todos os pontos dográ o de f nos quais a reta tangente é:(a) horizontal; (b) paralela à reta 2y − 20x − 50 = 0 ( ) perpendi ular à reta

8y + 2x− 10 = 0.Problema 7.Determine TODOS os pontos do grá o de y = f(x) = |x2 − 1|(x+ 1) ondea reta tangente é paralela ao eixo x

Page 21: Sumario´ - labMA/UFRJmcabral/livros/livro-calculo/... · 2019. 6. 12. · Sumario´ 1 Enunciados dos Exercícios 1.1 Exercícios de Limite. 1 1.1.1 Exercícios de Fixação. 1 1.1.2

1.3. EXERCÍCIOS DE DERIVADA 17Problema 8.Determine ondições sobre a, b, c ∈ R para que a urva:(a) y = ax3 + bx2 + cx+ π tenha uma úni a reta tangente horizontal;(b) y = ax2 + bx + c tenha x + y = 1 e y = −1 respe tivamente omo retas tangentesnos pontos x1 = −1 e x2 = 1.Problema 9.Cal ule as derivada (em relação a x) das funções:(a) (5x2 − 3x+ 4)300; (b) sen( 7√

cos(x2) + 4); ( ) x2 + 1

e−x + 1; (d) 3

√x+ t

x2 + k;(e) log(sen(5ex)) · x4; (f) arctan(log(3x2 + 1)); (g) earcsen(4−5x).Problema 10.Dado que f(4) = 3, f ′(4) = −5 e g(x) = 3 log(f(x) + x), determine g′(4).Problema 11.Considere m0, T0, K, a, b, c, d ∈ R. Cal ule:(a) f ′(x) se f(x) =

(

ax+ b

cx+ d

)2; (b) f ′(t) se f(t) = eKt cos(at).( ) f ′(θ) se f(θ) = K sen(aθ3 + b); (d) f ′′′(t) se f(t) = m0e(T0−t)/K ;Problema 12.Determine a equação da reta tangente e da reta perpendi ular ao grá o de:(a) y =

sen(x2)

xpara x =

π/2; (b) y = esen(−2x) no ponto (π, 1).Problema 13.Mostre que:(a) ex ≥ 1 + x para x ≥ 0.(b) a equação 2x3 − 15x2 + 60x+ 4 = 0 possui exatamente uma raiz real.Problema 14. (Apli ações do Teorema do Valor Médio)(a) Dois orredores ini iaram a orrida no mesmo instante e terminaram empatados. Proveque em algum instante durante a orrida ele têm a mesma velo idade.(b) Considere f diferen iável om f(0) = 0 e f ′(x) ≤ 1 para todo x > 0. Mostre quef(x) ≤ x para todo x > 0.( ) Mostre que existe uma úni a h : R → R diferen iável tal que: h′(x) = h(x);

h(0) = 1.Di a: Suponha que h1 e h2 são soluções. Dena f(x) =h1(x)

h2(x), al ule f ′(x) e f(0).(d) Considere f(x) = x2ex e g(x) = e

√x. Prove que existe um c ∈ (0, 1) tal que asretas tangentes ao grá o de f e de g são paralelas em x = c.Problema 15. Se f e g são funções diferen iáveis tais que f ′(2) = −1, f(2) = 3, g(−1) = 2,

g′(−1) = 6, determine o valor de h′(2) se:(a) h(x) = f(g(−x/2)); (b) h(y) = g−1(y).Problema 16. Sabendo que a equação da reta tangente ao grá o de y = f(x) no ponto(−1, 3) passa no ponto (0, 6), determine (f−1)′(3).

Page 22: Sumario´ - labMA/UFRJmcabral/livros/livro-calculo/... · 2019. 6. 12. · Sumario´ 1 Enunciados dos Exercícios 1.1 Exercícios de Limite. 1 1.1.1 Exercícios de Fixação. 1 1.1.2

18 CAPÍTULO 1. ENUNCIADOS DOS EXERCÍCIOS1.3.3 ExtrasExtra 1. Se f e g possuem derivada e são tais que: f(2) = 3, f ′(2) = −1, g(2) = −5,g′(2) = 2, determine:(a) w′(2) se w(x) =

4f(x)

g(x); (b) m′(0) se m(x) = e5xg(3x+ 2).Extra 2.Cal ule a derivada (em relação a x) das funções:(a) sen(x ex log x); (b) sen(sen(sen x))); ( ) 3arctan x; (d) √x+

√x;(e) sen(cosx sen x); (f) sen |1− x2|; (g) eex4 ; (h) log(sen(2x))√x2 + 1.Extra 3.Determine todos os pontos do grá o de y(x) = x3 + 2x2 − 4x + 5 nos quais areta tangente:(a) é horizontal; (b) é paralela à reta 2y + 8x− 5 = 0.Extra 4.Determine a, b ∈ R tais que a função abaixo tenham derivada em todos os pontos.

f(x) =

ax2 + b; x ≤ 1;1

x; x > 1

.Extra 5. Sabendo que g é ontínua em a e f(x) = (x− a)g(x), determine f ′(a).Extra 6.Determine a equação da reta tangente ao grá o da função no ponto indi ado.(a) y = x2 sen x no ponto (π, 0). (b) y = log(√x− 2), no ponto ((e + 2)2, 1).Extra 7.Determine:(a) os pontos da urva y =

1

xnos quais a reta tangente é paralela à reta 2x+ 3y = 0;(b) a(s) reta(s) tangente(s) ao grá o de y = e2x que ontem(êm) o ponto (5/2, 0).Extra 8. (Função res ente/de res ente) Mostre que:(a) x > log x para x > 1;(b) a equação −2x13 − 6x5 − x+ 10 = 0 possui exatamente uma raiz real.Extra 9. (Apli ações do Teorema do Valor Médio)(a) Suponha que f é uma função diferen iável em R e tenha duas raízes reais distintas.Mostre que f ′ tem, no mínimo, uma raiz real.(b) Considere uma função f diferen iável om f ′(x) ≤ 4 para todo x ∈ (2, 5). Prove que

f(5)− f(2) ≤ 12.( ) Mostre que existe uma úni a g função derivável em R tal que:g′′(x) = cos(2x+ log(x4 + 1)), g′(2) = −1 e g(3) = 5.Di a: Suponha que g1 e g2 são soluções. Dena f(x) = g1(x) − g2(x), al ule f ′′(x) e

f ′(2). Con lua que f ′(x) = · · · . Depois al ule f(3).

Page 23: Sumario´ - labMA/UFRJmcabral/livros/livro-calculo/... · 2019. 6. 12. · Sumario´ 1 Enunciados dos Exercícios 1.1 Exercícios de Limite. 1 1.1.1 Exercícios de Fixação. 1 1.1.2

1.3. EXERCÍCIOS DE DERIVADA 19Extra 10.Considere f(x) =

|x|k; x ∈ Q;

0; x 6∈ Q om k > 1. Cal ule pela denição f ′(0).Di a: Veja o Exemplo 58 da p.72.Extra 11.Determine uma fórmula para a derivada (fgh)′.Extra 12. Seja f(x) = cos(x5 + 2x + π/2). Sabendo que f(0) = 0, e que g(y) é a inversade f perto de y = 0, determine g′(0).Extra 13.(a) Determine a derivada de arcsenh utilizando o Teorema de Função inversa e identidadeshiperbóli as.(b) Prove que arcsen(x) = log(x+

√1 + x2).Extra 14.Deduza a fórmula da derivada de n

√x utilizando somente a fórmula da derivada de

xn: (xn)′ = nxn−1.1.3.4 DesaosDesao 1.Cal ule pela denição a derivada no ponto x = 0 de:(a) f(x) = x sen(1/x); x 6= 0;

0; x = 0;(b) g(x) = x2 sen(1/x); x 6= 0;

0; x = 0.Desao 2.Use o binmio de Newton para al ular pela denição a derivada de h(x) = xnpara n ∈ N.Desao 3.Considere f(x) = −x2

2. Determine uma função g tal que, para todo x > 0, areta tangente ao grá o de f em x seja paralela à reta normal ao grá o de g em x.Desao 4.Considere f(x) = x4 − 2x2 + x + 1. Determine dois pontos distintos do grá ode y = f(x) om a mesma reta tangente. Prove que a solução é úni a.Desao 5.Considere f uma função polinomial de grau 2. Mostre que a reta se ante aográ o de f nos pontos x1 e x2 é paralela à reta tangente no ponto médio x1 + x2

2quaisquerque sejam x1 e x2.Desao 6. Prove (veja outra prova no Desao da p.12) usando derivada de seno, osseno eexponen ial que

eiθ = cos θ + i sen θ.

Page 24: Sumario´ - labMA/UFRJmcabral/livros/livro-calculo/... · 2019. 6. 12. · Sumario´ 1 Enunciados dos Exercícios 1.1 Exercícios de Limite. 1 1.1.1 Exercícios de Fixação. 1 1.1.2

20 CAPÍTULO 1. ENUNCIADOS DOS EXERCÍCIOSDi a: Dena f(θ) =cos θ + i sen θ

eiθe derive. Quando derivar, trate i ∈ C omo uma onstante.Desao 7.Prove que existe um úni o par de funções s(x) e c(x) deriváveis para todo x ∈ Rtais que s′(x) = c(x);

s(0) = 0;e

c′(x) = −s(x);

c(0) = 1.Di a: Suponha que existam s1, c1 e s2, c2. Dena f = (s1 − s2)2 + (c1 − c2)

2. Mostreque f ′(x) = 0 e f(0) = 0. Aplique o TVM.Desao 8.Considere g(y) = y − ε sen(y).(a) prove que existe ε0 > 0 tal que, se |ε| < ε0, então g será uma função injetiva em R.Con lua que neste aso a função possui inversa.(b) Considere f = g−1 sua função inversa. Determine f ′(0).Desao 9. Seja ft : (0, 1) → R denido por f(x) = 1/qt se x = p/q ∈ Q fração irredutívelnão nula e f(x) = 0 aso ontrário. Prove que:(a) se t ≤ 2 então ft não é diferen iável em ponto algum;(b) se t > 2 então ft é diferen iável nos irra ionais.Desao 10. Seja f : R → R duas vezes derivável Prove que(a) f ′(a) = limh→0

f(a+ h)− f(a− h)

2h. Este é método da diferença entrada utilizado emanálise numéri a.(b) f ′′(a) = lim

h→0

f(a+ h) + f(a− h)− 2f(a)

h2.Desao 11. Seja p(x) = x3+ax2+bx+c. Determine ondições em a, b, c tais que p : R → Rseja uma bijeção om inversa ontínua.Desao 12.Dizemos que f : R → R é -Hölder se existem α,M > 0 tais que |f(x)−f(y)| ≤

M |x− y|α para todo x, y ∈ R. Prove que neste aso:(a) f é ontínua;(b) se α > 1 f é derivável. Con lua que f é onstante.Desao 13.Prove (por indução) a fórmula de Leibniz(f · g)(n) =

n∑

i=0

(

n

i

)

f (n−i) · g(i),onde (ni

)

=n!

i!(n− i)!e a notação f (m) signi a derivar a função f m-vezes.

Page 25: Sumario´ - labMA/UFRJmcabral/livros/livro-calculo/... · 2019. 6. 12. · Sumario´ 1 Enunciados dos Exercícios 1.1 Exercícios de Limite. 1 1.1.1 Exercícios de Fixação. 1 1.1.2

1.4. EXERCÍCIOS DE APLICAÇO DE DERIVADA 211.4 Exer í ios de Apli ação de Derivada1.4.1 Exer í ios de FixaçãoExer í io 1. Suponha que f(0) = 0, f ′ é ontínua e que f ′(0) = 5. Cal ule limx→0

f(x)

sen(x).Exer í io 2.Vamos al ular o limite lim

x→1

(x− 1)2

ex − eapli ando L'Hospital duas vezes. Assim,

limx→1

(x− 1)2

ex − e= lim

x→1

2(x− 1)

ex= lim

x→1

2

ex=

2

e2.Na realidade o limite é zero. Qual o erro?Exer í io 3. Sabe-se que f ′(2) = 4 e que f(2) = 5. Cal ule de forma aproximada(a) f(2.1); (b) f(1.95).Exer í io 4. Sabe-se que p(x) é o polinmio do segundo grau que melhor aproxima f(x) =

cos(x) perto do ponto x = π. Determine:(a) p(π); (b) p′(π); ( ) p′′(π).Exer í io 5.Esbo e o grá o de uma função ontínua para ada item abaixo que:(a) tenha um máximo lo al em x = −2 e um mínimo lo al em x = 1;(b) seja sempre res ente, mas até x = −2 om on avidade para ima e depois desteponto om on avidade para baixo.Exer í io 6.Considere uma f : [−3, 3] :→ R ujo quadro de sinais da função e das derivadasseja:−2 −1 0 1 2

f − − + + − −f ′ − + + − − +f ′′ + + − − + +Esbo e o grá o de y = f(x).Exer í io 7.Determine se é Verdadeiro ou Falso. Se for falso dê um ontraexemplo ou orrija.(a) se f ′′(x) > 0 para todo x ∈ [1, 2] então f ′ é res ente em [1, 2].(b) se f ′′(x) > 0 para todo x ∈ [1, 2] então f é possui on avidade para ima em [1, 2]( ) se h(x) = C para todo x ∈ [1, 2] então h não possui nenhum ponto do máximo nemmínimo lo al.Exer í io 8.Estude o Teorema 22 da p.114, o TVE (Teorema do Valor Extremo de Weiers-trass). Determine se é Verdadeiro ou Falso. Se for falso dê um ontraexemplo ou orrija.211.o t.2011

Page 26: Sumario´ - labMA/UFRJmcabral/livros/livro-calculo/... · 2019. 6. 12. · Sumario´ 1 Enunciados dos Exercícios 1.1 Exercícios de Limite. 1 1.1.1 Exercícios de Fixação. 1 1.1.2

22 CAPÍTULO 1. ENUNCIADOS DOS EXERCÍCIOS(a) Pelo TVE toda função ontínua em I = (−7, 100) possui um máximo em I.(b) Pelo TVE toda função ontínua em I = [0,+∞) possui um mínimo em I.( ) Pelo TVE toda função em I = [2, 3] possui um mínimo em I.(d) Pelo TVE toda função des ontínua em I = [2, 4] NO possui máximo neste intervalo.(e) Pelo TVE toda função ontínua em um intervalo ILIMITADO I NO possui máximoem I.Exer í io 9. Suponha que f é derivável em R e que f ′ se anula somente em 3 e 7.(a) É verdade que existe a ∈ [1, 10] tal que f(a) ≥ f(x) para todo x ∈ [1, 10]? Porque?(b) Explique omo podemos determinar a.( ) É verdade que existe c ∈ R tal que f(c) ≤ f(x) para todo x ∈ R? Porque?Exer í io 10. Sabendo f é ontínua em R e que f ′(x) > 0 para x < 0 e f ′(x) < 0, parax > 0, determine (se for possível) a, b, c, d ∈ R tais que(a) min

x∈[−5,−1]f(x) = f(a);(b) max

x∈[−2,3]f(x) = f(b);( ) min

x∈[−1,2]f(x) = f(c);(d) max

x∈[2,5]f(x) = f(d);Exer í io 11.Considere f(x) =

1

x. Determine, aso existam, para ada intervalo I abaixo,

maxx∈I

f(x), minx∈I

f(x) e os pontos xmax e xmin onde são atingidos o máximo e o mínimo.(a) I = [2, 3]; (b) I = (0, 1]; ( ) I = [−1,−4]; (d) I = [1,+∞); (e)I = (−∞, 0).Exer í io 12.Determine se é Verdadeiro ou Falso. Se for falso dê um ontraexemplo ou orrija. Suponha que todas as funções possuem derivadas em todos os pontos.(a) Se x = 4 é mínimo lo al de h então h′(4) = 0.(b) Se x = 2 é o máximo de f no intervalo [1, 4] então f ′(2) = 0.( ) Se x = 1 é o mínimo de f no intervalo [1, 4] então f ′(1) = 0.(d) Se g′(3) = 0 então x = 3 é o mínimo ou máximo lo al de g.Exer í io 13.Determine se é Verdadeiro ou Falso. Se for falso dê um ontraexemplo ou orrija.Sabendo que f e f ′ é derivável em I e a, b, c ∈ I:(a) f ′(b) = 0 e f ′′(b) = −1 então b é ponto de máximo lo al.(b) f ′(c) = 0 e f ′′(c) = 0 então c é NO é ponto de máximo nem mínimo de f em I.( ) se a é máximo lo al de f então a é máximo de f em I.Exer í io 14.Considere a função f esboçada na gura abaixo.(a) Determine os pontos de máximo e mínimo lo al de f .Determine os pontos de máximo e mínimo de f em:(b) [2, 4]; ( ) [−3, 1]; (d) [−1, 4].

Page 27: Sumario´ - labMA/UFRJmcabral/livros/livro-calculo/... · 2019. 6. 12. · Sumario´ 1 Enunciados dos Exercícios 1.1 Exercícios de Limite. 1 1.1.1 Exercícios de Fixação. 1 1.1.2

1.4. EXERCÍCIOS DE APLICAÇO DE DERIVADA 23Determine o sinal de f ′′ em:(e) x = −1.8; (f) x = 0; (g) x = 4.(h) Dentre os inteiros −3,−2, . . . , 4, determine os que estão próximos de pontos deinexão (tro a de on avidade) de f .Considere g′(x) = f(x). Determine os pontos de:(i) máximo e mínimo lo al de g; (j) inexão de g.x

y

−3 −2 −1 1 2 3 4f(x)

Exer í io 15.Considere f(x) = x4 − x3. Determine TODOS os pontos de:(a) máximo/mínimo lo ais de f .(b) máximo/mínimo de f no intervalo [−1, 2].( ) máximo/mínimo de f no intervalo [−1, 0].(d) máximo/mínimo de f em R.(e) máximo/mínimo de f em (−∞, −1].1.4.2 ProblemasProblema 1.Cal ule os limites abaixo:(a) limx→0

sen(8x)

e2x − 1(b) lim

x→0+(ex + 3x)1/x ( ) lim

x→0

ex − e−x

sen(5x)(d) lim

x→0

ax − bx

xProblema 2.Estime, através de uma aproximação linear lo al:(a) √65; (b) log(e2 − 0.1); ( ) arctan(1.2).Problema 3.Considere a função f(x) = ax3 + bx2 + cx+ d onde a > 0.(a) Mostre que f admite nenhum ou dois extremos lo ais. Sob que ondições ada umdesses asos o orre?(b) No aso em que f não admite extremos lo ais, quantas raízes reais f pode ter?( ) No aso em que f admite dois extremos lo ais, quantas raízes reais f pode ter?(d) Baseado nos itens anteriores, des reva um pro edimento para determinar o número deraízes reais de f .Problema 4. (grá os triviais) Esbo e o grá o de f e de uma função g tal que:(a) f(x) = x3 − 3x2 + 3x+ 1; (b) g′(x) = x3 − 4x.Problema 5.Esbo e o grá o de uma função y = f(x) tal que f(0) = 2 e f ′ é dado pelográ o abaixo.

Page 28: Sumario´ - labMA/UFRJmcabral/livros/livro-calculo/... · 2019. 6. 12. · Sumario´ 1 Enunciados dos Exercícios 1.1 Exercícios de Limite. 1 1.1.1 Exercícios de Fixação. 1 1.1.2

24 CAPÍTULO 1. ENUNCIADOS DOS EXERCÍCIOS-2 -1 0 1 2

x

y

f ’

-2 -1 0 1 2 x

y

f ’

-2 -1 0 1 2 x

y

f ’

-2 -1 0 1 2 x

y

f ’

Problema 6.Esbo e o grá o de uma função ontínua f nos maiores intervalos possíveisque verique todas as ondições indi adas simultaneamente.(a)• lim

x→0−f(x) = −∞, lim

x→0+f(x) = 1, f(0) = −1,

• limx→−∞

f(x) = 2, limx→+∞

f(x) = −1,• f ′(x) > 0 para x < −1, f ′(x) < 0 para −1 < x < 0, f ′(−1) = 0, f ′(x) < 0para x > 0.(b)• f(0) = 2, f(−2) = 1 e f ′(0) = 0.• lim

x→+∞f(x) = −1 e lim

x→−∞f(x) = 0.

• limx→2+

f(x) = +∞ e limx→2−

f(x) = −∞.• f ′(x) > 0 se x < 0 e f ′(x) < 0 se x > 0.• f ′′(x) < 0 se |x| < 2 e f ′′(x) > 0 se |x| > 2.Problema 7.Para as questões de esboço de grá o, antes do esboço deverá ser determinado:(a) TODOS os pontos de interseção om os eixos x e y;(b) os limites de no innito e TODAS as assíntotas;( ) os intervalos de res imento e de res imento;(d) TODOS os pontos de máximo e mínimo lo ais;(e) os intervalos om on avidade para ima e para baixo;Esbo e o grá o de ada uma das funções abaixo:(a) f(x) = 2x2 − 2x

(x− 2)(x+ 1). Di a: f ′(x) =

4 (1− 2x)

(x− 2)2 (x+ 1)2e

f ′′(x) =24 (x2 − x+ 1)

(x− 2)3 (x+ 1)3.(b) g(x) = 1 + x2

1− x2. Di a: g′(x) = 4

x

(1− x2)2e g′′(x) = 4

1 + 3 x2

(1− x2)3.( ) h(x) = x

(x− 1)2. Di a: h′(x) = − x+ 1

(x− 1)3e h′′(x) =

2 (x+ 2)

(x− 1)4.Problema 8.Esbo e o grá o da função:(a) f(x) = ex

x. Di a: f ′(x) =

(x− 1) ex

x2e f ′′(x) =

(x2 − 2 x+ 2) ex

x3.(b) f(x) = log(1− x2) + 1. Di a: f ′(x) =

2 x

x2 − 1e f ′′(x) = − 2 x2 + 2

(x2 − 1)2,

Page 29: Sumario´ - labMA/UFRJmcabral/livros/livro-calculo/... · 2019. 6. 12. · Sumario´ 1 Enunciados dos Exercícios 1.1 Exercícios de Limite. 1 1.1.1 Exercícios de Fixação. 1 1.1.2

1.4. EXERCÍCIOS DE APLICAÇO DE DERIVADA 25√1− e−1 ≈ 0.79.( ) f(x) = e(2−x)(x−1) + 1. Di a: f ′(x) = (3− 2x) e(2−x)(x−1) e

f ′′(x) = (4x2−12x+7) e(2−x)(x−1), 3/2−√2/2 ≈ 0.79 e 3/2+

√2/2 ≈ 2.20.(d) f(x) = x3 ex. Di a: f ′(x) = (x3 + 3 x2) ex e f ′′(x) = (x3 + 6 x2 + 6 x) ex,

−3 −√3 ≈ −4.7 e −3 +

√3 ≈ 1.26.Problema 9.Para ada função f e ada intervalo I abaixo, determine max

x∈If(x) e min

x∈If(x)e, se for possível, os pontos xmax e xmin onde o máximo/mínimo é atingidos.(a) f(x) = 1

sen(x)+

1

cos(x), I = (0, π/2).Di a: f ′(x) =

sen3 x− cos3 x

cos2 x sen2 x(b) f(x) = x+ 1x, I = (0, +∞), I = (0, 3], I = [3, 4].Di a: f ′(x) = 1− 4

x2.( ) f(x) = 3x4 − 4x3 + 12x2, I = [−1, 1] e [1, 2].Di a: f ′(x) = 12x(x2 − x+ 2) x.(d) f(x) = x

x+ 1em I = (−1, 1], I = [0, 1].Di a: f ′(x) =1

(x+ 1)2.Problema 10.Determine todos K ∈ R tais que a equação x

x4 + 3= K tenha pelo menosuma solução.Problema 11.En ontre dois números uja diferença seja 100 e ujo produto seja um mínimo.Problema 12.Uma hapa de metal de largura L deve ter duas bandas, de igual largura,dobradas ao longo do omprimento de maneira a formar uma alha retangular.Como devem ser feitas as dobras de tal forma que a alha omporte o maior volumepossível?

LProblema 13.Dispõe-se de 40m de o de arame para er ar um anteiro em um jardim ujaforma é a de um setor ir ular (fatia de pizza). Qual deve ser o raio do ír ulo para que o anteiro tenha a maior área possível ?

Page 30: Sumario´ - labMA/UFRJmcabral/livros/livro-calculo/... · 2019. 6. 12. · Sumario´ 1 Enunciados dos Exercícios 1.1 Exercícios de Limite. 1 1.1.1 Exercícios de Fixação. 1 1.1.2

26 CAPÍTULO 1. ENUNCIADOS DOS EXERCÍCIOSObs: A área de um setor ir ular é θr2/2, onde r é o raio do ír ulo e θ é o ângulo dosetor ir ular.θ

r

Problema 14.A tela do inema CABRALPLEX está a uma distân ia K do hão e possuialtura L. Um espe tador vai se sentar nesta sala, que é plana (não possui in linação), demodo que sentado em qualquer assento a distân ia entre seus olhos e o solo é h. A quedistân ia d da tela ele deve ar sentado para que per eba a maior imagem possível da tela?Note que a imagem é propor ional ao ângulo subentendido por seu olho e os extremos datela. Assumimos que a altura K > h, aso ontrário o melhor seria d = 0.h

L

K

B

d

θϕ

Problema 15.A página de um artaz deve ser retangular e ter uma área de A m2 ommargens laterais iguais a M m, e margens superior e inferior de N m. Determine asdimensões do artaz que permitirão a maior área impressa.

N

MM

N

Problema 16.Um tanque ilíndri o tem a forma de um ilindro om duas semiesferas em ada extremidade. Determine a forma do ilindro que:

Page 31: Sumario´ - labMA/UFRJmcabral/livros/livro-calculo/... · 2019. 6. 12. · Sumario´ 1 Enunciados dos Exercícios 1.1 Exercícios de Limite. 1 1.1.1 Exercícios de Fixação. 1 1.1.2

1.4. EXERCÍCIOS DE APLICAÇO DE DERIVADA 27(a) maximizará o seu volume, sabendo que sua área de superfí ie é A,(b) minimizará o seu usto de fabri ação sabendo que seu volume é V .Problema 17.(a) Sejam f(x) = 2 +

√6x− 2x2 e P = (2, 2). Determine a maior e a menor distân iade P aos pontos do grá o de f .(b) Qual a menor distân ia verti al entre as urvas y = x2 e y = − 1

x2?Problema 18.Determine as dimensões do retângulo ins rito num ír ulo de raio R que possuio menor e o maior perímetro;

Problema 19.En ontre as dimensões do retângulo de maior área que tem sua base sobre oeixo x e seus dois outros vérti es a ima do eixo x e sobre a parábola y = 27− x2.Problema 20.Maximize o volume do:(a) one reto ins rito numa esfera de raio R;(b) ilindro ir ular reto ins rito num one ir ular reto de raio R e altura H .

1.4.3 ExtrasExtra 1.Cal ule os limites abaixo.(a) limx→1

log(4x− 3)

x− 1(b) lim

x→+∞

x2

log x( ) lim

x→1

4√x− 1

5√x− 1

(d) limx→+∞

x√x(e) lim

h→0

2 cos(h)− 2 + h2

h4

Page 32: Sumario´ - labMA/UFRJmcabral/livros/livro-calculo/... · 2019. 6. 12. · Sumario´ 1 Enunciados dos Exercícios 1.1 Exercícios de Limite. 1 1.1.1 Exercícios de Fixação. 1 1.1.2

28 CAPÍTULO 1. ENUNCIADOS DOS EXERCÍCIOSExtra 2.Estime, através de uma aproximação linear lo al:(a) tan(0.05). (b) 3√28.Extra 3. Determine a série de Taylor de: (a) sen x em a = 0; (b) log x em a = 1; ( )

senh x em a = 0.Extra 4. Suponha que a > 0. Prove que se b for pequeno o su iente então vale a aproxi-mação: 3√a3 + b ≈ a+

b

3a2.Extra 5.Esbo e o grá o de uma função f e de uma função g tal que:(a) g′(x) = x2 − 5x+ 6 omo derivada; (b) f(x) = (x− 1)2(x+ 1)2.Extra 6.Esbo e o grá o de uma função y = f(x) tal que f(0) = −2 e que tenha omoderivada o seguinte grá o:

-

6

x

y

0−2 −1 1 2 3 4

Extra 7.Esbo e o grá o de uma função ontínua f que verique todas as ondições indi- adas simultaneamente.(a) f(0) = 3, f(2) = 1, f ′(0) = f ′(2) = 0f ′(x) > 0 se |x− 1| > 1 f ′(x) < 0 se |x− 1| < 1f ′′(x) < 0 se x < 1 f ′′(x) > 0 se x > 1(b) f(2) = 4; f(4) = −1.f ′(2) = 0; f ′(x) > 0 se x < 2; f ′(x) < 0 se x > 2;f ′′(4) = 0; f ′′(x) < 0 se x < 4; f ′′(x) > 0 se x > 4;lim

x→−∞f(x) = −∞; lim

x→+∞f(x) = −3.Extra 8.Para as questões de esboço de grá o, antes do esboço deverá ser determinado:(a) TODOS os pontos de interseção om os eixos x e y;(b) os limites de no innito e TODAS as assíntotas;( ) os intervalos de res imento e de res imento;(d) TODOS os pontos de máximo e mínimo lo ais;(e) os intervalos om on avidade para ima e para baixo;

Page 33: Sumario´ - labMA/UFRJmcabral/livros/livro-calculo/... · 2019. 6. 12. · Sumario´ 1 Enunciados dos Exercícios 1.1 Exercícios de Limite. 1 1.1.1 Exercícios de Fixação. 1 1.1.2

1.4. EXERCÍCIOS DE APLICAÇO DE DERIVADA 29(função ra ional) Esbo e o grá o de ada uma das funções abaixo:PS: Ignore on avidade, não al ule f ′′.(a) f(x) = (x+ 1)(x+ 2)

(x+ 3)(x− 1). Di a: f ′(x) = − x2 + 10 x+ 13

(x− 1)2 (x+ 3)2.(b) g(x) = x

(x− 3)(1− x)+ 3. Di a: g′(x) = x2 − 3

(x− 3)2 (x− 1)2.Extra 9. (função ra ional) Esbo e o grá o de ada uma das funções abaixo:(a) f(x) = x2

x2 + 3.Di a: f ′(x) = 6x

(x2 + 3)2e f ′′(x) = 18

1− x2

(x2 + 3)3.(b) g(x) = x2

4− x2+ 2.Di a: g′(x) = 8 x

(x− 2)2 (x+ 2)2e g′′(x) = 8

4 + 3 x2

(4− x2)3.Extra 10. (função não-ra ional) Esbo e o grá o da função:(a) f(x) = x log xDi a: f ′(x) = log x+ 1, f ′′(x) =

1

x.(b) f(x) = x e1−x2Di a: f ′(x) = (1− 2x2) e1−x2, f ′′(x) = 2x(2x2 − 3) e1−x2,

1/√2 ≈ 0.707, √

3/2) ≈ 1.22.( ) f(x) = x2 exDi a: f ′(x) = x(x+ 2) ex, f ′′(x) = (x2 + 4 x+ 2) ex,−2−

√2 ≈ −3.41, −2 +

√2 ≈ −0.58).Extra 11.Para ada função f e ada intervalo I abaixo, determine max

x∈If(x) e min

x∈If(x) e,se for possível, os pontos xmax e xmin onde o máximo/mínimo é atingidos.(a) f(x) = 8x2 − x4 em I = R; I = [−1, 1].Di a: f ′(x) = 4x(4− x2).(b) f(x) = 1

x2 + 1em I = [1, 2] e I = [−1,+∞).Di a: f ′(x) = − 2x

(x2 + 1)2.( ) f(x) = sen(cosx) em [0, 2π].Extra 12. (problema om modelo simples) Suponha que uma janela tenha a forma de umretângulo om um triângulo equilátero no topo. Assumindo que o perímetro é de 12m,determine as dimensões da janela para que penetre o máximo de luz possível.Di a: área do triângulo equilátero é L2

√3/4.Extra 13.Modele os seguintes problemas e depois resolva-os.(a) Determine as dimensões do retângulo om perímetro P > 0 que possui a maior área.

Page 34: Sumario´ - labMA/UFRJmcabral/livros/livro-calculo/... · 2019. 6. 12. · Sumario´ 1 Enunciados dos Exercícios 1.1 Exercícios de Limite. 1 1.1.1 Exercícios de Fixação. 1 1.1.2

30 CAPÍTULO 1. ENUNCIADOS DOS EXERCÍCIOS(b) Um ampo retangular está limitado por uma er a em três de seus lados e por um órrego reto no quarto lado. Determine as dimensões do ampo om área máxima que podeser er ado om uma er a de omprimento total P .( ) Um terreno retangular deve ser er ado om dois tipos de er a. Dois lados opostosdevem re eber uma er a reforçada que usta R$3,00 por metro, enquanto os outros doislados uma er a que usta R$2,00 por metro. Quais as dimensões do terreno de maior áreaque pode ser er ado om R$6.000,00?Extra 14.(a) Determine os números x e y, om soma igual a S, uja soma dos quadrados seja omenor possível.(b) Determine o número positivo tal que a diferença entre ele e o seu ubo seja a menore a maior possível.( ) Suponha que o produto de dois número reais positivos é igual a P > 0. Determine omínimo e máximo da soma destes dois números.Extra 15.Queremos fazer uma aixa em forma de paralelepípedo de base quadrada e abertaem ima, isto é, uma aixa sem tampa de base quadrada.(a) Se o volume desta aixa é V cm3, determine as dimensões que minimizam a quantidadede material.(b) Se temos A cm2 de material para fazer a aixa, determine o maior volume possívelpara esta aixa.( ) Se o volume desta aixa é V cm3 e o usto do material da base é duas vezes mais aro que o usto do material dos lados, determine as dimensões que minimizam o usto defabri ação.Extra 16. (guras e parte do texto retirados da Wikipédia) Em ópti a, o prin ípio de Fermatou prin ípio do menor tempo diz que o aminho de um raio de luz entre dois pontos deveser o que pode ser feito no menor tempo possível. Deste prin ípio pode ser deduzido a lei dereexão e a lei de refração de Snell. Vamos deduzir ambos neste exer í io.(a) Considere um raio de luz que parte de P e vai até Q depois de reetir no espelho emum ponto O. Determine a relação entre o ângulo de in idên ia θi e o ângulo de reexão θrpara que o tempo per orrido pelo raio seja o menor possível.

(b) Considere um raio de luz que parte de P e vai até Q passando de um meio onde aluz possui velo idade v1 para um meio onde a velo idade é v2. Determine a relação entre os

Page 35: Sumario´ - labMA/UFRJmcabral/livros/livro-calculo/... · 2019. 6. 12. · Sumario´ 1 Enunciados dos Exercícios 1.1 Exercícios de Limite. 1 1.1.1 Exercícios de Fixação. 1 1.1.2

1.4. EXERCÍCIOS DE APLICAÇO DE DERIVADA 31ângulos θ1 e θ2 e as velo idades (a Lei de Snell) para que o tempo per orrido pelo raio sejao menor possível.

Extra 17.Determine o ponto da urva indi ada mais próximo do ponto indi ado.(a) urva x2 − y2 = 1 e ponto (0, 2); (b) urva y = x3 e ponto (4, 0);( ) elipse 4x2 + y2 = 8 e ponto (1, 0); (d) urva y =√x e ponto (2, 0).Extra 18.Determine as dimensões do retângulo ins rito em um semi ír ulo de raio R quepossui a maior área.Extra 19.Um ilindro é gerado ao se girar um retângulo de perímetro P em torno de umde seus lados. Qual deve ser a razão entre os lados do retângulo de tal forma que o ilindrotenha o maior volume possível?Extra 20.Maximize o volume do:(a) ilindro ir ular reto ins rito numa esfera de raio R;(b) one reto ins rito, de abeça para baixo, om vérti e no entro da base de um one ir ular reto de raio R e altura H .Extra 21.Uma er a de altura H a em volta de um prédio bem alto. Se a er a está a umadistân ia L do prédio, qual a menor es ada que vai do hão por ima da er a até a parededo prédio?

HL

1.4.4 ⋆Problemas (Taxas Rela ionadas)Problema 1.Uma aixa está sendo puxada por uma orda que passa por uma roldana presaa 1m a ima do solo. Determine a taxa de variação do ângulo θ, indi ado na gura abaixo, noinstante em que a aixa se en ontra a 1m do ponto P , situado abaixo da roldana, sabendoque a aixa se deslo a a 2m/min.

Page 36: Sumario´ - labMA/UFRJmcabral/livros/livro-calculo/... · 2019. 6. 12. · Sumario´ 1 Enunciados dos Exercícios 1.1 Exercícios de Limite. 1 1.1.1 Exercícios de Fixação. 1 1.1.2

32 CAPÍTULO 1. ENUNCIADOS DOS EXERCÍCIOS1mP

θ

Problema 2.Quando o último vagão de um trem passa por baixo de um viaduto, um arro ruza o viaduto numa rodovia perpendi ular aos trilhos e a 10m a ima deles. O trem está a20m/s e o arro a 40m/s. Com que velo idade se afastam um do outro após 2s?Problema 3.Uma es ada de tamanho L está apoiada numa parede. Se a base for puxadaao longo do hão, afastando-se da parede om velo idade V , om que velo idade o topo daes ada estará se movendo para baixo quando ele está a uma altura H do solo?

Problema 4.Um tanque ni o om água e vérti e para baixo tem raio R metros no topo ealtura H metros. Se a água uir para dentro do tanque om taxa onstante de V m3/s, omque velo idade em m/s a profundidade da água vai res er quando o tanque estiver om Lmetros de profundidade?

Problema 5.Um balão eleva-se verti almente do solo om velo idade variável. Quando obalão está a 48m do solo, subindo om velo idade 3m/s, passa, exatamente sob ele um arroviajando a velo idade de 20m/s. Quatro segundos após este instante, om que velo idadevaria a distân ia entre eles?Problema 6.Considere a parábola y = −x2 +1 na gura abaixo, onde a reta t é tangente àparábola no primeiro quadrante em ada ponto P (x, y). Sabendo que a taxa de variação daabs issa de P ( oordenada x) é de 2 m/min, determine a taxa de variação do lado MQ dotriângulo PMQ, quando o ponto de tangên ia é P0(1/√2, 1/2).

Page 37: Sumario´ - labMA/UFRJmcabral/livros/livro-calculo/... · 2019. 6. 12. · Sumario´ 1 Enunciados dos Exercícios 1.1 Exercícios de Limite. 1 1.1.1 Exercícios de Fixação. 1 1.1.2

1.4. EXERCÍCIOS DE APLICAÇO DE DERIVADA 33x

y

y = −x2 + 1

t

M

Q

P

Problema 7.Uma fonte luminosa aproxima-se perpendi ularmente de uma parede om velo- idade onstante de a metros/segundo, projetando uma imagem ir ular sobre esta. Sabe-seque a abertura do fa ho de luz (o ângulo entre os raios limites) é de 90o. Cal ule a velo idade om que a área iluminada sobre a parede está diminuindo quando a distân ia da fonte até aparede é de k metros.

Problema 8.Um retângulo possui lados que variam om o tempo e está ins rito numa regiãotriangular onforme a gura abaixo. Determine om que taxa a área do retângulo está variandono instante em que sua altura y mede 36 m e está aumentando à taxa de 0,5 m/s. Nesteinstante a área está aumentando ou diminuindo?

y40 m 60 mProblema 9.Um balão esféri o está se esvaziando.(a) Suponha que seu raio de res e a uma taxa onstante de 15 m/min. Com que taxa oar (m3/s) estará saindo do balão quando o raio for igual a 9 m ?(b) Suponha que no instante t0, em que seu raio é R0, se esvazia segundo uma taxa dep0 m3/s. Determine a taxa de variação de sua área de superfí ie.Problema 10.Um one está ins rito em uma esfera onforme mostra gura abaixo. Se oraio da esfera está aumentando a uma taxa de 0,9 m/min e a altura do one está aumentandoa uma taxa de 0,8 m/min, om que taxa está aumentando o volume do one no instante emque o raio da esfera mede 1m e a altura do one mede 4/3m.

Page 38: Sumario´ - labMA/UFRJmcabral/livros/livro-calculo/... · 2019. 6. 12. · Sumario´ 1 Enunciados dos Exercícios 1.1 Exercícios de Limite. 1 1.1.1 Exercícios de Fixação. 1 1.1.2

34 CAPÍTULO 1. ENUNCIADOS DOS EXERCÍCIOS

Problema 11.O perímetro de um quadrado res e a uma taxa de 3m/s no instante t = 4.Neste momento sua área é de 100m2. Com qual velo idade sua área estará aumentando noinstante t = 4.Problema 12.Uma mulher de 1,80m de altura aminha em direção a um muro a uma razãode 4m/s. Diretamente atrás dela e a 40m do muro está um reetor 3m a ima do nível dosolo.Quão rápido o omprimento da sombra da mulher estará variando no muro quando elaestiver a meio aminho entre o reetor e o muro? A sombra estará esti ando-se ou en urtando-se?

Problema 13.Um erto tre ho do rio Amazonas é prati amente reto. Neste tre ho um bar odes e o rio paralelamente a sua margem, a uma distân ia de 3m da margem, om velo idade onstante de 10Km/h. A asa de um pes ador a nesta mesma margem do Amazonas, nabeira do rio.

barcoDetermine a velo idade om que o bar o se aproxima (ou se afasta) da asa do pes adorno instante em que ele está a 5m de distân ia da asa sabendo que ele:(a) se aproxima da asa; (b) já passou, e se afasta da asa.Problema 14.Mostre que:(a) se o raio de um ír ulo res e a uma taxa onstante, então sua área res e a uma taxapropor ional ao omprimento do raio.(b) se a aresta de um ubo res e a uma taxa onstante, então seu volume res e a umataxa propor ional à área da superfí ie.( ) se o volume de uma esfera está res endo a uma taxa propor ional à área de suasuperfí ie, então seu raio res e a uma taxa onstante.

Page 39: Sumario´ - labMA/UFRJmcabral/livros/livro-calculo/... · 2019. 6. 12. · Sumario´ 1 Enunciados dos Exercícios 1.1 Exercícios de Limite. 1 1.1.1 Exercícios de Fixação. 1 1.1.2

1.4. EXERCÍCIOS DE APLICAÇO DE DERIVADA 351.4.5 ⋆Problemas (Derivação Implí ita)Problema 1. Seja y = f(x) denida impli itamente em ada item abaixo. Determine aequação da reta tangente no ponto indi ado:(a) y3 + x2y = 130 em (1, 5); (b) x2 =x+ y

x− yem (−1, 0).Problema 2.Considere a urva x3 + y3 = 3xy. Determine os pontos onde a reta tangenteé verti al e onde é horizontal.Problema 3. Seja y = f(x) denida impli itamente por x2 − y2 +

√xy = 2 próximo aoponto (2, 2).(a) Cal ule f ′(2).(b) Determine a equação da reta tangente ao grá o de f(x) no ponto (2, 2).( ) Determine a equação da reta tangente ao grá o de g(x) = f(x)/x no ponto (2, 1).Problema 4.Para ada uma das funções y = f(x) denidas impli itamente perto de (x, y) =

(a, b) determine ([Co, p.485):• se a função é res ente ou de res ente perto de x = a;• f ′(a);• f ′′(a).(a) x5 + xy + y5 = 3 em (a, b) = (1, 1).(b) x cos(xy) = 0 em (a, b) = (1, π/2)Problema 5.En ontre o máximo e o mínimo de y = f(x) denida impli itamente por

x4 − xy + y4 = 253.Problema 6.Determine a, b ∈ R tais que (1, 1) perten e a urva denida impli itamente porx2y + ay2 = b e que a reta tangente nesse ponto é 4x+ 3y = 7.Problema 7.Determine a equação da reta tangente à urva xy = yx no ponto (k0, k0) omk0 6= e.1.4.6 DesaosDesao 1. (formas indeterminadas) Vamos mostrar que 00, (+∞)0 e 1+∞ podem dar qual-quer número. Cal ule os limites abaixo (use L'Hospital) assumindo que k > 0:(a) lim

x→0+x(log k)/(1+log x)[“ =′′ 00];(b) lim

x→+∞x(log k)/(1+log x)[“ =′′ (+∞)0];( ) lim

x→0(x+ 1)(log k/x)[“ =′′ 1+∞].Desao 2.Aproxime a função f(x) =

x

x− 2perto do ponto x = 1 por um polinmio dotipo p(h) = ah2 + bh + c.

Page 40: Sumario´ - labMA/UFRJmcabral/livros/livro-calculo/... · 2019. 6. 12. · Sumario´ 1 Enunciados dos Exercícios 1.1 Exercícios de Limite. 1 1.1.1 Exercícios de Fixação. 1 1.1.2

36 CAPÍTULO 1. ENUNCIADOS DOS EXERCÍCIOSDesao 3. O objetivo deste exer í io é obter uma fórmula fe hada para π. Para isto vamos al ular a série de Taylor do arctan.(a) Verique que f(x) =1

1 + x2=

1

2i

(

1

x− i− 1

x+ i

).(b) Determine fk(x) para todo k ∈ N (k-ésima derivada).( ) Determine a série de Taylor do arctan(x).(d) Prove que π

4= 1− 1

3+

1

5− 1

7+

1

9· · · .Desao 4. Sua asa possui um orredor longo de largura a que termina num orredor per-pendi ular a este de largura b. Vo ê deseja mover um sofá de largura c (menor que a e b !).Desprezando a altura ( onsidere o sofá omo um retângulo), qual o omprimento máximo dosofá que pode fazer a urva ? (Somente monte o problema, as ontas são ompli adas, epodem ser resolvidos somente numeri amente).Desao 5.Prove que a menor distân ia entre o ponto (a, b) até o grá o de y = f(x) émedido na reta normal ao grá o de f .Desao 6.Prove que a distân ia entre o ponto (x0, y0) e a reta ax + by + c = 0 é igual a

|ax0 + by0 + c|√a2 + b2

.Desao 7.Podemos aproximar log a (onde a > 1) pela soma das áreas de dois trapézios, on-forme a gura abaixo. Determine o ponto x de maneira que o erro da aproximação seja mínimo.1 x a

x

y

y=1/x

1 x a x

y

y=1/x

1 x a x

y

y=1/x

1 x a x

y

y=1/x

1 x a x

y

y=1/x

1 x a x

y

y=1/x

Desao 8.Maximize a área:(a) lateral do one reto ins rito numa esfera de raio R;(b) do ilindro ir ular reto ins rito num one ir ular reto de raio R e altura H .

Page 41: Sumario´ - labMA/UFRJmcabral/livros/livro-calculo/... · 2019. 6. 12. · Sumario´ 1 Enunciados dos Exercícios 1.1 Exercícios de Limite. 1 1.1.1 Exercícios de Fixação. 1 1.1.2

1.5. EXERCÍCIOS DE INTEGRAL 37Desao 9.Deseja-se atravessar um lago ir ular até um ponto diametralmente oposto. Pode-se nadar e orrer. A velo idade orrendo é o dobro da velo idade nadando. Qual deve ser oper urso para se hegar mais rapidamente ao lado oposto?Desao 10. ( urvatura) Dado uma urva y = f(x) queremos determinar o raio do ír uloque os ula esta urva no ponto x = c. Mais pre isamente, queremos determinar o raio do ír ulo que passa por (c, f(c)) om mesma tangente e mesma derivada segunda que f .Mostre que se η0 = f(c), η1 = f ′(c) e η2 = f ′′(c) então o raio do ír ulo é igual a(1 + η21)

3/2

|η2|. O inverso do raio é hamado de urvatura da urva no ponto x = c.Desao 11. Se a1 < · · · < an, en ontre o mínimo global de g(x) =

n∑

i=1

|x− ai|.Di a: omo a função é linear entre os intervalos, o mínimo o orre em um dos ai's.Considere omo g(x) se modi a quando se passa de um intervalo a outro. Tente fazer omn = 2 e depois om n = 3.Desao 12.Considere f(x) =

e−1/x2

; x > 0;

0; x ≤ 0.. Prove que f (n)(0) = 0 para todo n ∈ N(isto é, as derivadas de qualquer ordem vale 0 em x = 0).Obs: Neste aso o polinmio de Taylor al ulado em x = 0 será sempre p(x) = 0, e aaproximação não melhora om o aumento do grau do polinmio.

1.5 Exer í ios de Integral1.5.1 Exer í ios de FixaçãoExer í io 1.Determine se é Verdadeiro (provando a armativa) ou Falso (dando um ontra-exemplo):(a) Se ∫ b

a

f(x) dx = 0, então f(x) = 0 para todo x ∈ [a, b].(b) Se f(x) ≤ 0 para todo x ∈ [a, b], então ∫ b

a

f(x) dx ≤ 0.( ) Se ∫ 3

0

h(x) dx = 9 e mudarmos o valor da função em x = 1 e em x = 2, a integralvai mudar de valor.Exer í io 2.Estude a Denição 26 da p.149 e o Lema 9 da p.149 e resolva.Sabendo que ∫ 2

−1

f(x) dx = 5,

∫ 2

−1

g(x) dx = −3 e ∫ 0

−1

f(x) dx = 7, al ule:(a) ∫ −1

2

f(x) dx; (b) ∫ 2

−1

(

f(x) + 2g(x))

dx; ( ) ∫ 1

1

g(sen(x2)) dx;206.o t.2011

Page 42: Sumario´ - labMA/UFRJmcabral/livros/livro-calculo/... · 2019. 6. 12. · Sumario´ 1 Enunciados dos Exercícios 1.1 Exercícios de Limite. 1 1.1.1 Exercícios de Fixação. 1 1.1.2

38 CAPÍTULO 1. ENUNCIADOS DOS EXERCÍCIOS(d) ∫ 2

0

f(x) dx; (e) ∫ 2

−1

(∫ 0

−1

f(s)g(t) ds

)

dt.(f) ∫ 2

−1

h(x) dx se h(x) =

f(x); x 6= 1;

5; x = 1.Exer í io 3.Considere a função f(x) representada na gura abaixo.x

y

f(x)

1 2 3 4 5

2

−1Dena F (x) =

∫ x

0

f(s) ds. Usando a ideia de que a integral é área om sinal respondaaos seguintes itens.(a) Determine F (0), F (1), F (2), F (3).(b) Determine os intervalos onde F res e e de res e.( ) Determine os pontos de máximo e mínimo lo al de F .Exer í io 4.O aluno X es reveu que:Como a primitiva de 1

x2é −1

x, temos que ∫ 2

−2

dx

x2= −1

x

2

−2

= −(1

2)− (−1

2) = 0.O aluno Y es reveu que: Como 1

x2> 0 para todo x,

∫ 2

−2

dx

x2> 0Resolva o onito entre os alunos X e Y.Exer í io 5.Estude o Teorema 24 da p.151 (TFC). Considere h(x) =

∫ x

2

(5− t)5

t4 + 6dt. De-termine:(a) h(2); (b) intervalos onde h res e e de res e; ( ) pontos de máximo e mínimolo al.Exer í io 6.Estude o Corolário 10 da p.151 (TFC). Sabendo que h(s) = g′(s) para todo

s ∈ R e que g(x) = Kex3

+Bx− C, determine ∫ 1

−1

h(s) ds.Exer í io 7.Cal ule:(a) ∫ 1

0

(2x3 − 3x2 + 5) dx; (b) ∫ 1

0

|y2 − 1| dy; ( ) ∫ (3x+ et − 7x sen t) dt.

Page 43: Sumario´ - labMA/UFRJmcabral/livros/livro-calculo/... · 2019. 6. 12. · Sumario´ 1 Enunciados dos Exercícios 1.1 Exercícios de Limite. 1 1.1.1 Exercícios de Fixação. 1 1.1.2

1.5. EXERCÍCIOS DE INTEGRAL 39Exer í io 8.Os três melhores alunos da sala integraram a mesma função e en ontraram asseguintes respostas: −cos 2x

4+ C, −cos2x

2+ C e sen2x

2+ C. Como vo ê expli aisso? Será que algum (ou todos) erraram a integração?Exer í io 9.Estude a Seção 5.3 da p.154 (Integrais Impróprias). Sem al ular as integraisabaixo, es reva ada uma omo o limite de uma integral própria:(a) ∫ 5

−∞e−s2 ds; (b) ∫ 2

0

dx

log(5− x2); ( ) ∫ +∞

0

dy

1 + y4; (d) ∫ 2

1

dx

x10 − 1.Exer í io 10. Faça mudança de variáveis para provar que:(a) ∫ bc

ac

f(t) dt = c

∫ b

a

f(x) dx; (b) ∫ b+c

a+c

f(x− c) dx =

∫ b

a

f(u) du.Exer í io 11.Cal ule as seguintes integrais (por substituição):(a) ∫ 4√K − 3x dx; (b) ∫ 3x2 cos(x3) dx; ( ) ∫ log2(t)

tdt;(d) ∫ x

√3− 2x2 dx; (e) ∫ sen(θ)

cos(θ)dθ ; (f) ∫ cosx e5 senx dx.Exer í io 12.Cal ule as seguintes integrais (por partes):(a) ∫ x log x dx; (b) ∫ arctanx dx.Exer í io 13.Cal ule as seguintes integrais denidas:(a) ∫ 2

1

(3− 2x)4 dx; (b) ∫ +∞

log 3

e−x/4 dx;( ) ∫ π

π/4

sen(2θ) dθ; (d) ∫ +∞

1

1

s3ds.

1.5.2 ProblemasProblema 1.Considere f(x) =

2x se 0 < x ≤ 2,

−1 se 2 < x ≤ 4,

5− x se 4 < x ≤ 5.

Determine:(a) ∫ 3

0

f(x) dx; (b) ∫ 4

2

f(x) dx; ( ) ∫ 5

1

f(x) dx.Problema 2.Estude o Lema 9 da p.149 e prove que:(a) se f(x) ≤ M , então ∫ b

a

f(x) dx ≤ M(b− a);(b) ∫ 6e

e

4 sen(ex + 5x2 + x) log x dx ≥ −20e.

Page 44: Sumario´ - labMA/UFRJmcabral/livros/livro-calculo/... · 2019. 6. 12. · Sumario´ 1 Enunciados dos Exercícios 1.1 Exercícios de Limite. 1 1.1.1 Exercícios de Fixação. 1 1.1.2

40 CAPÍTULO 1. ENUNCIADOS DOS EXERCÍCIOSProblema 3.Considere F (x) =

∫ x

2

t2 − 1

t2 + 1dt. Determine:(a) os intervalos nos quais F é res ente e onde é de res ente;(b) os intervalos nos quais o grá o de F possui on avidade para baixo e onde é para ima;( ) o valor de x onde F atinge um mínimo lo al e o valor onde atinge um máximo lo al.Problema 4.Determine a equação da reta tangente ao grá o de

y(x) = log(2 + sen(x2 − π)) +

∫ x

√π

cos(s2) dsno ponto (√π, log 2).Problema 5.Cal ule:(a) f ′(1) se f(y) =

∫ y

1

et3

dt

4

cos(1 + s2) ds;(b) g′(8) se g(y) =

∫ 5

y

(∫ x

8

log(t3 + 1) dt

)

dx.Problema 6.Determine para quais p > 0 as integrais abaixo são nitas.(a) ∫ +∞

1

dx

xp; (b) ∫ 1

0

dx

xp.Problema 7. (integral indenida)(a)∫ cos(

√k)√

kdk; (b) ∫ 3x√

1− 3 x2dx; ( ) ∫ x sen(3x+ 1) dx;(d) ∫ sec2 x log(tanx) dx; (e) ∫ sen(√t) dt; (f) ∫ e2x cos x dx;(g) ∫ sen(log x) dx; (h) ∫ e3

√s ds; (i) ∫ ex

1 + e2xdx.Problema 8. (integral denida)(a) ∫ 1

0

xe−x2

dx; (b) ∫ 1

0

se−3s ds; ( ) ∫ +∞

e

dt

t(log t)3dt; (d) ∫ +∞

0

se−s/2 ds;(e) ∫ log 3

0

ex√1 + ex dx; (f) ∫ 1

0

x

1 + x2dx; (g) ∫ 8

1

1

x2

1 +1

xdx.Problema 9. (integral om módulo)(a) ∫ 4

0

x2|x− 2| dx; (b) ∫ 2

1/2

| log s| ds; ( ) ∫ 2

−2

|es−1 − 1| ds.Problema 10.Determine y(x) sabendo que:(a) dy

dx=

2x+ 1√x

e y(1) = 0; (b) dy

dx= xex

2+1 e y(1) = e2.

Page 45: Sumario´ - labMA/UFRJmcabral/livros/livro-calculo/... · 2019. 6. 12. · Sumario´ 1 Enunciados dos Exercícios 1.1 Exercícios de Limite. 1 1.1.1 Exercícios de Fixação. 1 1.1.2

1.5. EXERCÍCIOS DE INTEGRAL 41Problema 11. (Integrais Impróprias)(a) ∫ 16

0

dx4√x. (b) ∫ +∞

e

dx

x log3 x; ( ) ∫ 0

−∞

ex

3− 2exdx;Problema 12.Determine:(a) lim

x→+∞e−x2

∫ x

0

log(t9 + 3) dt;(b) uma função f tal que f(0) = 1 e que ∫ x

−π

e−sf ′(s) ds = 3x para todo x ∈ R.1.5.3 ExtrasExtra 1. Suponha que f é ontínua e lim

x→+∞f(x) = K. Determine lim

x→+∞

1

x

∫ x

0

f(t) dt ([Spp.239 no.34).Extra 2.Determine TODOS os valores de x ∈ R onde a função Si(x) = ∫ x

0

sen t

tdt possuipontos de máximo lo al.Extra 3.Determine a equação da reta tangente ao grá o de ada função no ponto indi ado:(a) f(x) = ∫ x

1

log(et + t− 1)et2

dx no no ponto x = 1;(b) h(x) = 7−∫ x

2

es

s2 + 1ds no ponto x = 2.Extra 4.Considere um móvel preso a uma mola e deslizando sobre uma superfí ie sem atrito(veja gura abaixo). Sua a eleração é dada por a(t) = Aω2 cos(ωt) ∀t ≥ 0 (onde A e ω são onstantes). No instante t = 0 o móvel está na posição x(0) = 0 e tem velo idade v(0) = 0.Determine a função x(t) que determina a posição do orpo ao longo do tempo.

x

A0−AExtra 5. Seja f(s) =

∫ s2

s

sen(t)

t2dt. Determine f ′(s).Extra 6.Considere g(x) =

∫ e2x

0

sen(t2) dt. Cal ule g′(x).

Page 46: Sumario´ - labMA/UFRJmcabral/livros/livro-calculo/... · 2019. 6. 12. · Sumario´ 1 Enunciados dos Exercícios 1.1 Exercícios de Limite. 1 1.1.1 Exercícios de Fixação. 1 1.1.2

42 CAPÍTULO 1. ENUNCIADOS DOS EXERCÍCIOSExtra 7. Sabendo que ∫ 3

−1

f(s) ds = 7 e ∫ 3

1

f(s) ds = 3 determine ∫ 0

−1

f(2x+ 1) dx.Extra 8.Determine a função y(θ) sabendo que:(a) dy

dθ= cos(5θ) + 3 e y(π) = 5π; (b) dy

dx=

1

2x+ 1e y(0) = 3.Extra 9. Suponha que um ponto move-se ao longo de uma urva y = f(x) no plano xy de talforma que a ada ponto (x, y) da urva a reta tangente tem in linação √

x+ 1. Determinea equação da urva sabendo que ela passa pelo ponto (0, 1).Extra 10. (integral om módulo)(a) ∫ 4

0

|x2 − 3x+ 2| dx; (b) ∫ 3

−3

1 + |x| dx; ( ) ∫ 2

−2

x∣

∣x2 − 2x∣

∣ dx.Extra 11. (integral indenida)(a) ∫ x2 log x dx; (b) ∫ √x log x dx; ( ) ∫ x(log x)2 dx; (d) ∫ sen x− cosx

cosx+ sen xdx(e) ∫ ex

e2x + 2ex + 1dx; (f) ∫ e3 sen(x)+4 cos(x) dx; (g) ∫ ex cos(ex + 3) dx;(h) ∫ ax dx om a > 0.Extra 12. (integral denida)(a) ∫ 1

1/2

e1/x

x2dx; (b) ∫ √

log 2

1

x3ex2

dx; ( ) ∫ 4

1

log x√x

dx;(d) ∫ π/2

0

cos θ√sen θ dθ; (e) ∫ 1

0

y2√1− y

dy; (f) ∫ 4π2

π2

sen√x√

xdx;(g) ∫ log 3

log 2

ex

ex + 4dx; (h) 1

0

t√t2 + 1

dt.Extra 13. (Integrais Impróprias)(a) ∫ +∞

1

log x

x2dx; (b) ∫ +∞

1

log x

xpdx (p > 1); ( ) ∫ 2

−∞

dx

(4− x)2;(d) ∫ +∞

0

e−x

√1− e−x

dx.Extra 14.Use integração por partes para provar as fórmula de redução de integral:(a) Se Im =

xmex dx, então Im = xmex −mIm−1.(b) (redução trigonométri a) Se Sm =

xm sen x dx e Cm =

xm cosx dx, entãoSm = −xm cosx+mCm−1 e Cm = xm sen x−mSm−1.

Page 47: Sumario´ - labMA/UFRJmcabral/livros/livro-calculo/... · 2019. 6. 12. · Sumario´ 1 Enunciados dos Exercícios 1.1 Exercícios de Limite. 1 1.1.1 Exercícios de Fixação. 1 1.1.2

1.5. EXERCÍCIOS DE INTEGRAL 43( ) Se Im =

senmx dx, então Im = − 1

msenm−1x cos x+

m− 1

mIm−2.(d) Se Ln =

(log x)n dx, então Ln = x(log x)n − nLn−1.(e) Mostre que L3 = x((log x)3 − 3(log x)2 + 6 log x− 6).1.5.4 ⋆Problemas (Integração e Substituição Trigonométri a)Problema 1.Determine:(a) ∫ cos3 x dx; (b) ∫ cos23x cos 5x dx; ( ) ∫ cos x sen x dx;(d) ∫ sen4x dx; (e) ∫ cos 4x sen 3x dx.Problema 2.Cal ule:(a) ∫ dx

(1− x2)3/2; (b) ∫ x2

√1− x2

dx; ( ) ∫ dx

(x2 + 2x+ 2)3/2;(d) ∫ √

x2 − 1

xdx; (e) ∫ dx

x2√x2 − 1

; (f) ∫ dx

x2√x2 + 25

.Problema 3.Determine:(a) ∫ dx√4− x2

; (b) ∫ 1

0

x√1− x2

dx; ( ) ∫ dx√x2 − 1

;(d) ∫ x3

(1− x2)3/2dx; (e) ∫ 4

2√2

√16− x2 dx.

1.5.5 ⋆Problemas (Integração de Funções Ra ionais)Problema 1.Cal ule ∫ +∞

4

2

x2 − 1dx.Problema 2.Cal ule:(a) ∫ 2x− 3

x2 − 8x+ 7dx; (b) ∫ 2x− 3

x3 − x2dx; ( ) ∫ x+ 4

x3 + 4xdx;(d) ∫ 6 dx

x2 + 8x+ 7; (e) ∫ x2 + 4x− 2

x4 − 4x2dx; (f) ∫ 2x2 + x+ 2

(x− 1)(x2 + 4)dx.Problema 3.Cal ule ∫ +∞

−∞

dx

x2 + bx+ cse b2 − 4c < 0.Problema 4. Suponha que a 6= b e a 6= 0. Cal ule:(a) ∫ dx

(x− a)(x− b), (b) ∫ dx

x2(x− a).

Page 48: Sumario´ - labMA/UFRJmcabral/livros/livro-calculo/... · 2019. 6. 12. · Sumario´ 1 Enunciados dos Exercícios 1.1 Exercícios de Limite. 1 1.1.1 Exercícios de Fixação. 1 1.1.2

44 CAPÍTULO 1. ENUNCIADOS DOS EXERCÍCIOSProblema 5. [Ha, p.252 Determine ∫

Ax+B

ax2 + 2bx+ cdx se:(a) ∆ = b2 − ac > 0; (b) ∆ < 0; ( ) ∆ = 0.1.5.6 DesaosDesao 1. Suponha que f é uma bijeção em [a, b]. Dena g(y) = f−1(y).(a) Prove que para todo x ∈ [a, b],

∫ x

a

f(s) ds = xf(x)− af(a)−∫ f(x)

f(a)

g(y) dy.(b) Determine a primitiva do arcsen x e log x utilizando (a).Desao 2.Prove que existe um polinmio p(x) tal que ∫ x3 ex dx = p(x) ex. Determinep(x).Di a: TFC.Desao 3.Prove que dado n ∈ N existem polinmios p(x) e q(x) tais que:(a) ∫ xn ex dx = p(x) ex; (b) ∫ (log x)n dx = x q(log x).Con lusão: podemos al ular ∫ x4 ex dx tentando determinar os oe ientes do polinmioe usando o TFC.Desao 4.Prove que dados a, b ∈ R, existem A,B ∈ R tais que

eax sen(bx) dx = eax(A sen(bx) +B cos(bx)).Use este resultado para provar que:∫

ex sen(2x) dx =ex

5(sen(2x)− 2 cos(2x)) + C.

e3x sen(2x) dx =ex

13(3 sen(2x)− 2 cos(2x)) + C.Di a: aplique o TFC e resolva um sistema do segundo grau.Obs: Existe resultado análogo para o osseno.Desao 5.Prove que não existe um polinmio p(x) tal que ∫ ex

2

dx = p(x)ex2 .Desao 6.Use integração por partes para provar as fórmula de redução de integral: Se

Im =

dx

(x2 + 1)m, então Im =

x

2(m− 1)(x2 + 1)m−1+

2m− 3

2(m− 1)Im−1;

Page 49: Sumario´ - labMA/UFRJmcabral/livros/livro-calculo/... · 2019. 6. 12. · Sumario´ 1 Enunciados dos Exercícios 1.1 Exercícios de Limite. 1 1.1.1 Exercícios de Fixação. 1 1.1.2

1.5. EXERCÍCIOS DE INTEGRAL 45Desao 7. Use integração por partes para provar as fórmula de redução de integral:(a) Se Tn =

tannx dx, então Tn =tann−1x

n− 1− Tn−2;(b) Se Sn =

secnx dx (n ≥ 2), então Sn =tan x secn−2x

n− 1− n− 2

n− 1Sn−2;Verique, usando o TFC, que( ) S1 =

sec x dx = log | sec x+ tan x|.Desao 8. Vamos mostrar omo determinar ∫ tanmx secnx dx, om m,n ∈ N, n,m > 1.Prove que:(a) se m = 2k + 1 (impar) então existe um polinmio P tal que esta integral é igual ap(sec x).Di a: separa um termo tan e substitua u = sec x. 1 + tan2x.(b) se m = 2k (par), então existe um polinmio Q tal que esta integral é igual a∫

Q(secx) dx e podemos apli ar o exer í io anterior para al ulá-la.Desao 9.Vamos ver omo integrar qualquer função ra ional de senos e ossenos ([Cop. 290). Isto in lui, entre outros: ∫ tanmx secnx dx, om m,n ∈ N (feita nos exer í iosanteriores om outra té ni a).Dena t(x) = tan(x/2).(a) Prove que sin x =2t

1 + t2e cosx =

1− t2

1 + t2.(b) Prove que dt

dx=

1 + t2

2.( ) Seja R(c, s) um quo iente de polinmios em 2 variáveis. Então

R(cosx, sen x) dx =

R

(

1− t2

1 + t2,

2t

1 + t2

)

2

1 + t2dt.Ou seja, transformamos numa integral de função ra ional.Use ( ) para al ular:(d) ∫ dx

1 + cosx; (e) ∫ dx

1 + sen x; (f) ∫ dx

sen x.Use ( ) para es rever omo integral de função ra ional:(g) ∫ sec x tan2x dx; (h) ∫ sec 2x tan 3x dx.Desao 10.Prove que ∫ 1

0

x−x dx =

∞∑

n=1

n−n.Di a: Es reva xx usando exponen ial; expanda utilizando a série de Taylor da exponen ial;integre termo a termo por partes. Somente por uriosidade: o valor da integral é aproxima-damente 1.29128.Desao 11.Mostre que ∫ +∞

0

e−y2 dy existe.

Page 50: Sumario´ - labMA/UFRJmcabral/livros/livro-calculo/... · 2019. 6. 12. · Sumario´ 1 Enunciados dos Exercícios 1.1 Exercícios de Limite. 1 1.1.1 Exercícios de Fixação. 1 1.1.2

46 CAPÍTULO 1. ENUNCIADOS DOS EXERCÍCIOSDi a: Não tente al ular a integral, faça uma estimativa utilizando ye−y2.Desao 12.Mostre que ∫ +∞

0

sen x

xdx existe através do seguinte roteiro:(a) Integre por partes e mostre que

∫ b

a

sen x

xdx =

cos a

a− cos b

b+

∫ b

a

cosx

x2dx.(b) Use o lado esquerdo para investigar ∫ 1

0

e o lado direito para investigar ∫ +∞

1

.Desao 13. Suponha que f é ontínua e satisfaça f(x) =

∫ x

0

f(s) ds. Prove que f(x) = 0para todo x ∈ R.Desao 14.Dena f(y) =

∫ b

a

ty dt. para 0 < a < b xos. Prove que f é ontínua em −1.Desao 15.Determine, utilizando a denição:(a) ∫ a

0

x3 dx; (b) ∫ a

0

sen(x) dxDi a: Ver [Co.Desao 16. (Lema de du Bois-Reymond3) Seja f : [a, b] → R ontínua. Prove que se∫ y

x

f(s) ds = 0 para todo x, y ∈ [a, b] então f(x) = 0 para todo x, y ∈ [a, b].Desao 17. (Função Gama de Euler4, generalização de fatorial para não-inteiros e om-plexos) Dena Γ(z) =

∫ ∞

0

e−ttz−1 dt. Prove que:(a) a integral onverge para z > 0; (b) Γ(n) = (n− 1)! para n ∈ N.Di a: Para (b) integre por partes e prove que Γ(z + 1) = zΓ(z).( ) Use a substituição u = tx e mostre que Γ(1/2) = 2

∫ +∞

0

e−y2 dy. Esta integral éimportante em estatísti a e pode-se mostrar que ∫ +∞

0

e−y2 dy =√π/2. Assim, Γ(1/2) = √

πe Γ(3/2) = (1/2)! = 1/2Γ(1/2) =√π/2. ([Sp p.327 no.25)Desao 18.Mostre que ∫ dx

x2 + 1= arctan x utilizando frações par iais omplexas, seguindoo roteiro abaixo:3Paul David Gustav du Bois-Reymond: ⋆ 02/12/1831, Berlim, Alemanha † 07/04/1889, Freiburg,Alemanha.4Leonhard Euler: ⋆ 15/04/1707, Basel, Suíça † 18/09/1783, São Petesburgo, Rússia.

Page 51: Sumario´ - labMA/UFRJmcabral/livros/livro-calculo/... · 2019. 6. 12. · Sumario´ 1 Enunciados dos Exercícios 1.1 Exercícios de Limite. 1 1.1.1 Exercícios de Fixação. 1 1.1.2

1.6. EXERCÍCIOS DE APLICAÇÕES DA INTEGRAL 47(a) Determine A,B ∈ C tais que 1

x2 + 1=

A

x+ i+

B

x− i.(b) Utilize (a) para mostrar que ∫ dx

x2 + 1=

1

2ilog

[

x− i

x+ i

].( ) Prove que se |z| = 1 então existe θ ∈ R tal que log z = iθ (note que tem outraspossibilidades: log z = i(θ + 2kπ), k ∈ Z).(d) Mostre que |(x− i)/(x+ i)| = 1.(e) Mostre que se x = tan θ, (x− i)/(x+ i) = e2iθ.(f) Con lua que 1

2ilog

[

x− i

x+ i

]

= θ = arctan x.Desao 19.Determine ∫ dx

a cosx+ b sen x, om a2 + b2 6= 0.Di a: Mostre que existem r, θ tais que a = r sen θ, b = r cos θ.Desao 20.O objetivo aqui é explorar fórmulas para as funções hiperbóli as análogas aos doseno e osseno.(a) Integre por partes e determine uma fórmula de re orrên ia para Shn =

senhn dxpara n ∈ N.(b) Desenvolva uma teoria análoga ao do Teorema 25 da p.161 para al ular∫

(

N∏

i=1

senh(aix)

)(

M∏

j=1

cosh(bjx)

)

dx om ai, bj ∈ R para i = 1, . . . , N e j = 1, . . .M .Di a: Use identidades trigonométri as hiperbóli as da p.60.1.6 Exer í ios de Apli ações da Integral1.6.1 Exer í ios de FixaçãoExer í io 1.Esbo e o grá o e al ule a área da região delimitada por:(a) y = ex + 2, y = ex, x = 0, x = 5.(b) y = 0, y = cos(x), x = −π/2, x = π/2.Exer í io 2.Considere os grá os de y = f(x) e y = g(x) representadas na gura abaixo.407.o t.2011

Page 52: Sumario´ - labMA/UFRJmcabral/livros/livro-calculo/... · 2019. 6. 12. · Sumario´ 1 Enunciados dos Exercícios 1.1 Exercícios de Limite. 1 1.1.1 Exercícios de Fixação. 1 1.1.2

48 CAPÍTULO 1. ENUNCIADOS DOS EXERCÍCIOSx

y

f(x)

g(x)

−1 1 2

3

6

Es reva uma (ou a soma de) integral(is) denida(s) que al ule a área delimitada por:(a) y = f(x) e y = g(x) para x ∈ [1, 2];(b) y = f(x) e y = g(x) para x ∈ [−1, 2];Exer í io 3.Considere os grá os de y = f(x) e y = g(x) do exer í io anterior. Es revauma (ou a soma de) integral(is) denida(s) que al ule o volume do sólido de revolução obtidopela rotação em torno do:(a) eixo x da região delimitada por y = 0 e y = f(x) para x ∈ [−1, 1];(b) eixo x da região delimitada por y = f(x) e y = g(x) para x ∈ [1, 2];( ) eixo y da região do item (b).Exer í io 4.Considere a região do plano delimitada por x = f(y) e x = g(y) indi ada nagura abaixo. Es reva uma integral que determine a área da região.x

y

x = f(y)

x = g(y)

3

−2

1 2

Exer í io 5. Suponha que Π(s) é o plano y = s em R3. Seja Ω ⊂ R3 um sólido ontidoentre os planos y = −2 e y = 4. Seja A(s) a área da interseção de Π(s) om Ω. Es revauma integral que determine o volume de Ω.Exer í io 6.Considere g : [1, 7] → R tal que −4 ≤ g(x) ≤ 5 para todo x ∈ [1, 7]. Sabendoque o valor médio de g no intervalo [1, 7] igual a K, prove que −4 ≤ K ≤ 5.

Page 53: Sumario´ - labMA/UFRJmcabral/livros/livro-calculo/... · 2019. 6. 12. · Sumario´ 1 Enunciados dos Exercícios 1.1 Exercícios de Limite. 1 1.1.1 Exercícios de Fixação. 1 1.1.2

1.6. EXERCÍCIOS DE APLICAÇÕES DA INTEGRAL 491.6.2 ProblemasProblema 1.Cal ule as áreas ha huradas das guras (a) e (b) abaixo. x

y

x

y

x

y

x

y

x

y

x

y

x

y

x

y

x

y

x

y

x

y

PSfrag repla ements

00000000000 11111111111

y = x2y = x2y = x2y = x2y = x2y = x2y = x2y = x2y = x2y = x2y = x2

y = x− x2y = x− x2y = x− x2y = x− x2y = x− x2y = x− x2y = x− x2y = x− x2y = x− x2y = x− x2y = x− x2 x

y

x

y

x

y

x

y

x

y

x

y

x

y

x

y

x

y

x

y

x

y

x

y

x

y

x

y

x

y

x

y

x

y

x

y

x

y

x

y

x

y

x

y

PSfrag repla ements

y = sen(x)y = sen(x)y = sen(x)y = sen(x)y = sen(x)y = sen(x)y = sen(x)y = sen(x)y = sen(x)y = sen(x)y = sen(x)y = sen(x)y = sen(x)y = sen(x)y = sen(x)y = sen(x)y = sen(x)y = sen(x)y = sen(x)y = sen(x)y = sen(x)y = sen(x)

y = cos(x)y = cos(x)y = cos(x)y = cos(x)y = cos(x)y = cos(x)y = cos(x)y = cos(x)y = cos(x)y = cos(x)y = cos(x)y = cos(x)y = cos(x)y = cos(x)y = cos(x)y = cos(x)y = cos(x)y = cos(x)y = cos(x)y = cos(x)y = cos(x)y = cos(x)(a) (b)Problema 2.Esbo e e al ule a área da região limitada:(a) y − x = 6, y − x3 = 0 e 2y + x = 0. Di a: x = 2 é raiz de x3 = x+ 6.(b) por y2 = 2x+ 4 e por y = x− 2.( ) inferiormente por y =√x, superiormente por y = 1 e lateralmente por x = 0.Problema 3.Considere a região do plano limitada superiormente por y = 4 +

√16− x2 einferiormente por y = 4 e y = 6 − x2, onforme indi ada na gura abaixo. Determine suaárea.

x

y

y = 4

y = 6− x2

y = 4 +√16− x2

Problema 4.Cal ule o volume do sólido de revolução gerado quando a região ha hurada nagura abaixo é girada em torno do eixo x e do eixo y. x

y

x

y

x

y

x

y

x

y

x

y

x

y

x

y

x

y

x

y

x

y

x

y

x

y

x

y

x

y

x

y

x

y

x

y

x

y

x

y

x

y

x

y

PSfrag repla ements

0000000000000000000000 1

1

1

1

1

1

1

1

1

1

1

1

1

1

1

1

1

1

1

1

1

1

1

1

1

1

1

1

1

1

1

1

1

1

1

1

1

1

1

1

1

1

1

1

2222222222222222222222

Problema 5.Esbo e a região do plano, determine sua área e al ule o volume do sólido derevolução obtido pela rotação em torno do eixo x da região do plano delimitada:

Page 54: Sumario´ - labMA/UFRJmcabral/livros/livro-calculo/... · 2019. 6. 12. · Sumario´ 1 Enunciados dos Exercícios 1.1 Exercícios de Limite. 1 1.1.1 Exercícios de Fixação. 1 1.1.2

50 CAPÍTULO 1. ENUNCIADOS DOS EXERCÍCIOS(a) por y = 3√x, y = 2 e x = 0;(b) a ima por y = e−x, abaixo pelo eixo x e a esquerda pela reta x = 1 (uma regiãoinnita).Problema 6. (sólido de revolução girado em torno de outros eixos) Determine o volume dosólido de revolução obtido quando a região limitada por x = y2 e x = y é girada em tornoda reta:(a) y = −1; (b) x = −1.Problema 7.Na gura abaixo, seja A o ponto de interseção da urva y = ex

2 om a reta L,e seja B o vérti e da parábola 4y = (x− 2)2. Suponha que a reta L passe por A e B. A retaL, a parábola e o grá o de y = ex

2 delimitam uma região Ω. Es reva uma soma de integraisque determine o volume do sólido de revolução obtido ao girar Ω em torno do eixo y.x

y

y = ex2

4y = (x− 2)2

L

Ω

1

1

B

A

Problema 8.Um bura o ilíndri o de raio a é feito passando pelo entro de uma esfera deraio r. Determine o volume do sólido (esfera om bura o no meio) remanes ente.Problema 9.Determine o volume do sólido uja base é o ír ulo (no plano xy) x2 + y2 = r2e ujas seções perpendi ulares ao eixo x são quadrados om um lado na base (no plano xy).Problema 10.Determine o volume do sólido uja base é limitada por y = x e y = x2 e ujasseções perpendi ulares ao eixo x são quadrados om um lado na base (no plano xy).Problema 11.Determine o valor médio das funções abaixo nos intervalos indi ados:(a) f(x) = x2 em [0, K]; (b) g(x) = sen(x) em [0, π].1.6.3 ExtrasExtra 1.Esbo e e al ule a área da região limitada por:(a) y = x2, y = 1/x, y = −2, x = −1 e x = 2.(b) y = 6πx− 3x2, y = cos(x)− 1( ) por y = x2 e y = 1− x2.

Page 55: Sumario´ - labMA/UFRJmcabral/livros/livro-calculo/... · 2019. 6. 12. · Sumario´ 1 Enunciados dos Exercícios 1.1 Exercícios de Limite. 1 1.1.1 Exercícios de Fixação. 1 1.1.2

1.6. EXERCÍCIOS DE APLICAÇÕES DA INTEGRAL 51Extra 2.Esbo e e es reva integrais que al ulem a área da região limitada por:(a) y = x3 − x e y = sen(πx) om x ∈ [−1, 1].(b) y = x3 − 3x2 + 2x e y = 3x− x2 − 2 (interseção em x = −1, 1, 2).Extra 3.Para ada n > 0, seja Ωn a região limitada pelo grá o de y = xn, o eixo x e a retax = 1. Se Wn é o volume do sólido obtido girando Ωn em torno do eixo x, e Vn é o volumedo sólido obtido girando-se Ωn em torno do eixo y, determine lim

n→+∞

Vn

Wn.Extra 4.Neste exer í io vamos mostrar omo denir log para depois denir a exponen ial.Dena f(x) =

∫ x

1

dx/x. Fingindo que vo ê não sabe a primitiva de 1/x, somente mudandovariável, prove que f(ab)f(a) + f(b).Extra 5. Seja R a região do plano delimitada pelas urvas y = c− x2 e 2x2 − 2c para c > 0.(a) Esbo e R; (b) Determine c > 0 tal que a área de R seja igual a 32.Extra 6.Esbo e a região do plano e al ule o volume do sólido de revolução obtido pelarotação em torno do eixo x da região do plano delimitada:(a) a ima pelo grá o de f(x) = √

log(x)− 1

x, abaixo pelo eixo x e a esquerda por x = e(região innita).(b) por y = log(x), por y = 0 e para x ∈ [1, e2].Extra 7.Esbo e a região do plano e es reva integrais que al ulem o volume do sólido derevolução obtido pela rotação em torno do eixo x e em torno do eixo y da região do planodelimitada por:(a) y = 1/(x2 + 5), y = 0, x = 0, x = 2.(b) y = x/2 e y =

√x.( ) y =

√x, y = 6− x e y = 0.Extra 8.A base de um sólido é a região (do plano xy) limitada por y2 = 4x e a reta x = 9.Cada plano perpendi ular ao eixo x intersepta o sólido num quadrado om um lado na base(no plano xy). Cal ule seu volume.Extra 9.Prove, utilizando somente o Prin ípio de Cavalieri, que o volume da esfera de raio

R mais o volume do one duplo om raio da base R e altura R é igual ao volume do ilindrode raio R e altura 2R.Obs: Trata-se de questão lássi a do Ensino Médio (será que ainda se ensina isso?): nãoutilizamos ál ulo!Extra 10.Uma alota esféri a é uma porção da esfera obtida através de um orte por umplano de uma esfera (veja gura abaixo) . Se o raio da esfera é r, a altura da alota é h e oraio da alota é a, determine o volume desta alota.

Page 56: Sumario´ - labMA/UFRJmcabral/livros/livro-calculo/... · 2019. 6. 12. · Sumario´ 1 Enunciados dos Exercícios 1.1 Exercícios de Limite. 1 1.1.1 Exercícios de Fixação. 1 1.1.2

52 CAPÍTULO 1. ENUNCIADOS DOS EXERCÍCIOSPSfrag repla ements

Extra 11.Considere o sólido de revolução gerado pela rotação da região limitada por y =√x+ 1, y = 0, x = 0 e x = 2 em torno do eixo x. Determine o valor de a tal que oplano x = a orta este sólido em duas partes de mesmo volume.Extra 12.Considere a elipse de equação x2

a2+

y2

b2= 1. Determine o volume do elipsoideobtido quando se gira esta elipse em torno do eixo x.Extra 13.Considere a região delimitada pelo eixo x e pelo grá o de y =

1√

x(x2 + 1).Cal ule o volume do sólido de revolução obtido ao girarmos o esta região em torno do eixo xpara x ≥ 1.1.6.4 ⋆Problemas (Comprimento de Curvas no Plano)Problema 1.Determine o omprimento do grá o da função:(a) y = f(x) = log(x+

√x2 − 1) para x ∈ [1, 2].(b) y = f(x) =

√16− x2 para x ∈ [0, 4];( ) y = f(x) = log x para x ∈ [1, 2];(d) y = g(x) = − log(

√2 cos x) para x ∈ [−π/4, π/4].Problema 2. Seja Cn o omprimento da urva xn + yn = 1. Prove que lim

n→+∞Cn = 4

√2.Problema 3.Prove que o omprimento de uma poligonal dada pela fórmula om integral éigual a soma dos tamanhos dos segmentos, ada um al ulado utilizando Pitágoras.1.6.5 ⋆Problemas (Área de Superfí ie de Sólido de Revolução)Problema 1.Cal ule a área da superfí ie de revolução gerada pela rotação em torno doeixo x da urva:(a) y = x3 para x ∈ [0, 1]. (b) y = x2 para x ∈ [0, 1]. ( ) y = e−x para x ≥ 0.

Page 57: Sumario´ - labMA/UFRJmcabral/livros/livro-calculo/... · 2019. 6. 12. · Sumario´ 1 Enunciados dos Exercícios 1.1 Exercícios de Limite. 1 1.1.1 Exercícios de Fixação. 1 1.1.2

1.6. EXERCÍCIOS DE APLICAÇÕES DA INTEGRAL 531.6.6 DesaosDesao 1.Os eixos de dois ilindros, ada um om raio r se inter eptam fazendo um ânguloreto (veja gura abaixo). Determine o volume da região omum aos dois ilindros.PSfrag repla ements

Este é onhe ido omo sólido de Steinmetz. Di a: Considere planos paralelos ao planoque ontem os eixos.Uma generalização é dada na gura abaixo.PSfrag repla ements

Desao 2.Cal ule a área da lúnula (interseção de dois ír ulos), um de raio r e outro R, ujos entros estão afastados uma distân ia L. Assumimos que L,R, r > 0. Veja nas gurasabaixo ilustrações de lúnulas em inza:PSfrag repla ements PSfrag repla ements

Desao 3.Aproxime a área do grá o de y = f(x) ≥ 0 para x ∈ [a, b] utilizando a somada área de trapézios. Mostre que obtemos a mesma fórmula que já tínhamos obtido antes:∫ b

a

f(x) dx.Desao 4. Seja f uma função par, isto é f(x) = f(−x) para todo x ∈ R. Prove que os oe ientes de Fourier bk = 0. E se f for impar, isto é f(−x) = −f(x) para todo x ∈ R. Oque pode-se on luir sobre oe ientes ak?

Page 58: Sumario´ - labMA/UFRJmcabral/livros/livro-calculo/... · 2019. 6. 12. · Sumario´ 1 Enunciados dos Exercícios 1.1 Exercícios de Limite. 1 1.1.1 Exercícios de Fixação. 1 1.1.2

54 CAPÍTULO 1. ENUNCIADOS DOS EXERCÍCIOS

Page 59: Sumario´ - labMA/UFRJmcabral/livros/livro-calculo/... · 2019. 6. 12. · Sumario´ 1 Enunciados dos Exercícios 1.1 Exercícios de Limite. 1 1.1.1 Exercícios de Fixação. 1 1.1.2

Capıtulo 2

Respostas dos Exercıcios2.1 Limite2.1.1 Exer. de Fixação da p.1Exer í io 1. (a) 3; (b) o limite não existe. Cal u-lando os laterais: limx→b−

f(x) = 6; limx→b+

f(x) = 1.( ) 5.não existe limite em c: o grá o possui umaquebra.Exer í io 2. (a) Verdadeiro. (b) Falso: é inter-valo entrado (x − (−2) = x + 2) em −2 omraio 1: (−4,−1). ( ) Falso para x < 0. Corretoé √x2 = |x|. (d) Falso: o limite é 4. O valor dafunção no ponto não importa para o ál ulo dolimite.Exer í io 3. (a) Falso. Tome f(x) = 4; x ≤ 3;

5; x > 3,então quando x → 3− o limite é 4. Assim, neste aso o limite não existe.(b) Falso. O limite quando x → 2− é 4 poisa existên ia do limite impli a na existên ia doslimites laterais ( om o mesmo valor).( ) Falso. Tome f(x) =

4; x 6= 2;

5; x = 2, entãoo limite quando x → 2 é 4 mas f(2) = 5.(d) Falso. Se o limite quando x → 3 existe,os laterais existem e assumem o mesmo valor.Exer í io 4. (a) lim

x→1−f(x) = 5, lim

x→1+f(x) = 7,

limx→1−

f(x) não existe.(b) todos limites são 5.( ) todos limites são 7.(d) limx→2−

f(x) = 7, limx→2+

f(x) = 9, limx→2−

f(x)não existe.(e) todos limites são 7.(f) todos limites são 9.009.set.2011

Exer í io 5. (a) a função alterna entre 1, quandocos(x) > 0, e −1, quando cos(x) < 0. Nospontos onde cos(x) = 0 ela não está denida.

x

y

f(x) =cos(x)

| cos(x)|

−5π2 −3π

2−π

2π2

3π2

5π2

y = 1

y = −1

(b)x

yf(x) =

|x|

Exer í io 6.(a) Translação verti al de uma unidade dográ o de √x.

x

y

(a) y = 1 +√x

1

(b) Translação verti al de duas unidades dográ o de sen(x).55

Page 60: Sumario´ - labMA/UFRJmcabral/livros/livro-calculo/... · 2019. 6. 12. · Sumario´ 1 Enunciados dos Exercícios 1.1 Exercícios de Limite. 1 1.1.1 Exercícios de Fixação. 1 1.1.2

56 CAPÍTULO 2. RESPOSTAS DOS EXERCÍCIOSx

y

(b) y = 2 + senx

1

2

3

( ) translação horizontal do log por uma uni-dade seguido por translação verti al de duas uni-dades (faça duas guras antes de obter a respostaabaixo).x

y

( ) y = log(x− 1) + 2

1 2

2

(d) Translação horizontal do grá o de−1/x3,que é pare ido om o grá o de −1/x.x

y

(d) y =−1

(x+ 2)3y = −2

−2

(e) Raízes do polinmio: −1,−2. Esbo e ográ o da parábola (x+1)(x+2) e depois reitaem torno do eixo x (efeito do módulo).

x

y

−2 −1(e) y = |(x+ 1)(x+ 2)|(f) Esbo e o grá o da parábola ex, transladeverti almente em 2 unidades e depois reita emtorno do eixo x (efeito do módulo). A hamos oponto de reexão resolvendo ex − 2 = 0, o queimpli a que x = log(2).x

y

(f) y = |ex − 2|

log(2)y = −2Exer í io 7. (a) −1 ( an ele termos iguais). (b)1/2 ( an ele x no numerador e denominador). ( )0 (somente numerador se anula).Exer í io 8.Dizemos que o limite de f(x) quandox tende a +∞ é +∞ se f(x) a tão grande epositivo quanto quisermos para todo x grande osu iente.Exer í io 9. (a) Análise simples de dois termosquadráti os. Será positiva em [−

√3,−1) e em

(1,√3]. (b) O termo x3 − 1 possui a raiz 1.Pelo Teorema D'Alembert pode ser fatorado por

x − 1. Fazendo divisão de polinmios obtemosque x3 − 1 = (x − 1)(x2 + x + 1). Cal ulandoDelta, vemos que o segundo polinmio possui 2raízes omplexas. Como a > 0, o termo x2+x+1 ≥ 0. Fazendo quadro de sinais om x− 1, x ex2−4 (podemos ignorar o termo sempre positivox2+x+1) obtemos que será negativa em (−2, 0)e [1, 2).Exer í io 10. (a) Raízes são −3, 1, 2.

−3 1 2

x− 2 − − − +

x+ 3 − + + +

1− x + + − −0 0 0

p(x) + − + −

Page 61: Sumario´ - labMA/UFRJmcabral/livros/livro-calculo/... · 2019. 6. 12. · Sumario´ 1 Enunciados dos Exercícios 1.1 Exercícios de Limite. 1 1.1.1 Exercícios de Fixação. 1 1.1.2

2.1. LIMITE 57x

y

−3 1 2

(a) p(x) = (x− 2)(x + 3)(1 − x)(b) Raízes são −1, 2.−1 2

(x− 2)2 + + +

x+ 1 − + +

0 0q(x) − + +

x

y

−1 2

(b) q(x) = (x− 2)2(x+ 1)( ) Raízes são 2, 3, 5.2 3 5

3− x + + − −(x− 2)2 + + + +

x− 5 − − − +

0 0 0r(x) − − + −

x

y

2 3 5

( ) r(x) = (3− x)(x− 2)2(x− 5);Exer í io 11. (a) −∞. (b) +∞. ( ) −1. (d) (afunção vale x2 para x > 0 e −x2 para x < 0)0. (e) não existe pois depende de qual lado seaproxima. (f) −∞ (0 + 1/0− = 0−∞ = −∞).(g) +∞.Exer í io 12. (a) 1. (b) +∞. ( ) 6. (d) +∞.(e) +∞. (f) +∞. (g) 3. (h) 5/4. (i) +∞.Exer í io 13. (a) não pode; (b) pode.Exer í io 14. (a) Falso. Se q(x) = x−1 o limitenão existe; se q(x) = −(x− 1)2 o limite é −∞.(b) Falso. Se f(x) = q(x) então o limite será1. ( ) Verdadeiro. O denominador vai para −1.Assim, 0/(−1) = 0 (não é indeterminação).Exer í io 15.A ondição (i) ex lui a letra (b).Tanto (iii) quanto (iv) ex lui letra (d). Final-mente a letra ( ) não representa uma função:qual valor de f(0.99999)? São três possibilida-des: logo não é função. Resposta: (a).Exer í io 16.

x

y

(a) 1

1

2

−2

x

y

(b) 1

1

2

Page 62: Sumario´ - labMA/UFRJmcabral/livros/livro-calculo/... · 2019. 6. 12. · Sumario´ 1 Enunciados dos Exercícios 1.1 Exercícios de Limite. 1 1.1.1 Exercícios de Fixação. 1 1.1.2

58 CAPÍTULO 2. RESPOSTAS DOS EXERCÍCIOSx

y

( ) 1

1

2

Exer í io 17. (a) omo seno é limitado por ±1,temos que −√

|x| ≤√

|x| sen(1/x) ≤√

|x|.Apli ando o Teorema do Sanduí he, on luímosque o limite é 0.(b) substituindo variável, o limite é 3. ( )substituindo variável, o limite é e5. (d) −∞. (e)e−2 (fazendo y = −2x).Exer í io 18. (a) É falso. O limite pode não exis-tir. Por exemplo g des ontínua em x = 3/2:g(x) = 1 para x ≤ 3/2 e g(x) = 2 aso ontrá-rio. (b) Como −1 ≤ cos(y) ≤ 1,

− 1

x2≤ cos(

√x2 + 1)

x2≤ 1

x2.Assim, pelo Teorema do Sanduí he, omo

limx→+∞

−1

x2= lim

x→+∞1

x2= 0,

limx→+∞

cos(√x2 + 1)

x2= 0.2.1.2 Problemas da p.4Problema 1.

x

y

−3 3

−3(a) f(x) = −√9− x2; |x| ≤ 3

|x| − 3; |x| > 3.

x

y

1

1

(b) f(x) = √x− 1; x ≥ 1;

log(x) + 1; x < 1.Problema 2. (a) e (b) o limite é 0. Em ( ) olimite não existe pois os ila entre 0 e 1.Problema 3. (a) 1 (ra ionalize o numerador). (b)4 (note que para x próximo de 4, |x| = x e ra i-onalize). ( ) −1/2 (ra ionalize).Problema 4. (a) não existe pois o valor os ilaentre 1 e −1.(b) −∞.( ) para x > 2, omo |x− 2| = x− 2, an e-lamos os termos e a função é x+ 1. para x < 2, omo |x − 2| = 2 − x obtemos que a função é−(x+1). Assim para x → 2+ o limite é 2+1 = 3;para x → 2− o limite é −(2 + 1) = −3. Logo olimite não existe.(d) Para x próximo de −5 o numerador é sem-pre negativo ( er a de −2). Assim para x → −5+o limite é −∞; para x → −5− o é +∞. Logo olimite não existe.Problema 5. (a) −∞. (b) −∞. ( ) −∞ ( 1x −1x2 = x−1

x2 ). (d) 1 (para x → 2−, |x − 2| =2 − x. (e) −1 (para x → −2, |x| = −x). (f) 4(note que 2 é raiz dupla: a3 − 5a2 + 8a − 4 =(a − 1)(a − 2)2). (g) 0 (o limite é 0/3 = 0).(h) −2. (i) 9/7 (trivial). (j) 3 (rearrumando onumerador obtemos (x2 + x − 2)/x). (k) 0. (l)3 (x3 + 1 = (x+ 1)(x2 − x+ 1)). (m) 1.Problema 6. (a) −1 (para x pequeno, numeradorvale √

x2 = −x). (b) −2/3. ( ) √10/5;(d) +∞. (e) +∞ (para x pequeno, vale−3y3/(

√10y2)). (f) sen(−2) (para x pequeno,numerador vale 4

√x6 = −4x3).Problema 7. (a) eab (mude variável para y =

ax). (b) 0. ( ) se b < 0 obtemos+∞ (+∞+∞).O aso interessante é se b > 0 (+∞−∞). Nesta aso, se c > b2 o limite é +∞, se c < b2 olimite é −∞, se c = b2 o limite é a/(2b). (d)a−b2√c(ra ionalizando). (e) b−a

2√c(ra ionalizando: uidado que aqui √x2 = −x!).Problema 8. (a) 0. (b) b/a. ( ) +∞ se c > 0,(note que 2m > m + 2 se m > 10) . (d) a/c.(e) √c/a. (f) 0.

Page 63: Sumario´ - labMA/UFRJmcabral/livros/livro-calculo/... · 2019. 6. 12. · Sumario´ 1 Enunciados dos Exercícios 1.1 Exercícios de Limite. 1 1.1.1 Exercícios de Fixação. 1 1.1.2

2.1. LIMITE 59Problema 9. (a) quando x → 0− é 1, quandox → 0+ é 0.(b) para x > 0 a função vale 1/x− 1/x = 0,para x < 0 vale 1/x − (−1/x) = 2/x. Assimquando x → 0+ é 0, quando x → 0− é −∞.Problema 10.Assintotas verti ais: x = −2 ex = 4. Assintota horizontal: y = 4.

x

y

−3 3

y = 4

x = 4x = −2

Problema 11. (a) É uma pegadinha, pois pode-mos simpli ar a função para (x+1)(x−1)/(x−1) = x+ 1 para x 6= 1 (função não esta denidano 1). Assim a função é a reta y = x + 1 massem estar denida em x = 1.

x

yy = x+ 1

(a) y =x2 − 1

x− 1

−1 1

2

(b) O sinal da função é dado pelo denomina-dor, já que o numerador é sempre positivo (igual a1). O sinal é: |x| > 1 a função é positiva, |x| < 1é negativa. Assintotas verti ais (quando denomi-nador se anula): x = ±1. A assíntota horizontalé y = 0 (o eixo x) pois o no ±∞ é 0.

x

y

(b) y =1

x2 − 1

x = 1x = −1( ) Como o denominador é sempre positivo(x2 + 1 > 0 para todo x), o sinal da função éo mesmo do numerador: positiva para x > 0 enegativa para x < 0. Como o denominador nun ase anula, não possui assintotas verti ais. Como olimite no ±∞ é 0, possui assintota horizontal y =0 (eixo x). A função passa no (0, 0). Note queela tem que ser positiva para x > 0 e onvergirpara 0 no +∞. Com estas informações zemoso esboço mais simples possível.

x

y

( ) y =x

x2 + 1(d) Assintotas verti ais (denominador se anula):x = 0 e x = 2. Assíntotas horizontais (limite no±∞): y = 1. Fazendo o quadro de sinais obte-mos o omportamento perto das assintotas.

x

y

(d) y =x2 − 1

x(x− 2)

x = 2

y = 1

(e) Assintotas verti ais (denominador se anula):x = 2 e x = −2. Assíntotas horizontais (limite

Page 64: Sumario´ - labMA/UFRJmcabral/livros/livro-calculo/... · 2019. 6. 12. · Sumario´ 1 Enunciados dos Exercícios 1.1 Exercícios de Limite. 1 1.1.1 Exercícios de Fixação. 1 1.1.2

60 CAPÍTULO 2. RESPOSTAS DOS EXERCÍCIOSno ±∞): y = −3. Fazendo o quadro de sinaisobtemos o omportamento perto das assintotas.x

y

(e) y =3x2 − 3

4− x2

x = 2x = −2

y = −3−1 1

Problema 12.Para (a) e (b). O grá o de h(x)é formado por duas retas pontilhadas: uma emy = x, a ima dos ra ionais e outra no y = −x,a ima dos irra ionais (vide gura abaixo). Logoem (a) e (b) o limite não existe.

x

yy = xy = −x

h(x)Para ( ) e (d): O grá o de h(x)/x2 é for-mado por duas retas pontilhadas: uma em y =1/x, a ima dos ra ionais e outra no y = −1/x,a ima dos irra ionais

x

y

h(x)

x2Logo em ( ) o limite não existe: nos ra ionaisvai para +∞, nos irra ionais para −∞. Em (d)

o limite é 0.Em (e): O grá o de h(x)/x é formado porduas retas pontilhadas: uma em y = 1, a imados ra ionais e outra no y = −1, a ima dos irra- ionaisx

y

y = 1

y = −1

h(x)

xLogo em (e) o limite não existe.Problema 13. (a) Pelo Teorema do Sanduí he olimite é 0.(b) quando x → 5, |f(x) − 3| → 0. Logof(x) → 3.Problema 14. (a) 4. (b) 3 (troque variável paray = 1/x2). ( ) 1/3 ( oloque o cos em evidên ia).(d) 2/5. (e) 0 (use Teorema do sanduí he e limiteo seno ompli ado por ±1). (f) e−10; (g) Troquevariável para y = x − π. Assim, x = π + y.Assim sen(π+y) = senπ cos(y)+sen(y) cos π =

− sen y. Pelo limite fundamental, limy→0

− sen y

y=

−1.(h) Pelo limite fundamental e pela deniçãode módulo, dará 1 se x → 0+ e −1 se x → 0−.2.1.3 Extras da p.6Extra 1.(a) Come e om o grá o de sen e faça ree-xão em torno do eixo x obtendo grá o de | sen |.Depois faça translação verti al por uma unidade.

x

y

π2

3π2

π2−π

2−3π2 (a) y = | sen(x)| − 1(b) Come e om y = |x|. Faça translaçãoverti al de uma unidade. Reita o grá o no eixo

x novamente.

Page 65: Sumario´ - labMA/UFRJmcabral/livros/livro-calculo/... · 2019. 6. 12. · Sumario´ 1 Enunciados dos Exercícios 1.1 Exercícios de Limite. 1 1.1.1 Exercícios de Fixação. 1 1.1.2

2.1. LIMITE 61x

y

(b) y = ||x| − 1|( ) Come e transladando horizontalmente ográ o de módulo por duas unidades. Depoistranslade verti almente por uma unidade.x

y

( ) y = |x+ 2| − 1

−2

Extra 2.x

y

x = 1

y = 4

1

−2Extra 3.Como sengr(x) = sen(πx/180) (assimsengr(90) = sen(π90/180) = sen(π/2), substi-tuindo variável obtemos que o limite vale π/180.Extra 4. (a) Para x > 0, y = x+ |x| = x+ x =2x, para x < 0, y = x + |x| = x + (−x) = 0.Assim o grá o é:

x

yy = 2x

(a) y = x+ |x|(b) Por denição, x ≥ ⌊x⌋. Além disso adiferença 0 ≤ f(x) = x−⌊x⌋ < 1. Por exemplo,

para x no intervalo [0, 1), ⌊x⌋ = 0 e portantof(x) = x − 0 = x. Para x no intervalo [1, 2),⌊x⌋ = 1 e portanto f(x) = x − 1. Para x nointervalo [−1, 0), ⌊x⌋ = −1 e portanto f(x) =x− (−1) = x+ 1. Assim o grá o é:

x

y

(b) y = x− ⌊x⌋−2 −1 1 2

y = 1

Extra 5. (a) Não existe pois quando x → 1+ vale1, x → 1− vale −1. (b) +∞. ( ) 8/6 = 4/3.(d) 0 (aplique Teorema do Sanduí he e use queseno é limitado por 1 e −1).Extra 6. (a) 0; (b) 3 (para x grande, 2x+ |x| =2x + x = 3x). ( ) 1 (para x pequeno, 2x +|x| = 2x − x = x). (d) −∞ (para x pequeno,x+ |x|+ 1 = x− x+ 1 = 1).Extra 7. (a) 0. (b) 0.Extra 8.

x

y

y = 1

y = 2

(a) f(x)x

y

y = x

y = x2

(b) g(x)2.1.4 Desaos da p.7Desao 1. (a) Para x grande (basta que x > 1)temos que 0 < 1x < 1. Assim para x > 1, ⌊ 1x⌋ =

0. Logo o limite vale zero pois a função vale zeropara x > 1.(b) Para x pequeno (basta que x < −1) te-mos que −1 < 1x < 0. Assim para x < −1,

Page 66: Sumario´ - labMA/UFRJmcabral/livros/livro-calculo/... · 2019. 6. 12. · Sumario´ 1 Enunciados dos Exercícios 1.1 Exercícios de Limite. 1 1.1.1 Exercícios de Fixação. 1 1.1.2

62 CAPÍTULO 2. RESPOSTAS DOS EXERCÍCIOS⌊ 1x⌋ = −1. Como a função vale −x para x < −1,o limite vale +∞ = −(−∞).

x

y

y = 1

y = −x

y = 1− x11

213

−1 −12( ) y = x⌊ 1x⌋(d) Vamos utilizar o Teorema do Sanduí he.Para x > 0, observe que ⌊1/x⌋ vale no máximo

1/x e, no mínimo 1/x− 1. Logo,1

x− 1 ≤

1

x

≤ 1

x.Multipli ando ambos os lados por x (que é posi-tivo e não altera as desigualdades) obtemos que

1 − x ≤ x⌊ 1x⌋ ≤ 1. Passando ao limite x → 0+e apli ando o Teorema do Sanduí he obtemos a onvergên ia para 1.Para x < 0 fazemos um ra io ínio similar para on luir que 1 ≤ x⌊ 1x⌋ ≤ 1 − x. Passando ao li-mite x → 0− e apli ando o Teorema do Sanduí heobtemos a onvergên ia para 1. Como os limiteslaterais são iguais, o limite existe e é igual a 1.Desao 2. (a) Note que trata-se de uma indeter-minação do tipo ∞0. Em uma linha: ex res emuito mais rápido do que x. Assim para x grande,(ex + x)1/x ≈ (ex)1/x = e. Com rigor, olo-que ex em evidên ia: (ex(1 + x/ex))1/x = e(1 +x/ex)1/x. Agora o termo (1 + x/ex) → 1 e1/x → 0. Assim (1 + x/ex)1/x → 10 = 1.(b) Note que trata-se de uma indeterminaçãodo tipo ∞0. Em uma linha: Como em (a), para xgrande (1 + x) ≈ x. Assim temos que al ular olimite xα/ log x. Troque variável para y = log(x)(assim x = ey): al ule o limite (ey)α/y → eα.Desao 4.Embora o denominador se anule nolimite, o sinal dele alterna sempre. Assim o nolimite a função os ila entre +∞ e −∞. Perto

do zero teremos um innidade de pontos onde afunção se aproxima de ±∞.Desao 7.Como |f(x)| ≤ M , −M ≤ f(x) ≤M . Logo, −M |g(x)| ≤ f(x)g(x) ≤ M |g(x)|.Como lim

x→1g(x) = 0, lim

x→1|g(x)| = 0. Apliqueo Teorema do Sanduí he depois de veri ar que

limx→1

−M |g(x)| = limx→1

M |g(x)| = 0.Desao 8. (a) +∞; (b) +∞; ( ) +∞; (d) +∞;(e) 0Desao 9.Detalhando ( ):0 <

+∞∑

n=q+1

q!

n!=

1

q + 1+

1

(q + 2)(q + 1)+ . . .

<1

2+

1

4+

1

8+ · · · = 1.

2.2 Continuidade2.2.1 Exer. de Fixação da p.8Exer í io 1. (a) Falso. O limite deve ser igualao valor da função no ponto. Exemplo: f(x) =x

x; x 6= 0;

2; x = 0;O limite no zero é 1 mas f(0) = 2.(b) Verdade. Se f é ontínua o limite existe.Se o limite existe, ambos limites laterais existem.( ) Falso. O limite pode ser igual, omo no ontraexemplo do item (a) deste exer í io.Exer í io 2. (a) Somente é ontínua em A.(b) Em B e D, embora o limite exista, ele di-fere do valor da função no ponto: o grá o possuium salto. Em C, os limites laterais existem masdiferem entre si. Assim não existe limite em C:o grá o possui uma quebra.( ) A des ontinuidade é removível somenteem B e D, pois o limite existe e basta redenir afunção no ponto; em C, para qualquer valor que se oloque na função em x = C a função ontinuarásendo des ontínua.Exer í io 3. (a) somente (I). Note que (II) e (III)são des ontínuas em 0 e −2 respe tivamente. (b)(I), (II) e (III). ( ) (I) e (III).Exer í io 4. (a) f(x) =

1; x ≤ 1;

2; 1 < x < 2;

3; 2 ≤ x.012.set.2011

Page 67: Sumario´ - labMA/UFRJmcabral/livros/livro-calculo/... · 2019. 6. 12. · Sumario´ 1 Enunciados dos Exercícios 1.1 Exercícios de Limite. 1 1.1.1 Exercícios de Fixação. 1 1.1.2

2.2. CONTINUIDADE 63(b) A função parte inteira (veja Figura nap. 61) para x > 0: f(x) = 0; x ≤ 0;

⌊x⌋; x > 0.Exer í io 5. (a) não existe valor possível pois oslimites laterais são distintos: a des ontinuidadenão é removível.(b) Como o limite é +∞, que não é um nú-mero, não existe k. Se pudéssemos olo ar valorinnito, olo aríamos k = +∞.( ) Pelo Teorema do Sanduí he o limite quandox → 0 é zero. Assim oloque k = 0 para tornara função ontínua.Exer í io 6. (a) Falso. Pode ter. Basta os ilarentre estes pontos.(b) Verdadeiro: pelo menos uma em [2, 3] epelo menos uma e, [3, 4], onde a função tro a desinal.( ) Falso. O TVI garante pelo menos uma,mais pode ter mais de uma.Exer í io 7. (a) Falso. Quando nas e uma ri-ança a função dá um pulo de uma unidade ins-tantaneamente: não existe 1/5 de habitante et .(b) Verdadeiro. Nos res emos diariamenteuma quantidade innitamente pequena. Nossaaltura não dá pulos.Exer í io 8. (a) Falso. Se f(1/2) = −10 tería-mos vários pontos om valor negativo.(b) Falso. Se f for des ontínua pode não terraiz.Exer í io 9. (a) Errado. O orreto é se K ∈[2, 5], então existe c ∈ [−3,−1] tal que f(c) =K; (b) Correto pois se K ∈ [3, 4] então K ∈[2, 5]. Logo, pelo TVI, existe c ∈ [−3,−1] talque f(c) = K.( ) Errado. O intervalo [0, 3] NO está on-tido em [2, 5].Exer í io 10.Como f é ontínua, pelo Teorema,f ·f = f2 (produto de funções ontínuas) é ontí-nua. Assim, pelo Teorema novamente, f · (f2) =f3 (produto de funções ontínuas) é ontínua.Também pelo Teorema, 5f3 ( onstante vezes fun-ção ontínua) é ontínua. Pelo Lema, x é ontí-nua. Pelo Teorema x ·x = x2 produto de funções ontínuas) é ontínua. Pelo Lema 1 (função ons-tante) é ontínua. Pelo Teorema, x2 + 1 (somade funções ontínuas) é ontínua. Finalmente,pelo Teorema h, que é o quo iente de funções ontínuas, é ontínua.

2.2.2 Problemas da p.9Problema 1. (a) Nos pontos onde o denominadorse anula f(x) → ±∞. Nestes pontos a funçãoé des ontínua. Nos outros pontos, omo se tratada divisão de funções ontínuas (1 e sen(x)), elaé ontínua. R: 0,±π,±2π,±3π,.(b) O denominador nun a se anula pois cos(x)vale no mínimo −1: assim 2 − 1 = 1 ≤ 2 +cos(x) ≤ 2 + 1 = 3 para todo x ∈ R. Portanto, omo g é quo iente de funções ontínuas om de-nominador que nun a se anula, g é ontínua emR e o onjunto dos pontos de des ontinuidade é∅ (vazio).( ) Veja o grá o na Figura da p. 61. O on-junto dos pontos de des ontinuidade é Z.(d) Esbo e o grá o: uma úbi a pontilhadae uma reta pontilhada. É ontínua onde elas se ruzam (porque?) nos pontos onde x3 = x, istoé, em x = 0, x = 1 e x = −1. É des ontínua emR− 0, 1,−1.Problema 2.Cal ulando os limites no 0:lim

x→0−f(x) = |0+2| = 2, lim

x→0+f(x) = 3−0 = 3.Como eles diferem no 0, não existe lim

x→0f(x) eportanto a função é des ontínua no 0. Nos outrospontos é ontínua.Para x grande e negativo, f(x) = |x − 2|.Assim lim

x→−∞f(x) = lim

x→−∞|x− 2| = +∞.Problema 3. (a) Note que f(0) = 0 < 10 e que

limx→+∞

f(x) = +∞ (veja Exemplo 43 da p.54).Logo existe M > 0 tal que f(M) > 10. PeloTVI existe c ∈ [0,M ] tal que f(c) = 10.(b) Dena h(x) = log(x) − e−x. Queremosen ontrar b > 0 tal que h(b) = 0. Quandox → 0−, log(x) → −∞ e e−x → 1. Logo,lim

x→0−h(x) = −∞. Quando x → +∞, log(x) →

+∞ e e−x → 0. Logo, limx→+∞

h(x) = +∞. As-sim existem M,N om 0 < M < N e tais queh(M) < 0 e h(N) > 0. Como h é ontínua, peloTVI existe d ∈ [M,N ] tal que h(b) = 0.( ) Dena g(x) = f(x) − x. Se g(c) = 0,então f(c) = c. Note que g(0) = f(0) − 0 =f(0) ≥ 0 e g(1) = f(1) − 1 ≤ 0. Se em umdos extremos g se anular nos teremos obtido oc. Caso ontrário, g(1) < 0 < g(0). Pelo TVI(g é ontínua pois é a subtração de duas funções ontínuas), existe c ∈ [0, 1] om g(c) = 0. Esteresultado é uma versão simpli ado do Teoremado Ponto Fixo de Brower.(d) Suponha, por ontradição, que não é ver-

Page 68: Sumario´ - labMA/UFRJmcabral/livros/livro-calculo/... · 2019. 6. 12. · Sumario´ 1 Enunciados dos Exercícios 1.1 Exercícios de Limite. 1 1.1.1 Exercícios de Fixação. 1 1.1.2

64 CAPÍTULO 2. RESPOSTAS DOS EXERCÍCIOSdade que f(x) < 0. Assim, existiria um t ∈ [0, 2] om f(t) ≥ 0. Como f não se anula em [0, 2], naverdade f(t) > 0. Como f(−1) = −3, apli andoo TVI em [1, t] (f é negativa em 1 e positiva emt) on luímos que existe um c ∈ [1, 2] tal quef(c) = 0. Como isto é um absurdo, on luímosque f(x) < 2 no intervalo [0, 2].Problema 4. (a) Simplique o (x−2)2 no nume-rador e denominador. a = 5.(b) Impossível. Teríamos que ter a = 3 e −2ao mesmo tempo.( ) a = 1.(d) Impossível pois o limite em x = 0 nãoexiste.(e) Impossível pois teríamos que ter a = +∞,que não é um número real.(f) a = 3/4.Problema 5.Temos que resolver o sistema

2a+ b = |2− 1| = 1,−2a+ b = | − 2− 1| = 3.Obtemos a = −1/2, b = 2.Problema 6. Suponha que não e que existam

a, b ∈ R, a 6= b, tais que f(a) 6= f(b). Como osirra ionais estão em todo lugar em R (são den-sos em R), existe um irra ional k entre f(a) ef(b). Como f é ontínua, pelo TVI existe c ∈ Rtal que f(c) = k é irra ional. Contradição poisassumimos que f(x) é ra ional para todo x.2.2.3 Extras da p.10Extra 1.Ela somente é ontínua em x = 0 poisse x está próximo de 0 e x ∈ Q então f(x) = 1e se x 6∈ Q então f(x) ≈ 1 + |0| = 1. Logo olimite quando x → 0 é 1, que é igual ao valor dafunção. Logo é ontínua em x = 0.Em qualquer x 6= 0 o limite não existe pois seestá próximo de x 6= 0 e x ∈ Q então f(x) = 1e se x 6∈ Q então f(x) ≈ 1 + |x| 6= 1. Logoo onjunto dos pontos de des ontinuidade é R−1.Extra 2. (a) Dividindo-se por (x− 1) duas vezeso numerador e o denominador, vamos obter o li-mite. Logo a = −1. (b) a = 2. ( ) Impossível.(d) a = 0. (e) a = −1 +

√2 ou a = −1 −

√2.(f) Impossível. Geometri amente, um reta saindoda origem não temo omo ompletar de forma ontínua uma função que valia 1 para x < 0.Extra 3. (a) Falso. Pode ter raiz no meio (penseem algo do tipo seno, que os ila).

(b) Não. O TVI garante pelo menos duasraízes, mas não exatamente duas.Extra 4. (a) Dena f(x) = x + 2 sen(x) − 1.Como f(0) = −1 e f(π) = π − 1 > 0, pelo TVIf possui raiz.(b) Se o polinmio p é de grau impar omtermo de maior grau axk então, se k > 0,lim

x→−∞p(x) = −∞ e lim

x→+∞p(x) = +∞. Assimexistem M,N tais que p(M) < 0 e p(N) > 0.Como p é ontínua, pelo TVI existe c ∈ [M,N ] ⊂

R tal que p(c) = 0 ∈ [p(M), p(N)]. Se k < 0então (os limites se invertem) limx→−∞

f(x) = +∞e limx→+∞

f(x) = −∞ e o resto é análogo.( ) Dena h(x) = sen(π sen(x)) − sen(x).Como h(π/6) = 1/2 e h(π/2) = −1 (sinaisopostos), pelo TVI existe c ∈ [π/6, π/2] tal queh(c) = 0, isto é, tal que sen(π sen(c)) = sen(c).(d) Como h(0) = 1, h(π) = h(−π) = 1−2 =−1, apli ando o TVI nos intervalos [−π, 0] e [0, π]vamos obter duas raízes distintas para h.Extra 5.Temos que resolver o sistema

a+ b = 1,4a+ b = −8.Obtemos a = −3, b = 4.Extra 6.Use a denição utilizando exponen ialda p.60 e explore propriedades da exponen ial.2.2.4 Desaos da p.11Desao 1.Esbo e dois grá os olo ando no eixo

x a hora e no eixo y os pontos do per urso. Noprimeiro dia a função omeça do iní io do per- urso e termina no m. No dia seguinte, omeçano m do per urso e termina no iní io. Comoos per ursos são funções ontínuas, os grá os se ruzam em pelo menos um ponto, o que signi- a passar na mesma hora (em dias distintos) nomesmo ponto do per urso.Desao 2.Ver [Sp p.70 no.17.Desao 3.Ver [Sp p.98 no.6.Desao 4.Pelas propriedades do logaritmo, o-lo ando a em evidên ia,log(a+h) = log(a(1+

h

a)) = log(a)+log(1+

h

a).Quando h → 0, tro ando variável vemos que

log(1 + h/a) → log(1) = 0. Assim obtemos a ontinuidade de log.

Page 69: Sumario´ - labMA/UFRJmcabral/livros/livro-calculo/... · 2019. 6. 12. · Sumario´ 1 Enunciados dos Exercícios 1.1 Exercícios de Limite. 1 1.1.1 Exercícios de Fixação. 1 1.1.2

2.3. DERIVADA 65Desao 5.Basta apli ar as expansões em série daexponen ial, seno e osseno. Depois basta agru-par os termos om e sem i e utilizar as identi-dades: i0 = i4 = i8 = · · · = 1, i1 = i5 =i9 = · · · = i, i2 = i6 = i10 = · · · = −1,i3 = i7 = i11 = · · · = −i. Assim,eiθ = 1 + (iθ) +

(iθ)2

2+

(iθ)3

3!+

(iθ)4

4!+

(iθ)5

5!· · ·

= 1 + iθ − θ2

2− i

θ3

3!+

θ4

4!+ i

θ5

5!· · ·

= 1− θ2

2+

θ4

4!− · · ·+ i

(

θ − θ3

3!+

θ5

5!· · ·)

= cos θ + i sen θ.Desao 6.Basta fazer ontas an elando a partereal ou a imaginária.Desao 7.Dena J = f(I). Dados x, y ∈ J , om x < y sem perda de generalidade, vamosmostrar que dado z ∈ (x, y), z ∈ J (imagem).De fato, omo x, y ∈ J = f(I), existem u, v ∈ Itais que f(u) = x, f(v) = y. Pelo TVI, omoz ∈ (x, y), existe w ∈ [u, v] (ou [v, u]) tal quef(w) = z. Note que w ∈ I pois I é um intervalo.Portanto f(w) ∈ f(I) = J é um intervalo.Desao 8. (a) ∀M < 0,∃δ > 0 tal que se 0 <x− c < δ, então f(x) < M . (b) ∀M > 0,∃N <0 tal que se x < N , então f(x) > M . ( ) ∀ε >0,∃δ > 0 tal que se 0 < c−x < δ, então |f(x)−L| < ε.2.3 Derivada2.3.1 Exer. de Fixação da p.12Exer í io 1. y − 3 = (x− (−2)) · 3 = 3(x+ 2).Assim a reta tangente é y = 3x+ 9.Exer í io 2. (a) Falso. f(x) = |x− 3| possui umbi oem x = 3.(b) Falso. f(x) = 0 e g(x) = x − 2. Entãof(2) = g(2) = 0 mas f ′(2) = 0 e g′(2) = 1.( ) Falso. f(x) = x−10. f ′(1) = 1 e f(1) =−9.Exer í io 3. (a) Como no intervalo [x1, x3] a fun-ção é um segmento de reta, f ′(x1) = 2 = f ′(x2).Note que f ′(x3) não existe pois grá o possui umbi o.007.out.2011

(b) Note que f ′(x5) = 0 ou algo próximo eque f ′(x6) > f ′(x2) pois a in linação da reta tan-gente é maior em x6. Também f ′(x4) < 0 poisa função de res e ai. Assim, f ′(x4) < f ′(x5) <f ′(x2) < f ′(x6).Exer í io 4.

x

y

f ′(x)

4 12−5

8Exer í io 5. (a) h′(2) = f ′(2)g(2)+f(2)g′(2) =−1(−5) + 3(2) = 11.(b) h′(2) = f ′(2)g(2) − f(2)g′(2)

g(2)2.Logo h′(2) =

−1(−5) − 3(2)

(−5)2=

−1

25.Exer í io 6.Cal ulando o oe iente angular dareta tangente, f ′(1) = 1 e g′(1) = 0 (reta tan-gente horizontal). Assim:(a) f ′(1)g(1)+g′(1)f(1) = 1(3/2)+0(1) = 3/2.(b) 5f ′(1)− 3g′(1) = 5(1)− 3(0) = 5.Exer í io 7. (a) Velo idade é h′(t) = −32t. As-sim h′(2) = −64.(b) Quando h(t) = 0? Para t =√125/2.( ) Velo idade h′(

√125/2) = −16

√125.(d) A eleração h′′(t) = (−32t)′ = −32. As-sim a a eleração é −32 para todo t.Exer í io 8. (a) ex log x+ ex

x . (b) − senx(x+5)−cos x(x+5)2

.( ) − sen(x3 + 1)(3x2). (d) 0 (a função é ons-tante em relação a x). (e) cos x

1 + senx. (f) Para

x > 2 a derivada é 1, para x < 2 a derivada é−1. Em x = 2 a derivada não existe.Exer í io 9. (a) 4πr2. (b) 6k+ 1

k2. ( ) log t+1.(d) πsπ−1. (e) √

3 = elog√3. Logo (

√3)x =

ex log√3. Assim a derivada é log(

√3)ex log

√3 =

(log√3)(

√3)x. (f) 0.Exer í io 10. (a) Pelo TVM, existe c ∈ [1, 5] talque f(5) − f(1) = f ′(c)4. Multipli ando por

4 a desigualdade −4 ≤ f ′(x) ≤ 3 obtemos oresultado.( ) Pelo TVM, para todo h existe c ∈ [0, h] talque f(h)−f(0) = f ′(c)h. Como h > 0 podemosmultipli ar a desigualdade −4 ≤ f ′(x) ≤ 3 semalterar os sinais das desigualdades.Exer í io 11. Seja S(t) a altura do objeto emfunção do tempo. Então S(0) = 100, S(5) = 0.Assim, S(5)− S(0)

5− 0=

−100

5= −20. Pelo TVM

Page 70: Sumario´ - labMA/UFRJmcabral/livros/livro-calculo/... · 2019. 6. 12. · Sumario´ 1 Enunciados dos Exercícios 1.1 Exercícios de Limite. 1 1.1.1 Exercícios de Fixação. 1 1.1.2

66 CAPÍTULO 2. RESPOSTAS DOS EXERCÍCIOSexiste um instante t ∈ (0, 5) tal que S′(t) = −20,a velo idade do objeto.Exer í io 12.Como (f ′(x))′ = 0 para todo x ∈R, f ′(x) = onstante. Como f ′(−3) = 0, a ons-tante é zero. Assim on luímos que f ′(x) = 0para todo x ∈ R. Logo f(x) = onstante. Comof(5) = π, a onstante é π. Assim on luímosque f(x) = π para todo x ∈ R.Exer í io 13.(a) h′(2) = f ′(g(2))g′(2) = f ′(3)g′(2). Comog′(2) é o oe iente angular da tangente, g′(2) =(3−2)/(2−0) = 1/2. Do mesmo modo, f ′(3) =(0−2)/(3−0) = −2/3. Assim, h′(2) = −2

3 · 12 =−2

6 = −13 .(b) Como g(2) = 3, g−1(3) = k(3) = 2.Como k é a inversa de g, k(g(x)) = x. Logo,

k′(g(x))g′(x) = 1. Assim, k′(g(2))g′(2) = 1 ouk′(3)g′(2) = 1. Como g′(2) = 1

2 pelo item (a),k′(3) = 1/g′(2) = 2.Exer í io 14.Mar amos no grá o os pontos ondeele ruza o zero e onde a reta tangente é horizon-tal.

x

y

a b c d e f g(a) f ′ é positiva em (−∞, b), (c, d) e (f,+∞).f ′ é negativa em (b, c) e (d, f).(b) f é injetiva em (−∞, b), ou (b, c), ou(c, d), ou (d, f), ou (f,+∞).( ) f é res ente em (a, e) e (g,+∞). f éde res ente em (−∞, a) e (e, g).(d) f é injetiva em (−∞, a) ou (a, e) ou (e, g)ou (g,+∞).Exer í io 15.Dena A = cos(arcsen(x/a)). Comosen( y)+cos2(y) = 1, tomando y = arcsen(x/a)),sen(y) = x/a e assim x2/a2 + A2 = 1, ou seja,A =

1− x2/a2 = 1/a√a2 − x2.2.3.2 Problemas da p.15Problema 1. (a) f(x + h) − f(x) = 1

(x+h)2 −1x2 = x2−(x+h)2

x2(x+h)2 = −2xh+h2

x2(x+h)2 . Dividindo por hobtemos f(x+h)−f(x)h = −2x+h

x2(x+h)2. Fazendo h →

0 obtemos, f ′(x) = −2xx2(x)2

= − 2x3 .(b) f(x+h)−f(x) = 1√

x+h− 1√

x=

√x−

√x+h√

x+h√x.Multipli ando por √x+

√x+ h obtemos:

x−(x+h)√x+h

√x(√x+

√x+h)

= −h√x+h

√x(

√x+

√x+h)

.Dividindo por h obtemos:f(x+h)−f(x)

h = −1√x+h

√x(

√x+

√x+h)

.Quando h → 0 obtemos:f ′(x) = −1√

x√x(√x+

√x)

= − 12x

√x.( ) Para x > 0 temos que f(x) = x2 − x, uja derivada pela denição, é

f(x+ h)− f(x)

h=

2xh+ h2 − h

h= 2x+h−1.Assim, om h → 0, a derivada é 2x − 1. Deforma análoga, omo para x < 0 f(x) = x− x2,a derivada pela denição é 1 − 2x. Finalmente,para x = 0 temos que al ular pela denição:

f(0 + h) − f(0) = |h|(h − 1). Dividindo por hobtemos que f ′(0) = limh→0 |h|/h(h − 1) nãoexiste, pois pela direita o limite será −1 e pelaesquerda, 1.(d) Fazendo de forma análoga ao item ( ),para x > 0 a derivada é 2x, para x < 0 é −2x epara x = 0 o limite (f(0 + h) − f(0))/h é zero.Assim, f ′(0) = 0.Problema 2.Para garantir ontinuidade em x =1 devemos ter: (1)2 = a(1) + b, ou a + b = 1.Para que as derivadas laterais sejam iguais emx = 1 devemos ter 2x = a em x = 1, ou a = 2.Assim b = 1− a = −1.Problema 3.Primeiro note que 0 ≤ |f(0)| ≤|0|k = 0. Assim |f(0)| = 0, isto é, f(0) = 0.Agora pela denição, f ′(0) = lim

h→0

f(h)− f(0)

h=

limh→0

f(h)

h.Observe que 0 ≤ |f(h)|

|h| ≤ |h|kh = |h|k−1.Como k > 1, k − 1 > 0. Assim, lim

h→0|h|k−1 = 0.Logo, pelo teorema do Sanduí he, lim

h→0

|f(h)||h| =

0. Logo f ′(0) = limh→0

f(h)

h= 0.Problema 4. (a) Possui derivada em todos ospontos x 6= 2 igual a zero pois é onstante. Em

x = 2 é des ontínua e portanto também não éderivável em x = 2.(b) f(x) = ex − 1 se ex − 1 > 0, isto é, seex > 1. Tomando log dos dois lados, se x > 0.Assim, f(x) = ex−1 se x > 0 e f ′(x) = ex. Poroutro lado, f(x) = −(ex − 1) = 1− ex se x < 0.Assim f ′(x) = −ex se x < 0. Em x = 0 o grá opossui um bi o e a função não é derivável.( ) Fazendo análise de sinal do polinmio dosegundo grau (3−x)(x+1) (parábola om raízes

Page 71: Sumario´ - labMA/UFRJmcabral/livros/livro-calculo/... · 2019. 6. 12. · Sumario´ 1 Enunciados dos Exercícios 1.1 Exercícios de Limite. 1 1.1.1 Exercícios de Fixação. 1 1.1.2

2.3. DERIVADA 673 e −1 om on avidade para baixo), on luímosque h(x) = (3 − x)(x + 1) se −1 < x < 3 eh(x) = −(3 − x)(x + 1) aso ontrário. Assim,h′(x) = −2x + 2 se −1 < x < 3 e h′(x) =2x − 2 se x < −1 ou x > 3. Em x = −1 ex = 3 o grá o possui um bi o e a função nãoé derivável.Problema 5. (a) A velo idade é s′(t) = 4t

(t2 + 1)2.A a eleração é s′′(t) = 4(t2 + 1)2 − 16t2(t2 + 1)

(t2 + 1)4.Logo s′(0) = 0 e s′′(0) = 4. Ela vai parar quandoa velo idade s′(t) = 0, ou seja, quando t = 0.(b) A velo idade é s′(t) = cos t. A a eleraçãoé s′′(t) = − sen t. Logo s′(0) = 1 e s′′(0) = 0.Ela vai parar quando a velo idade s′(t) = cos t =

0, ou seja, quando t = 2kπ ± π/2 para k ∈ Z.Problema 6. (a) Nos pontos onde f ′(x) = 6x2−4x = 0, isto é, x = 0 ou x = 2/3.(b) Rees revendo a reta 2y − 20x − 50 = 0 omo y = 10x + 25, observamos que o oe i-ente angular é 10. Assim queremos saber quandof ′(x) = 6x2 − 4x = 10, isto é, x = −1, x = 5/3.( ) o oe iente angular da reta 4y + 2x −10 = 0 é −1/2. Para que seja perpendi ular, areta deverá ter oe iente = −1/(−1/2) = 2.Assim queremos saber quando f ′(x) = 6x2 −4x = 2, isto é, x = 1, x = −1/3.Problema 7.Deve-se analisar separadamente dois asos. Se x2 − 1 > 0, isto é x > 1 ou x <−1, a função é (x2 − 1)(x + 1), uja derivada é3x2+2x−1, ujas raízes 1/3 e −1 não perten emao domínio. Se x2 − 1 < 0, então −1 < x < 1e a função é (1 − x2)(x + 1), uja derivada é−3x2− 2x+1, ujas raízes são novamente 1/3 e−1. Assim em x = 1/3 a derivada é zero e a retatangente é paralelo ao eixo x. Em x = −1 temosque apli ar a denição. Cal ulando (f(−1) = 0)

f(x)− f(−1)

x− (−1)= |x2 − 1|.Quando x → −1 o limite tende a zero. Assim

f ′(−1) = 0 e x = −1 é ponto onde a reta tan-gente é paralela ao eixo x.Problema 8. (a) y′ = 3ax2 + 2bx+ c. Para quetenha uma úni a tangente horizontal, queremosque a equação y′ = 3ax2 + 2bx + c = 0 tenhasolução úni a. Para isto basta que ∆ = (2b)2 −4(3a)c = 0, isto é, que b2 = 3ac.(b) y′(x) = 3ax + b. O oe iente angularde x + y = 1 é 1 = y′(−1) = b − 3a. O oe- iente angular de y = −1 é 0 = y′(1) = 3a + b.

Resolvendo o sistema obtemos que b = 1/2 ea = −1/6. Assim c pode ter qualquer valor.Problema 9.(a) 300(5x2 − 3x+ 4)299(10x− 3).(b) Primeiro rees reva 7

√· = (·)1/7. Depoisapli ando a regra da adeia,cos(

(

cos(x2) + 4)1/7

)

·

· 17

(

cos(x2) + 4)−6/7

(− sen(x2))(2x).( ) 2xe−x + 2x+ e−xx2 + e−x

(e−x + 1)2.(d) 1/3(x + t)−2/3(x2 + k)− (x+ t)1/3(2x)

(x2 + k)2.(e) 5 cos(5 ex)exx4

sen(5 ex)+ 4 log(sen(5 ex))x3.(f) 6x

((log(3x2 + 1))2 + 1)(3x2 + 1).(g) − 5earcsen(4−5x)

1− (4− 5x)2.Problema 10. g′(x) = 3

f(x) + x(f ′(x)+1). As-sim, g′(4) = 3

f(4) + 4(f ′(4) + 1) = −12

7.Problema 11. (a) f ′(t) = 2

(ax+ b)(ad− bc)

(cx+ d)3.(b) f ′(t) = KeKt cos(at)− aeKt sen(at).( ) f ′(θ) = 3aKθ2 cos(aθ3 + b).(d) f ′′′(t) = −m0

K3 e(T0−t)/K .Problema 12. (a) y′ = cos(x2) · 2x2 − sen(x2)

x2.Logo y′(

π/2) = −2/π e y(√

π/2) =√

2/π.Assim a equação da reta tangente é: y−√2/π =−2/π(x −

π/2). Da reta perpendi ular é y −√

2/π = π/2(x−√

π/2).(b) y′ = esen(−2x) cos(−2x)(−2).Logo y′(π) = −2 e y(π) = 1. Assim a equaçãoda reta tangente é: y − 1 = −2(x− π). Da retaperpendi ular é y − 1 = 1/2(x − π).Problema 13. (a) Considere f(x) = ex−(1+x).Derivando f ′(x) = ex − 1 é positiva para x > 0.Logo f é res ente para x > 0. Como f(0) = 0,a função é positiva para x > 0.(b) Considere g(x) = 2x3 − 15x2 + 60x+ 4.Como limx→+∞

g(x) = +∞ e limx→−∞

g(x) = −∞,existem pontos onde a função é positiva e ne-gativa. Pelo TVI existe pelo menos uma raiz.Note que g′(x) = 6x2 − 30x + 60 é sempre po-sitivo (para todo x ∈ R) pois é um polinmiodo segundo grau om raízes omplexas (∆ < 0 e

Page 72: Sumario´ - labMA/UFRJmcabral/livros/livro-calculo/... · 2019. 6. 12. · Sumario´ 1 Enunciados dos Exercícios 1.1 Exercícios de Limite. 1 1.1.1 Exercícios de Fixação. 1 1.1.2

68 CAPÍTULO 2. RESPOSTAS DOS EXERCÍCIOSa = 6 > 0). Assim, g é res ente para todo Re portanto injetiva. Assim a raiz é úni a pois afunção é injetiva.Problema 14. (a) Suponha que f e g represen-tam a posição dos orredores em função do tempo.Por hipóteses f(0) = g(0) ( omeçam no mesmoinstante). Suponha que eles terminaram a orridano instante T . Assim, f(T ) = g(T ) (terminaramempatados). Se h = f − g, h(0) = h(T ) = 0.Pelo TVM (ou Teorema de Rolle), existe c ∈(0, T ) tal que h′(c) = 0 = f ′(c)− g− (c), isto é,f ′(c) = g′(c).(b) Pelo TVM, f(x)− f(0) = f ′(c)x. Comox > 0 e f ′(c) ≤ 1 para todo c > 0 e f(0) = 0,f(x) = f(x)− f(0) ≤ x.( ) Seguindo a di a, omo h′i = hi para i =1, 2,

f ′(x) =h′1h2 − h1h

′2

(h2)2=

h1h2 − h1h2(h2)2

= 0.Logo f é onstante. Como f(0) =h1(0)

h2(0)= 1,

f(x) = 1 para todo x ∈ R. Logo 1 =h1(x)

h2(x), istoé, h1(x) = h2(x) para todo x ∈ R.(d) Seja h = f − g. Como h(0) = h(1) = 0,pelo Teorema de Rolle, existe c ∈ (0, 1) tal que

h′(c) = 0 = f ′(c) − g′(c). Logo f ′(c) = g′(c) eportanto as tangentes são paralelas.Problema 15.(a) h′(x) = f ′(g(−x/2))g′(−x/2)(−1/2). As-sim, h′(2) = f ′(g(−1))g′(−1)(−1/2) == f ′(2)(6)(−1/2) = −1(6)(−1/2) = 3.(b) Como h(g(x)) = x, h′(g(x))g′(x) = 1.Como g(−1) = 2, h(g(−1)) = −1 = h(2). As-sim h′(2) = h′(g(−1)) = 1/g′(−1) = 1/6.Problema 16.O oe iente angular da reta tan-gente é ∆y

∆x=

3− 6

−1− 0= 3. Logo, f ′(−1) = 3.Note que f(−1) = 3 ou f−1(3) = −1. Logo

(f−1)′(3) =1

f ′(f−1(3))=

1

f ′(−1)=

1

3.2.3.3 Extras da p.17Extra 1. (a) w′ = 4

f ′g − fg′

g2. Logo w′(2) =

4(−1)(−5) − 3(2)

(−5)2= − 4

25.(b) m′(x) = 5e5xg(3x+2)+ e5xg′(3x+2)3.Logo m′(0) = 5g(2) + 3g′(2) = (5)− 5+3(2) =

−19.

Extra 2. (a) cos(x ex log x)(ex log x+xex log x+ex).(b) cos(sen(senx)) cos(senx) cos x.( ) 3arctan x = elog 3 arctan x. Logo a derivadaé 3arctan x log 3

1 + x2.(d) 2

√x+ 1

4√x√

x+√x.(e) cos(cos x senx)(cos2 x− sen2x).(f) Esta função vale sen(1−x2) se −1 < x <

1. Logo a derivada neste intervalo é −2x cos(1−x2). Fora deste intervalo (em x < −1 ou x >1) a função vale sen(x2 − 1), uja derivada é2x cos(x2 − 1). Nos pontos x = ±1 a funçãopossui um bi o, e não possui derivada.(g) Primeiro es revemos em forma de função:exp(exp(x4)). A derivada é:exp(exp(x4)) exp(x4)4x3 ou 4x3ee

x4

ex4 .(h) 2 cos(2x)

√x2 + 1

sen(2x)+

log(sen(2x))x√x2 + 1Extra 3. (a) Nos pontos onde y′(x) = 3x2+4x−

4 = 0, isto é, x = −2 ou x = 2/3.(b) Rees revendo a reta 2y + 8x − 5 = 0 omo y = −4x + 5/2, observamos que o o-e iente angular é −4. Assim queremos saberquando f ′(x) = 3x2 + 4x − 4 = −4, isto é,x = 0, x = −4/3.Extra 4.Para garantir ontinuidade em x = 1devemos ter: a(1) + b = 1

1 , ou a + b = 1. Paraque as derivadas laterais sejam iguais em x = 1devemos ter 2ax = − 1

x2em x = 1, ou a = −1

2 .Assim b = 1− a = 32 .Extra 5.Uma solução é: f ′(x) = g(x) + (x −

a)g′(x). Como f ′(a) = g(a) + (a − a)g′(a) =g(a). O problema desta solução é que não sabe-mos se g pode ser derivada.A solução orreta é: Note que f(a) = (a −a)g(a) = 0 e f(a+ h) = (a+ h− a)g(a+ h) =hg(a + h). Assim, f(a + h) − f(a) = hg(a +

h). Logo, f(a+ h)− f(a)

h= g(a + h). Assim,

f ′(a) = limh→0

f(a+ h)− f(a)

h= lim

h→0g(a + h).Este limite é igual a g(a) pois g é ontínua em a.Assim, f ′(a) = g(a).Extra 6. (a) y′ = 2x senx + x2 cos x. Logo,

y′(π) = −π2. Assim a reta tangente é y =−π2(x− π).(b) y′ =

1√x− 2

12√x. Logo, y′((e + 2)2) =

Page 73: Sumario´ - labMA/UFRJmcabral/livros/livro-calculo/... · 2019. 6. 12. · Sumario´ 1 Enunciados dos Exercícios 1.1 Exercícios de Limite. 1 1.1.1 Exercícios de Fixação. 1 1.1.2

2.3. DERIVADA 691

2e(e + 2). Assim a reta tangente é y − 1 =

1

2e(e + 2)(x− (e+ 2)2).Extra 7. (a) y′ = − 1

x2 . Para que duas retas se-jam paralelas, basta que possua o mesmo oe- iente angular. Como o oe iente angular de2x + 3y = 0 é −2

3 , queremos determinar xR talque y′ = − 1x2 = −2

3 . Logo x = ±√3√2.(b) y′ = 2e2x. A reta tangente no ponto

c, e2c é y− e2c = 2e2c(x− c). Para que passe em(x, y) = (5/2, 0) temos que resolver: 0− e2c =2e2c(5/2 − c). Vamos obter que c = 3.Extra 8. (a) Considere f(x) = x − log x. Noteque f(1) = 1−0 = 1 > 0 e que f ′(x) = 1− 1

x > 0para x > 1. Assim a função é res ente parax > 1 e é positiva em 1. Logo f(x) > 0 paratodo x ∈ R, ou x − log(x) > 0, o que impli aque x ≥ log x.(b) Considere g(x) = −2x13 − 6x5 − x+10.Como lim

x→+∞g(x) = −∞ e lim

x→−∞g(x) = +∞,existem pontos onde a função é positiva e nega-tiva. Pelo TVI existe pelo menos uma raiz. Como

g′(x) = −26x12 − 30x4 − 1 é sempre negativa(para todo x ∈ R), g é de res ente para todo Re portanto injetiva. Assim a raiz é úni a pois afunção é injetiva.Extra 9. (a) Por hipótese existem a, b ∈ R omf(a) = f(b) = 0. Pelo TVM (ou pelo Teoremade Rolle) existe c ∈ (a, b) tal que f ′(c) = 0. Logof ′ possui uma raiz real.(b) Pelo TVM existe um c ∈ (2, 5) tal quef(5)− f(2)

5− 2= f ′(c). Logo, f(5)−f(2) = 3f ′(c).Como por hipótese f ′(x) ≤ 4, f(5)− f(2) ≤ 12.( ) Como f ′′(x) = g′′1−g′′2 = cos(2x+log(x4+

1)) − cos(2x + log(x4 + 1)) = 0 para todo x ∈R, on luímos que f ′(x) é onstante. Note quef ′(2) = g′1(2) − g′2(2) = −1 − (−1) = 0. Logof ′(x) = 0 para todo x. Assim f é onstante.Note que f(3) = g1(3) − g2(3) = 5 − 5 = 0.Logo f(x) = 0 para todo x. Con luímos queg1(x) = g2(x) para todo x ∈ R.Extra 10.Pela denição, omo f(0) = 0,

f ′(0) = limh→0

f(0 + h)− f(0)

h= lim

h→0

f(h)

h.Agora f(h) = 0 ou f(h) = |h|k, dependendo se

h ∈ Q ou não. Nos dois asos, |f(h)| ≤ hk.

Assim, usando a ontinuidade da função módulo,|f ′(0)| =

limh→0

f(h)

h

= limh→0

f(h)

h

≤ limh→0

hk

h

= limh→0

|h|k−1 = 0.Portanto, 0 ≤ |f ′(0)| ≤ 0, ou seja, |f ′(0)| = 0 eportanto f ′(0) = 0.Extra 11.Note a beleza na simetria da resposta:(fgh)′ = f ′gh+ fg′h+ fgh′.Extra 12. f ′(x) = − sen(x5+2x+π/2)(5x4+2).Logo f ′(0) = −2. Como g(f(0)) = 0 = g(0),g′(0) =

1

f ′(0)= −1

2Extra 13. (a) A derivada é 1√1 + x2

.(b) Como 2 senh(y) = ey − e−y,Extra 14. Seja g(x) = n√x. Então g(xn) = x,

g′(xn)nxn−1 = 1. Logo g′(xn) = 1/nx1−n. Co-lo ando y = xn, g′(y) = 1/ny(1−n)/n.2.3.4 Desaos da p.19Desao 1. (a) Esta função não é derivável nozero poisf(0 + h)− f(0)

h=

h sen(1/h)

h= sen(1/h).Quando h → 0 o limite não existe.(b) Como,

g(0 + h)− g(0)

h=

h2 sen(1/h)

h= h sen(1/h),pelo Teorema do Sanduí he o limite quando h →

0 é zero. Assim, g′(0) = 0.Desao 2.Pelo binmio de Newton:(x+ h)n =

n∑

i=0

n!

i!(n− i)!xihn−i =

= xn + nxn−1h+ · · ·+ hn.Assim,(x+ h)n − xn = nxn−1h+ · · ·+ hn.Aqui temos termos om h, h2, . . . , hn. Dividindopor h, somente o primeiro termo não terá h:

(x+h)n−xn

h = nxn−1 + (termos om h)+ hn−1.Se zermos h → 0, sobrará apenas o termonxn−1.

Page 74: Sumario´ - labMA/UFRJmcabral/livros/livro-calculo/... · 2019. 6. 12. · Sumario´ 1 Enunciados dos Exercícios 1.1 Exercícios de Limite. 1 1.1.1 Exercícios de Fixação. 1 1.1.2

70 CAPÍTULO 2. RESPOSTAS DOS EXERCÍCIOSDesao 3.O oe iente angular da reta tangenteao grá o de f no ponto x é f ′(x) = −x. O oe iente angular da reta tangente ao grá o deg no ponto x é g′(x). Queremos que g′(x) =−1/f ′(x), isto é, que g′(x) = 1/x. Logo g(x) =log x ou, de forma geral, g(x) = C + log x.Desao 4.Como a equação da reta tangente éy = f(a)+f ′(a)(x−a), para que a reta tangentey = f(b) + f ′(b)(x − b) seja igual igualamos os oe ientes angulares (f ′(a) = f ′(b), 4a3−4a =4b3 − 4b) e e lineares (f(a) − af ′(a) = f(b) −bf ′(b)).Coe ientes angulares iguais: É laro que 4a3−4a = 4b3 − 4b impli a que b3 − a3 = b− a. Su-pondo b − a 6= 0 ou seja, a 6= b dividimospor b− a e obtemos a2 + ab+ b2 = 1.Coe ientes lineares iguais: Note que f(a)−af ′(a) = −3a4 + 2a2 + 1. É laro que −3a4 +2a2+1 = −3b4+2b2+1 impli a que 3(b4−a4) =2(b2 − a2). Supondo que a + b 6= 0 (veremosque isto impli a na inexistên ia de solução), omob − a 6= 0, b2 − a2 6= 0. Assim dividindo amboslados por b2 − a2 obtemos que a2 + b2 = 2/3.Utilizando a equação a2+ab+b2 = 1 on luímosque ab = 1/3. Assim a = 1/(3b). Substituindoobtemos que a = b = 1/

√3 ou a = b = −1/

√3,o que não é permitido pois supomos que a 6= b.Assim a úni a possibilidade é que a + b = 0(a = −b). Substituindo em a2 + ab + b2 = 1obtemos que a2 = 1, ou seja, a = ±1. Assima solução é a = 1 e b = −1. Como f(1) = 1e f ′(1) = 1, y = 1 + (x − 1) = x, ou y = xé a reta tangente que passa simultaneamente em

(1, f(1)) e (−1, f(−1)).Pelo desenvolvimento, esta é a úni a soluçãodo problema.Desao 5.Considere f(x) = ax2+bx+c. Assim,f ′(x) = 2ax + b. A se ante possui oe ienteangular: ax21 + bx1 − ax22 − bx2

x1 − x2=

=a(x21 − x22) + b(x1 − x2)

x1 − x2= a(x1 + x2) + b.A reta tangente no ponto médio possui oe- iente angular f ′((x1 + x2)/2) = a(x1 + x2) + bDesao 6.Como f ′(θ) = 0 para todo θ ∈ R,pelo TVM f é onstante. Como f(0) = 1, f(θ) =

1 para todo θ ∈ R. Assim, cos θ + i sen θ

eiθ= 1 eobtemos o resultado.Desao 7.Note que onhe emos uma solução:

s = sen e c = cos. A questão aqui é a uni idade.

Desao 8. (a) Como g′(y) = 1 − ε cos y, to-mando ε0 = 1/2 (ou qualquer valor positivo es-tritamente menor que 1), g′(y) > 0 para todoy. Assim g será estritamente res ente em R eportanto uma função injetiva. Portanto g possuiinversa.(b) Pelo teorema da função inversa, omog(0) = 0, f ′(0) = 1/g′(0) = 1/(1 − ε).Desao 9.Deixo omo desao para o leitor.Desao 10. (a) Soma e subtraia f(a): f(a +h)− f(a) + f(a)− f(a− h). Dividindo por h epassando ao limite obteremos 2f ′(a). Dividindopor 2 obtemos o resultado.(b) Deixo omo desao.Desao 11.A derivada de p é: p′(x) = 3x2 +2ax+b. Para que a derivada seja sempre positiva,e portante p sempre res ente, devemos ter ∆ ≤4a2 − 12b < 0, isto é, a2 − 3b ≤ 0.Desao 12. (a) basta passar ao limite dos doislados. (b) Divida os dois lados por |x− y|. Façax → y. Note que a derivada será zero em todosos pontos. Portanto, f será onstante.2.4 Apli ação de Derivada2.4.1 Exer. de Fixação da p.20Exer í io 1.O limite é 5 por L'Hospital.Exer í io 2.Não podemos apli ar L'Hospital duasvezes, somente uma vez obtendo lim

x→1

2(x− 1)

ex=

2(1− 1)

e= 0.Exer í io 3. (a) f(2.1) ≈ f(2)+f ′(2)(2.1−2) =

5 + 4(0.1) = 5.4.(b) f(1.95) ≈ f(2) + f ′(2)(1.95 − 2) = 5 +4(−0.05) = 4.8.Exer í io 4. p(π) = f(π) = −1, p′(π) = f ′(π) =− sen(π) = 0, p′′(π) = f ′′(π) = − cos(π) = 1.Exer í io 5. (a)011.o t.2011

Page 75: Sumario´ - labMA/UFRJmcabral/livros/livro-calculo/... · 2019. 6. 12. · Sumario´ 1 Enunciados dos Exercícios 1.1 Exercícios de Limite. 1 1.1.1 Exercícios de Fixação. 1 1.1.2

2.4. APLICAÇO DE DERIVADA 71x

y

−3 −2 −1 1 2(b)x

y

−2Exer í io 6.x

y

−3 −1 1 3

Exer í io 7. (a) e (b) Ambas verdadeiras. ( )Falso. Todos os pontos em [1, 2] são de máximoe de mínimo simultaneamente pela denição.Exer í io 8. (a) Falso. I tem que ser um inter-valo fe hado omo I = [−6, 99]. (b) Falso. Item que ser limitado e fe hado. ( ) Falso. Afunção tem que ser ontínua. (d) Falso. Mesmodes ontínua pode ter máximo. (e) Falso. Consi-dere I = R e a função ontínua f(x) =1

x2 + 1.O máximo é em x = 0.Exer í io 9. (a) Como f é ontínua em um inter-valo fe hado e limitado, podemos apli ar o TVE(Teorema do Valor Extremo de Weierstrass), Te-orema 22 da p.114, que garante que existe a.(b) Devemos omparar o valor da função nosextremos do intervalo om o valor da função nospontos ríti os. Assim omparando f(1), f(10),

f(3), f(7), determinaremos o máximo. Ou seja,o máximo será um dos pontos: 1, 3, 7 ou 10.( ) Não ne essariamente. Note que NO po-demos apli ar o TVE pois o intervalo não é limi-tado. Um exemplo é tomar uma f que vai para−∞ quando x → −∞.

Exer í io 10. (a) a = −5. (b) b = 0. ( ) podeser em c = −1 ou c = 2. (d) d = 2.Exer í io 11. (a) maxx∈I

f(x) = 1/2, xmax = 2,minx∈I

f(x) = 1/3, xmin = 3.(b) maxx∈I

f(x) = +∞, não existe xmax,minx∈I

f(x) = 1, xmin = 1.( ) maxx∈I

f(x) = −4, xmax = −1/4,minx∈I

f(x) = −1, xmin = −1.(d) maxx∈I

f(x) = 1, xmax = 1,minx∈I

f(x) = 0, não existe xmin =.(e) maxx∈I

f(x) = 0, minx∈I

f(x) = −∞,não existem xmax nem xmin.Exer í io 12. (a) Verdadeiro, pois se é mínimo lo- al então a derivada é zero. (b) Verdadeiro, poisse é máximo no interior do intervalo, então é má-ximo lo al. ( ) Falso, pois está no extremo do in-tervalo. Pode ser zero mas não ne essariamente.(d) Falso. Um ponto om derivada zero podenão ser máximo nem mínimo, omo por exemplog(x) = (x − 3)3, que possui derivada nula emx = 3 mas não é máximo nem mínimo.Exer í io 13. (a) Verdadeiro. (b) Falso, pode sere pode não ser. Exemplo é f(x) = 3, onde TODOponto é de máximo lo al (e de mínimo lo al) em-bora f ′ = f ′′ = 0. ( ) Falso, nem todo máximolo al é máximo em um intervalo. O máximo podeo orrer no extremo do intervalo e a derivada nãopre isa ser zero neste ponto.Exer í io 14. (a) máximos lo ais: x = −2 e x =3. mínimos lo ais: x = 0.(b) Mínimo em x = 4, máximo em x = 3.( ) Mínimo em x = −3, máximo em x = 1.(d) Mínimo em x = 0, máximo em x = 3.(e) f ′′(−1.8) < 0. (f) f ′′(0) > 0. (g)f ′′(4) < 0. (h) x = −1 e x = 1.(i) mínimo lo al em x = 1. máximo lo al emx = 4. Olhe o sinal de g′ antes e depois destespontos.(j) onde g′′(x) = f ′(x) = 0? pontos de ine-xão de g: x = −2 e x = 3.Exer í io 15. (a) Como f ′(x) = 4x3 − 3x2 =x2(4x−3), os pontos ríti os são x = 0, x = 3/4.Note que o sinal da derivada é: f ′(x) < 0 parax < 3/4 e f ′(x) > 0 para x > 3/4. Assimx = 0 NO é extremos lo al. Somente x = 3/4é mínimo lo al.(b) Devemos omparar f(−1) = 2, f(2) = 8,f(3/4) = −27/256. Assim o máximo em I é em

Page 76: Sumario´ - labMA/UFRJmcabral/livros/livro-calculo/... · 2019. 6. 12. · Sumario´ 1 Enunciados dos Exercícios 1.1 Exercícios de Limite. 1 1.1.1 Exercícios de Fixação. 1 1.1.2

72 CAPÍTULO 2. RESPOSTAS DOS EXERCÍCIOSx = 2 e o mínimo em x = 3/4.( ) Aqui basta omparar f(−1) = 2 omf(0) = 0. Assim o máximo é em x = −1 e omínimo em x = 0.(d) No extremo do intervalo x → ±∞ a fun-ção f(x) → +∞. Assim ela não tem máximo. Omínimo é no ponto ríti o x = 3/4.(e) No extremo x → −∞ a função f(x) →+∞. No extremo x = 1, f(−1) = 2. Nenhumponto ríti o perten e ao intervalo. Assim ela nãotem máximo e o mínimo é em x = −1.2.4.2 Problemas da p.23Problema 1. (a) 4. (b) Tomando o log obtemosque se y = (ex + 3x)1/x, log y =

log(ex + 3x)

x.Apli ando L.H. log( lim

x→0+y) = 4. Logo lim

x→0+y =

e4. ( ) 2/5. (d) Note que (ax)′ = (log a)ax.Assim o limite é log a− log b.Problema 2. (a)√65 ≈√64+

1

2√64

(65−64) =

8+1

16. (b) log(e2−0.1) ≈ log(e2)+

1

e2(−0.1) =

2− 1

10e2. ( ) Re ordando, arctan′(x) = 1

x2 + 1.Assim, arctan′(1) = 1

2 . Assim arctan(1.2) ≈tan(1) + 1

2(1.2 − 1) =π

4+ 0.1.Problema 3. (a) Como f ′(x) = 3ax2 + 2bx+ c,os extremos lo ais vão o orrer (possivelmente) so-mente nos pontos onde f ′(x) = 0. Se a equaçãopossuir duas raízes reais distintas, o sinal de f ′passará de positivo para negativo ou vi e-versaem ada raiz: assim um ponto será de máximoe o outro de mínimo lo al. Se possuir uma raizdupla, omo a > 0, f ′(x) ≥ 0 para todo x ∈ R.Assim o ponto onde f ′ se anula não será de má-ximo nem mínimo. Finalmente se f ′ não possuirraiz real, omo a > 0, f ′(x) > 0 para todo x ∈ R.Assim a função será sempre res ente, sem extre-mos lo ais.(b) Se f não possui extremos lo ais então

f ′(x) ≥ 0 para todo x ∈ R. Assim f poderápossuir no máximo 1 raiz. Como é polinmio degrau impar, pelo TVI (porque?) possui no mínimouma raiz. Con luímos que f possui exatamente1 raiz.( ) Se f possui 2 extremos lo ais, temos queveri ar se o mínimo lo al é menor que zero ounão e se o máximo lo al é menor que zero ou não(faça uma gura). Se ambos forem menor que

zero ou ambos maiores que zero, f admite so-mente uma raiz real. Se o máximo lo al é maiorque zero e o mínimo lo al menor que zero, f ad-mite exatamente 3 raízes reais.(d) Determine ( aso existam) os dois pontos ríti os distintos x0 < x1 de f , isto é, pontos taisque f ′(x0) = f ′(x1) = 0. Caso não existam ouexista somente um, a função possui somente umaraiz real.Como a > 0 ne essariamente x0 é máximoe x1 é minimo (basta olhar sinal de f ′, que vempositivo até x0, a negativa em (x0, x1) e voltaa ser positivo em x1. Se f(x0) > 0 > f(x1)possui 3 raízes reais, aso ontrário somente umaraiz real.Problema 4. (a) f ′(x) = 3x2 − 6x+ 3 = 3(x−1)2, uja úni a raiz é x = 1. Assim f ′(x) ≥ 0para todo x ∈ R. Logo esta função é sempre res ente. Como f ′′(x) = 6x − 6, ela tro ade on avidade em x = 1. Quando x → +∞,f(x) → +∞ e quando x → −∞, f(x) → −∞.Não possui assíntota verti al nem horizontal.Embora f ′(1) = 0, omo f ′ > 0 perto dex = 1, este ponto não é de máximo nem mínimo.

x

y

1

(b) Como g′(x) = x(x2 − 4), a derivada seanula em 0,±2. Analisando sinal de g′ (quadro desinais) on luímos que g de res e até −2, res eem (−2, 0), de res e em (0, 2), e res e de 2 emdiante. Com isso vemos que os pontos x = ±2são de mínimo lo al e x = 0 é de máximo lo- al. Como g′′(x) = 3x2−4, a on avidade mudaem ± 2√3, sendo para ima antes de − 2√

3e de-pois de 2√

3e para baixo em (− 2√

3, 2√

3). Assimum esboço para grá o (não é úni o pois pode-sesomar onstante a g) é:

Page 77: Sumario´ - labMA/UFRJmcabral/livros/livro-calculo/... · 2019. 6. 12. · Sumario´ 1 Enunciados dos Exercícios 1.1 Exercícios de Limite. 1 1.1.1 Exercícios de Fixação. 1 1.1.2

2.4. APLICAÇO DE DERIVADA 73

x

y

−2 22√3

− 2√3Problema 5.O esboço deverá ter uma f(x) = 2para x ∈ [0, 1] pois f ′(x) = 0 neste intervalo.Ela deverá de res er para x > 1 om on avidadepara baixo pois f ′′ < 0. Entre −2 e 0 ela deverá res er pois f ′ > 0 neste intervalo. No entantoa on avidade deve ser para ima até −1 e parabaixo depois. Até o −2 ela deve deve de res er om on avidade para ima e um mínimo lo al em

x = −2 pois a derivada se anula em −2. Assimobtemos:x

y

−2 −1 1 2

Problema 6. (a) Possui duas assintotas horizon-tais: y = 2 e y = −1. Possui assintota verti alem x = 0. Possui um máximo lo al em x = −1.x

y

y = 2

y = −1−1

1

(b) Possui duas assintotas horizontais: y = 0e y = −1. Possui assintota verti al em x = 2.Possui um máximo lo al em x = 0.

x

y

x = 2

y = −1−2

1

2

Problema 7. (a) Intersepta os eixos em (0, 0) e(1, 0). Assintotas verti ais em x = 2 e x = −1.Assintota horizontal: y = 2. Sinal de f ′ é igualao sinal de 1 − 2x: a função res e até x = 1/2e de res e depois. Em x = 1/2 a função tem ummáximo lo al.O sinal de g′′ é igual ao sinal de (x− 2)(x+1) (note que x2 − x + 1 > 0 pois as raízes são omplexas): on avidade para ima até x = −1e depois de x = 2. Con avidade para baixo em(−1, 2).

x

y

y = 2

−1 212(b) Intersepta os eixos em (0, 1). Assintotasverti ais em x = ±1. Assintota horizontal: y =

−1. Sinal de g′ é igual ao sinal de x: a funçãode res e até x = 0 e res e depois. Em x = 0 afunção tem um mínimo lo al.O sinal de g′′ é igual ao sinal de 1−x2: on a-vidade para baixo até x = −1 e depois de x = 1.Con avidade para ima em (−1, 1).x

y

y = 1 −1 1

( ) Intersepta os eixos em (0, 0). Assintota

Page 78: Sumario´ - labMA/UFRJmcabral/livros/livro-calculo/... · 2019. 6. 12. · Sumario´ 1 Enunciados dos Exercícios 1.1 Exercícios de Limite. 1 1.1.1 Exercícios de Fixação. 1 1.1.2

74 CAPÍTULO 2. RESPOSTAS DOS EXERCÍCIOSverti al em x = 1. Assintota horizontal: y = 0.Sinal de h′: a função de res e até x = −1, res eem (−1, 1), de res e depois de x = 1. Em x =−1 a função tem um mínimo lo al.O sinal de h′′ é igual ao sinal de x+2: on a-vidade para baixo até x = −2, Con avidade para ima depois.

x

y

−2 1Problema 8. (a) Não intersepta os eixos (nun avale zero e não está denida em x = 0). Assintotaverti al em x = 0. Assintota horizontal: y = 0.Sinal de f ′ é igual ao sinal de x − 1 pois exe x2 são sempre positivas: a função de res e atéx = 1 e res e depois de x = 1. Em x = 1 afunção tem um mínimo lo al.O sinal de f ′′ é igual ao sinal de x3 pois opolinmio x2 − 2x+ 2 possui raízes omplexas e omo oe iente de x2 é positivo, x2−2x+2 > 0para todo x ∈ R. Assim f ′′ é negativa para x < 0e positiva para x > 0. Portanto on avidade parabaixo para x < 0, Con avidade para ima parax > 0.

x

y

1(b) Note que a função está denida somenteonde 1− x2 > 0, isto é, para x ∈ (−1, 1). Inter-septa os eixos em (0, 1) e quando log(1− x2) =−1, isto é, quando 1 − x2 = e−1. Portantoquando x2 = 1− e−1, isto é, x = ±

√1− e−1 ≈

±0.79 (pelo software Maxima). Logo inter eptao eixo x em (±0.79, 0). Assintota verti al emx = ±1 (onde temos log 0 = −∞!). Assintota

horizontal não existe (função nem esta denidapara x > 1 nem x < −1).Sinal de f ′ é igual a de −2x para x ∈ (−1, 1)pois x2 − 1 < 0 neste intervalo. Assim a função res e para x < 0 e de res e para x > 0. Emx = 0 a função tem um máximo lo al.O sinal de f ′′. Note que o numerador 2x2+2é sempre positivo e omo o denominador é iguala (x2 − 1)2, que é sempre positivo, por ter sinalde menos na frente será sempre negativa. Assimf ′′ < 0 e a on avidade é sempre para baixo.

x

y

x = 1x = −1

√1− e−1−

√1− e−1

1

( ) Intersepta os eixos em (0, 1 + e−2). Nãopossui Assintota verti al. Assintota horizontal:y = 1.Sinal de f ′ é igual ao sinal de 3 − 2x poisexponen ial de qualquer oisa é sempre positiva.Portanto a função res e até x = 3/2 e de res edepois. Em x = 3/2 a função tem um máximolo al.O sinal de f ′′ é igual ao sinal de 4x2 − 12x+7. As raízes são: 3/2 ±

√2/2. A on avidadepara baixo em 3/2 −

√2/2, 3/2 +

√2/2), ou,aproximadamente, em (0.79, 2.20). Con avidadepara ima fora deste intervalo.

x

y

y = 1

0.79 2.2032(d) Intersepta os eixos em (0, 0). Não tem

Page 79: Sumario´ - labMA/UFRJmcabral/livros/livro-calculo/... · 2019. 6. 12. · Sumario´ 1 Enunciados dos Exercícios 1.1 Exercícios de Limite. 1 1.1.1 Exercícios de Fixação. 1 1.1.2

2.4. APLICAÇO DE DERIVADA 75Assintota verti al. Assintota horizontal: y = 0quando x → −∞.Sinal de f ′ é igual ao sinal de x+3, pois o-lo ando em evidên ia x2, que é sempre positivo,obtemos isto. Note que a derivada será zero emx = −3 e em x = 0. Note que em zero a derivadaNO tro a de sinal, ontinuando positiva. Assima função de res e até x = −3 e res e depois.Em x = −3 a função tem um mínimo lo al. Oponto x = 0 possui derivada zero (é ponto rí-ti o) mas não é máximo nem mínimo lo al pois afunção res e em torno de x = 0 (f ′(x) > 0 parax próximo mas diferente de zero).O sinal de f ′′ é igual ao sinal de x(x2+6x+6).As raízes são 0,−3±

√3.

−3−√3 ≈ −4.7 e −3 +

√3 ≈ −1.26.Fazendo quadro de sinais vamos obter que: on avidade para baixo até x = −3 −

√3 ≈

−4.7 e também no intervalo (−3 +√3, 0) ≈

(−1.26, 0). A Con avidade será para ima em(−3 −

√3, −3 +

√3) ≈ (−4.7, −1.26) e tam-bém para x > 0.

x

y

−3−4.7 −1.26Problema 9. (a) O ponto ríti o é a solução desen3(x) = cos3(x), e portanto se tan3(x) =1, ou seja, quando tan x = 1, o que o orre sex = π/4. Quando x → 0+ ou x → π/2−,f(x) → +∞. Assim o mínimo é em x = π/4 om f(π/4) = 2

√2 e NO existe máximo em

I. Portanto maxx∈I

f(x) = +∞, não existe xmax,minx∈I

f(x) = 2√2, xmin = π

4.(b) O úni o ponto ríti o é em x = 2 (f ′(2) =

0). Quando x → 0+ ou x → +∞, f(x) → +∞.Assim em I = (0, +∞) o mínimo é em x = 2 e omáximo não existe. Portanto maxx∈I

f(x) = +∞,não existe xmax, minx∈I

f(x) =5

2, xmin = 2.Em I = (0, 3], omo 2 ∈ I, o mínimo é em

x = 2 e o máximo não existe pois próximo de 0f(x) → +∞. Portanto max

x∈If(x) = +∞, não

existe xmax, minx∈I

f(x) =5

2, xmin = 2.Em I = [3, 4] não tem ponto ríti o. Logo omáximo e o mínimo estão nos extremos: f(3) =

3 + 1/3 e f(4) = 4 + 1/4. Logo o mínimo éem x = 3 e o máximo em x = 4. Portantomaxx∈I

f(x) = 4 + 1/3, xmax = 4, minx∈I

f(x) =

3 + 1/3, xmin = 3.( ) Note que o termo da derivada x2 − x+2possui raízes omplexas. Como o termo de maiorgrau é x2, x−x+ 2 > 0 para todo x ∈ R. Logoa úni a raiz é x = 0, om sinal de f ′ igual aosinal de x. Como f de res e até x = 0 e res edepois, x = 0 é mínimo lo al.Assim em [−1, 1] omparamos f(−1) = 19,f(1) = 11, f(0) = 0. Portanto max

x∈If(x) = 19,

xmax = −1, minx∈I

f(x) = 0, xmin = 0.Em [1, 2], não tem ponto ríti o, basta om-parar f(2) = 64 e f(1) = 11. Portantomaxx∈I

f(x) =

64, xmax = 2, minx∈I

f(x) = 11, xmin = 1.(d) Note que f ′ é sempre positiva. Logo f ésempre res ente. Note que limx→−1−

f(x) = +∞Em I = (−1, 1] temos que limx→−1+

f(x) =

−∞. Assim não possui mínimo. O máximo seráem x = 1 om f(1) =1

2. Portanto max

x∈If(x) =

1

2, xmax = 1, min

x∈If(x) = −∞, não existe xmin =.Em I = [0, 1], omo f(0) = 0, max

x∈If(x) =

1

2, xmax = 1, min

x∈If(x) = 0, xmin = 0.Problema 10.Determine o máximo e o mínimode f(x) =

x

x4 + 3em R. Con lua que K ∈

[−1/4, 1/4].Problema 11.MODELAGEM: Se x, y são os nú-meros, y − x = 100, p = xy mínimo. Comoy = x + 100, p(x) = (x + 100)x. Queremosminimizar p(x) para x ∈ R.RESOLUÇO: Como p(x) → +∞ quandox → ±∞, o mínimo é no ponto de derivada zero.Como p′(x) = 2x + 100, x = −50 é o ponto dederivada zero, om y = −50 + 100 = 50. Logoos números são 50 e −50.Problema 12.MODELAGEM: Suponha que a do-bra tenha omprimento x. A alha terá a formade um retângulo om lado x e L− 2x (o que so-brou para base. Como o volume é propor ionala área deste retângulo, queremos o máximo def(x) = x(L− 2x) para x ∈ [0, L/2].

Page 80: Sumario´ - labMA/UFRJmcabral/livros/livro-calculo/... · 2019. 6. 12. · Sumario´ 1 Enunciados dos Exercícios 1.1 Exercícios de Limite. 1 1.1.1 Exercícios de Fixação. 1 1.1.2

76 CAPÍTULO 2. RESPOSTAS DOS EXERCÍCIOSRESOLUÇO: Como é equação do segundograu om on avidade para baixo, o máximo é noponto de derivada zero. Como f ′(x) = L− 2x−2x = L − 4x, x0 = L/4. Assim a alha deveráter a forma de um retângulo om dimensões L/4e L/2.Problema 13.MODELAGEM: Seja r o raio do ír ulo e θ o ângulo do setor ir ular. Quere-mos maximizar a área a = θr2/2. O perímetrodeste setor é 2r mais θr. Assim, 40 = 2r + θr.Logo, θ = 40/r − 2. Logo queremos o má-ximo de a(r) = 20r − r2. Note que θ varia en-tre 0 e 2π. Como 40 = 2r + θr, para θ = 0,r = 20 e para θ = 2π, r = 20/(1 + π). Assimr ∈ [20/(1 + π), 20].RESOLUÇO: Trata-se de uma equação dosegundo grau. a′(r) = 20 − 2r. Logo a derivadaé zero em r0 = 10. Como 20/(1 + π) < 20/4 =5 < 10 (π > 3), o máximo é em r0 = 10.Problema 14.MODELAGEM: Vamos modelar in-troduzindo θ para o ângulo e B = K − h para adiferença entre a distân ia da tela ao hão e a al-tura dos olhos do espe tador. Note que se h → 0ou h → +∞ o ângulo θ → 0.Por trigonometria simples, tanϕ =

B

de tan(θ+

ϕ) =L+B

d. Assim, ϕ = arctan(B/d) e θ +

ϕ = arctan((L +B)/d). Logo, o ângulo θ(d) =arctan((L+B)/d) − arctan(B/d).Queremos maximizar θ(d) para d ∈ (0,+∞).RESOLUÇO: Derivando obtemos

θ′(d) =L(BL+B2 − d2)

(B2 + d2)((L +B)2 + d2).Queremos determinar d0 tal que θ′(d0) = 0. Comoo denominador é sempre positivo e L > 0, a úni araiz da derivada é d0 tal que BL+B2 − d20 = 0,isto é (solução positiva) d0 = √

B2 +BL.Problema 15.MODELAGEM: Sejam x e y asdimensões do artaz. Sua área A = xy. A áreaimpressa será igual a (x− 2M)(y − 2N). Elimi-nando y = A/x obtemos que queremos maximi-zar a área impressa f(x) = (x−2M)(A/x−2N) om x ∈ [2M,A/(2N)].RESOLUÇO: Di a: Resolva o problema omA = 50,M = 2, N = 4. Vou dar a soluçãogeral. Como f ′(x) = A/x−2N−(x−2M)A/x2,os zeros da derivada são ±

AM/N . Queremossomente a solução positiva x0 =√AM/N . Noteque nos extremos a área impressa f é zero. Assim

o máximo é de fato em x0 se nos erti armos quex0 ∈ [2M,A/(2N)].Vamos provar que de fato x0 ∈ [2M,A/(2N)].Para que o problema faça sentido a área A deveser maior que a área das margens (2M)(2N) =4MN . Assim, 4MN < A. Logo, 4M2 < AM/N ,e portanto 2M <

AM/N = x0. Por outrolado, AM/N < A2/(4N2). Logo, √AM/N =x0 < A/(2N).Problema 16.MODELAGEM ( omum aos doisitens): Seja h a altura e r o raio das semiesferas.O volume é V = 4/3πr3 + πr2h e a área desuperfí ie é A = 4πr2 + 2πrh.(a) MODELAGEM: Se xarmos a área emA, tiramos que πrh = (A − 4πr2)/2. Assim,V (r) = 4/3πr3 + r(A − 4πr2)/2. Queremosmaximar V (r) em [0,

A/(4π)] ( hegamos nestevalor tomando h = 0 na relação A = 4πr2+2πrh). RESOLUÇO: Vamos al ular o ponto rí-ti o. Como V ′(r) =A− 4πr2

2, V ′(r0) = 0 se

A = 4πr20. Assim a derivada é zero no extremodo intervalo r0 =√

A/(4π). Note que V (0) = 0e V ′(0) = A/2 > 0. Alem disso V ′(x) > 0 paratodo x ∈ [[0,√

A/(4π)]. Logo V res e neste in-tervalo e portanto r =√

A/(4π) é o ponto ondeV (r) assume o máximo.(b) MODELAGEM: O usto de fabri ação épropor ional a área de superfí ie A. Como o vo-lume V é xo, tiramos que πrh = (V−4/3πr3)/r.Assim,A(r) = 4πr2 + 2(V − 4/3πr3)/r =

6V + 4πr3

3r.Queremos minimizar A(r) para r ∈ (0, 3

3V

4π)( hegamos neste valor tomando h = 0 na relação

V = 4/3πr3 + πr2h).RESOLUÇO: Note que A(r) → +∞ quandor → 0+ ou r → +∞. Assim o mínimo o orreráem um ponto ríti o. Como A′(r) =

8πr3 − 6V

3r2.Assim a derivada se anula somente em r0 =

3

3V

4π.Pode-se onrmar que o mínimo é em r = r0 poiso sinal da derivada é sempre negativa.Problema 17. (a) MODELAGEM: Queremos mi-nimizar o quadrado da distân ia g(x) = (x−2)2+

(f(x) − 2)2 = (x − 2)2 + |6x − 2x2|. Note queo domínio de f é onde 6x − 2x2 > 0, isto é em[0, 3].

Page 81: Sumario´ - labMA/UFRJmcabral/livros/livro-calculo/... · 2019. 6. 12. · Sumario´ 1 Enunciados dos Exercícios 1.1 Exercícios de Limite. 1 1.1.1 Exercícios de Fixação. 1 1.1.2

2.4. APLICAÇO DE DERIVADA 77RESOLUÇO: Apli ando a denição de mó-dulo observamos que |6x − 2x2| = 6x − 2x2se x ∈ [0, 3]. Assim g(x) = −x2 + 2x + 4 sex ∈ [0, 3]. Em [0, 3], g′(x) = −2x+ 2 e g′(1) =0. Temos que omparar g(0) = 4, g(1) = 5 eg(3) = 1. Observamos que o mínimo é em x = 3 om g(3) = 1 e o máximo é em x = 1 omg(1) = 5.(b) MODELAGEM: A distân ia verti al f(x)é igual a diferença entre os y's. Assim, queremoso mínimo de f(x) = x2 +

1

x2para x ∈ R.RESOLUÇO: Note que f(x) → +∞ para

x → ±∞. Logo o mínimo será no ponto dederivada zero. Como f ′(x) = 2x− 2

x3, os pontos ríti os são 1 e −1. Como f(1) = f(−1) = 2, omínimo é em x = 1 ou x = −1.Problema 18.MODELAGEM: Sejam x, y os la-dos do retângulo. O perímetro P = 2x + 2y.Note que ligando-se o entro do ír ulo a umvérti e do retângulo obtemos um triângulo retân-gulo om lados x/2, y/2, R. Assim, por Pitágo-ras, x2 + y2 = 4R2. Logo, y =

√4R2 − x2 para

x ∈ [0, 2R]. Assim queremos o máximo e mínimode P (x) = 2x+ 2√4R2 − x2 para x ∈ [0, 2R].RESOLUÇO: Como P ′(x) = 2− 2x√

4R2 − x2.Note que P ′(x) = 0 se, e somente se, 2√4R2 − x2 =

2x. A raiz positiva será x0 = R√2. Como

x20+y20 = 4R2, y0 = R√2. Comparando P (0) =

4R = P (2R) e P (R√2) = R3

√2. Assim, omo

4 < 3√2, o maior perímetro será R3

√2 para oquadrado de lado R

√2. O menor será para oretângulo degenerado de lados 0 e 2R, om perí-metro 4R.Problema 19.MODELAGEM: Vamos xar x omosendo o ponto do eixo x que é um dos vérti- es do retângulo. Automati amente os outrosvérti es vão ser (x, y(x)), (−x, y(x)) e (−x, 0).Assim a área A = (2x)y(x) = 2(27x − x3).Note que omo as raízes da parábola são ±

√27,

x ∈ [−√27,

√27] e queremos maximizar A(x) =

2(27x − x3).RESOLUÇO: Como A′(x) = 2(27 − 3x2),os pontos ríti os são x = ±3, que perten emao intervalo. Note que A(±√27) = 0. Assim omáximo será em x = 3 onde A(3) = 108. Noteque y(x) = 18. Assim as dimensões são 2x = 6por y = 18Problema 20. (a) MODELAGEM: Seja r o raioe h a altura do one ins rito na esfera. O volume

do one é V =1

3πr2h. Note que omo 1

3π éum onstante, maximizar a função f = r2h é umproblema equivalente. Como é função de duasvariáveis, devemos eliminar uma delas.Ligando-se o entro da esfera até um dos pon-tos do ír ulo da base do one observamos o tri-ângulo retângulo om hipotenusa R e atetos r e

h−R. Logo, por Pitágoras, (h−R)2+ r2 = R2,Assim, r2 = 2hR − h2. Logo f(h) = h(2hR −h2). Note que h ∈ [0, 2R]. Assim queremos omáximo de f(h) para h ∈ [0, 2R].RESOLUÇO: Note que f(0) = f(2R) = 0.Como f ′(h) = 4hR − 3h2 = h(4R − 3h), ospontos ríti os são h = 0 e h = 4R/3. Como oponto zero não é de máximo, o máximo é quandoh = 4R/3.(b) MODELAGEM: Seja r o raio e h a alturado ilindro ins rito no one. O volume do ilindroé V = πr2h. Como é função de duas variáveis,devemos eliminar uma delas. Note que ortandoo one temos uma semelhança de triângulos: aaltura H do one está para R assim omo H −hestá para r. Assim, H

R=

H − h

r. Logo, r =

R(H − h)

H. Logo queremos maximizar V (h) =

πh

(

R(H − h)

H

)2. Note que h ∈ [0, H]. Assimqueremos o máximo de V (h) para h ∈ [0, H].RESOLUÇO: Note que V (0) = V (H) =

0. Como V ′(h) =πR2(H − 3h)(H − h)

H2(vaiobter-se equação do segundo grau om raízes He H/3). Como V (H) = 0, o máximo é para h =

H/3 (não pre isa al ular V (H/3) =4πHR2

27,que obtive om o Maxima).2.4.3 Extras da p.27Extra 1. (a) 4. (b) +∞ ( ) 5/4. (d) Note que

x√x = x1/x = (elog x)1/x = elog x/x. Quando

x → +∞, log x/x → 0. logo o limite é e0 = 1.(e) 1/12 (LH mais de uma vez).Extra 2. (a) tan(0.05) ≈ tan(0)+tan′(0)(0.05−0) = 0.05. (b) 3

√28 ≈ 3

√27 + 1/(3

3√272)(28 −

27) = 3 + 1/(27).Extra 3. (a) Veja na apa do livro ()o i loda derivada do seno. No entanto somente é dife-rente de zero cos 0 = 1 e − cos 0 = −1. Assim,somente os termos om expoente impar são não-nulos, alternando sinal entre −1 e 1. Portanto,

Page 82: Sumario´ - labMA/UFRJmcabral/livros/livro-calculo/... · 2019. 6. 12. · Sumario´ 1 Enunciados dos Exercícios 1.1 Exercícios de Limite. 1 1.1.1 Exercícios de Fixação. 1 1.1.2

78 CAPÍTULO 2. RESPOSTAS DOS EXERCÍCIOS olo ando os fatoriais, a série é:senx ≈ x− x3

3!+

x5

5!− x7

7!.(b) Veja (novamente) na apa do livro ()o i lo da derivada do log. Colo ando x = a = 1obtemos fatorial om o sinal alternando. Isto vai an elar o fatorial do denominador. Portanto, asérie é:

log(1 + x) ≈ x− x2

2+

x3

3− x4

4.( ) Veja (novamente) na apa do livro ()o i lo da derivada do sen e osseno hiperbóli o.Como senh 0 = 0 e cosh 0 = 1, a série de Taylorterá apenas os termos impares, sempre om sinalpositivo:

senh(x) = x+x3

3!+

x5

5!+

x7

7!+

x9

9!+ · · ·Extra 4. Seja f(x) = 3

√x. Como f ′(x) =

1

3x2/3,

f ′(a3) =1

3a2. Fazendo a série de Taylor perto de

x = a3, f(a3 + b) ≈ f(a3) + f ′(a3)b = a+b

3a2.Extra 5. (a) g′(x) = (x − 3)(x − 2). Assim afunção res e antes de x = 2, de res e em (2, 3)e res e depois de x = 3. Além disso, omo

g′′(x) = 2x − 5, a on avidade é para baixo atéx = 5/2 e para ima depois.

x

yg(x)

2 352(b) f ′(x) = 4x(x − 1)(x + 1) e f ′′(x) =

12x2 − 4. Note que f(x) ≥ 0 e é zero somenteem x = ±1. Pela derivada, a função de res e atéx = −1, res e em (−1, 0), de res e em (0, 1) e

res e de 1 em diante. Os pontos x = ±1 são demínimo lo al. O ponto x = 0 é de máximo lo al.Quando x = 0, y = 1.Pela f ′′, a on avidade é para ima até−1/√3,para baixo em (−1/

√3, 1/

√3) e para ima nova-mente para x > 1/

√3.

x

y f(x)

−1 11√3

− 1√3Extra 6.Extraímos do grá o informação sobre res imento e de res imento. Basta olhar ondea função do grá o é positiva e onde é negativa.Assim, f(x) de res e ate x = −2, em (0, 2) edepois de x = 4. Ela res e em (−2, 0), (2, 4).Agora se observarmos o grá o podemos ob-ter a informação sobre a derivada da função re-presentada, isto é, sobre a derivada segunda de

f . Assim, f ′′(−1) = f ′′(1) = f ′′(3) = 0. Obser-vando o sinal de f ′′ on luímos que a on avidadede f(x) é para ima até x = −1, em (1, 3). A on avidade é para baixo em (−1, 1) e depois dox = 3.

x

y

f(x)

−2 −1 1 2 3 4

Extra 7. (a) Ponto de máximo lo al em x = 0 emínimo lo al em x = 2.x

y

1 2

3

(b) Assintota horizontal y = −3 e máximo

Page 83: Sumario´ - labMA/UFRJmcabral/livros/livro-calculo/... · 2019. 6. 12. · Sumario´ 1 Enunciados dos Exercícios 1.1 Exercícios de Limite. 1 1.1.1 Exercícios de Fixação. 1 1.1.2

2.4. APLICAÇO DE DERIVADA 79lo al em x = 2.x

y

2 4

y = −3

Extra 8. (a) Assíntotas verti ais em x = 1 e x =−3, horizontal em y = 1. Intersepta o eixo xem x = −1 e x = −2. O sinal da derivada serádado pelo polinmio −x2−10x−13, ujas raízessão: −5± 2

√3, que são aproximadamente −8.4e −1.5.

x

y

f(x) =(x+ 1)(x+ 2)

(x+ 3)(x− 1)

y = 1

−1

(b) Note que as assíntotas verti ais são x = 3e x = 1. A horizontal é y = 0. O sinal daderivada é igual ao de x2 − 3: a função de res eem (−√3,√3) e res e fora. Tem máximo lo alem −

√3 e mínimo lo al em √

3.Ignoramos a derivada segunda pois ela vai darg′′(x) = 2

x3 − 9x+ 12

(3− x)3(x− 1)3. Note que o polin-mio do denominador é do ter eiro grau, e portantonão sabemos omo al ular a raiz ( om o Maximaobtemos que a úni a raiz real é −32/3 − 31/3.).

x

y

f(x) =x

(x− 3)(1 − x)+ 3

Extra 9. (a) A função res e para x > 0 e de- res e para x < 0 pois o sinal de f ′ é deter-minado por x. A derivada se anula em x = 0,que é ponto de mínimo lo al. A assintota hori-zontal é y = 1, verti al não tem. Intersepta oseixos somente em (0, 0). Con avidade p/ imaem (−1, 1) para baixo fora.x

y

f(x) =x2

x2 + 3

y = 1

−1 1

(b) A função res e para x > 0 e de res epara x < 0 pois o sinal de f ′ é determinado porx. A derivada se anula em x = 0, que é pontode mínimo lo al. A assintota horizontal é y = 1,verti al x = ±2. A on avidade é determinadapelo sinal de 4− x2 (pois o numerador é semprepositivo): on avidade p/ ima em (−2, 2) parabaixo fora.

Page 84: Sumario´ - labMA/UFRJmcabral/livros/livro-calculo/... · 2019. 6. 12. · Sumario´ 1 Enunciados dos Exercícios 1.1 Exercícios de Limite. 1 1.1.1 Exercícios de Fixação. 1 1.1.2

80 CAPÍTULO 2. RESPOSTAS DOS EXERCÍCIOSx

y

f(x) =x2

4− x2+ 2

x = 2 x = −2

2y = 1

Extra 10. (a) Note que a função está denidasomente para x > 0. Note que limx→0+

x log x = 0por L'Hospital. Intersepta os eixos em (0, 0) e(1, 0).Não possui assíntota verti al nem horizontal.Sinal de f ′. Note que log x = −1 quandox = e−1 ≈ 0.36. A função de res e até x = e−1,e res e depois. Em x = e−1 a função tem ummínimo lo al.O sinal de f ′′ é sempre positivo para x >0, o domínio da função. Assim a on avidade ésempre para ima.

x

y

f(x) = x log x

1e(b) Intersepta os eixos em (0, 0). Não temAssintota verti al. Assintota horizontal: y = 0.Sinal de f ′ é igual ao sinal de 1− 2x2, ujasraízes são ±1/

√2 ≈ ±0.707. Assim a função res e em (− 1√

2, 1√

2) ≈ (−0.7, 0.7). Ela de res efora deste intervalo. Em x = − 1√

2a função temum mínimo lo al e em x = 1√

2a função tem ummáximo lo al.O sinal de f ′′ é igual ao sinal de 2x(2x2− 3).Assim a on avidade para ima em (−

3/2, 0) ≈(−1.22, 0) e para x >

3/2) ≈ 1.22, Con avi-dade para baixo em (0,√

3/2) ≈ (0, 1.22) e parax < −

3/2 ≈ −1.22.

x

y

f(x) = xe1−x2

1√2

−1√2( ) Intersepta os eixos em (0, 0). Não temAssintota verti al. Assintota horizontal: y = 0.Sinal de f ′ é igual ao sinal de x(x+2). Assima função de res e em (−2, 0) e res e fora desteintervalo. Em x = −2 a função tem um máximolo al e em x = 0 tem um mínimo lo al.O sinal de f ′′ é igual ao sinal de x2 + 4x +

2. Assim a on avidade para baixo em (−2 −√2, −2 +

√2) ≈ (−3.41, −0.58) Con avidadepara ima fora deste intervalo.

x

y

f(x) = x2ex−2Extra 11. (a) Pontos ríti os são x = 0, x =

±2. Temos que omparar f(0) = 0 om f(2) =f(−2) = 16. Além disso, quando x → ±∞,f(x) → −∞.Assim, em I = R, max

x∈Rf(x) = 16, xmax = 2ou −2, min

x∈Rf(x) = −∞, não existe xmin =.Em I = [−1, 1], devemos omparar f(−1) =

f(1) = 7, f(0) = 0 (úni o ponto ríti o no in-tervalo. Logo maxx∈I

f(x) = 7, xmax = 1 ou −1,minx∈I

f(x) = 0, xmin = 0.(b) Úni o ponto ríti o é x = 0.Em I = [1, 2], que não ontém o ponto rí-ti o, devemos omparar f(1) =1

2e f(2) =

1

5.Assim max

x∈If(x) = 1/2, xmax = 1, min

x∈If(x) =

1/5, xmin = 2.Em I = [−1,+∞), omo f(x) → 0 quandox → +∞, f(0) = 1 e f(−1) = 1/2, max

x∈If(x) =

1, xmax = 0, minx∈I

f(x) = 0, não existe xmin.( ) Note que f ′(x) = − sen(x) cos(cos(x)).Logo os pontos ríti os vão ser onde senx = 0 ou

Page 85: Sumario´ - labMA/UFRJmcabral/livros/livro-calculo/... · 2019. 6. 12. · Sumario´ 1 Enunciados dos Exercícios 1.1 Exercícios de Limite. 1 1.1.1 Exercícios de Fixação. 1 1.1.2

2.4. APLICAÇO DE DERIVADA 81onde cos(y) = 0. Assim a derivada será nula emx = 0, π pela equação senx = 0. Para a outra,y = π/2 ≈ 1.57. Como y = cos x, esta equaçãoé impossível pois nun a cos x = 1.57 > 1. Omesmo o orrerá om outros valores.Assim devemos omparar f(0) = sen 1, f(π) =sen−1 = − sen 1, f(2π) = sen 1.Assim max

x∈If(x) = sen 1, xmax = 0,

minx∈I

f(x) = − sen 1, xmin = π.Extra 12.MODELAGEM Se o triângulo equilá-tero tem lado x, o retângulo possuirá lados x e y.Como a quantidade de luz é propor ional a áreada janela, queremos maximizar a área da janelaa = xy + x2

√3/4 (área retângulo mais semi- ír ulo). Esta é uma função de duas variáveis.Utilizando a restrição que o perímetro da janela

12 = 3x+2y, obtemos que y = 6−3/2x. Assima(x) = x(6 − 3/2x) + x2

√3/4. Olhando parao perímetro 12 = 3x + 2y, vemos que os asosextremos são x = 4 (y = 0) e x = 0. Assimqueremos o máximo de a(x) para x ∈ [0, 4].RESOLUÇO: Como a′(x) = 6−(3−

√3/2)x,

x0 = 6/(3−√3/2) ≈ 2.811 < 4 é o úni o ponto ríti o e perten e a [0, 4]. Nos extremos do inter-valo, a(0) = a(4) = 0.Extra 13. (a) MODELAGEM: Considere x e y omo as dimensões do retângulo. Então quere-mos maximizar a área a = xy. Como são duasvariáveis, utilizamos a restrição P = 2x + 2ypara eliminar uma delas. Assim y = P/2 − x.Logo queremos o máximo de a(x) = xP/2− x2.Note que x pode ser 0 no mínimo, mas omo

P = 2x+2y e y pode valer no mínimo 0, x podevaler no máximo P/2. Assim queremos o máximode a(x) = xP/2− x2 para x ∈ [0, P/2].RESOLUÇO: Como a′(x) = P/2−2x, x0 =P/4 é o úni o ponto ríti o e perten e ao inter-valo [0, P/2]. Como a(x) é uma parábola om on avidade para baixo, o máximo é em x0. Neste aso, omo 2x0 + 2y0 = P , 2y0 = P − 2P/4 =P/2. Assim x0 = y0 = P/4 e portanto o retân-gulo é um quadrado.(b) MODELAGEM: Considere x e y omoas dimensões do ampo, om y o lado opostoao órrego. Então queremos maximizar a áreaa = xy. Como são duas variáveis, utilizamos arestrição P = 2x + y para eliminar uma delas.Assim y = P − 2x. Logo queremos o máximode a(x) = xP − 2x2. Note que x pode ser 0 nomínimo, mas omo P = 2x+ y e y pode valer nomínimo 0, x pode valer no máximo P/2. Assim

queremos o máximo de a(x) = xP − 2x2 parax ∈ [0, P/2].RESOLUÇO: Como a′(x) = P − 4x, x0 =P/4 é o úni o ponto ríti o e perten e ao inter-valo [0, P/2]. Como a(x) é uma parábola om on avidade para baixo, o máximo é em x0 =P/4. Neste aso, omo 2x0 + y0 = P , y0 =P − 2P/4 = P/2 > x0 = P/4.( ) MODELAGEM: Considere x e y omo asdimensões do terreno, onde x é da er a refor-çada. Então queremos maximizar a área a = xy.Como são duas variáveis, utilizamos a restriçãodo usto total da er a 6000 = 3(2x) + 2(2y).Assim 6000 = 6x+4y ou 3000 = 3x+2y. Logo,y = 1500 − 3/2x. Logo queremos o máximo dea(x) = x(1500−3/2x). Note que x pode ser 0 nomínimo, mas omo 3000 = 3x+2y e y pode valerno mínimo 0, x pode valer no máximo 1000. As-sim queremos o máximo de a(x) = 1500x−3/2x2para x ∈ [0, 1000].RESOLUÇO: Como a′(x) = 1500−3x, x0 =500 é o úni o ponto ríti o e perten e ao intervalo[0, 1000]. Como a(x) é uma parábola om on- avidade para baixo, o máximo é em x0 = 500.Neste aso, omo y0 = 1500 − 3/2x0 = 750.Extra 14. (a) MODELAGEM: Queremos o má-ximo de x2 + y2 mas om x + y = S. Logoqueremos maximizar f(x) = x2 + (S − x)2 parax ∈ R.RESOLUÇO: Como f ′(x) = 2x−2(S−x) =4x − 2S, o ponto ríti o é x0 = S/2. Como afunção de res e antes de x0 (f ′ < 0) e res edepois, este ponto é de mínimo lo al e global (naverdade f(x) → +∞ quando x → ±∞). Neste aso y0 = S − x0 = S/2 = x0.(b) MODELAGEM: Queremos máximo e mí-nimo de g(x) = x− x3 para x ∈ [0,+∞].RESOLUÇO: Como g′(x) = 1 − 3x2, ospontos ríti os são ±

√3/3. Mas somente x0 =√

3/3 ∈ [0,+∞]. Pelo sinal da derivada, x0 émáximo lo al e positivo. Note que g(0) = 0,lim

x→+∞g(x) = −∞. Logo o máximo é em x0 eNO existe mínimo.( ) MODELAGEM: Queremos o máximo eo mínimo de x + y om xy = P > 0. As-sim queremos o mínimo de h(x) = x +

P

x om

x ∈ (0,+∞).RESOLUÇO: Como h′(x) = 1 − P

x2, ospontos ríti os são ±

√P . Mas somente √

P ∈(0,+∞) Note que h(x) → +∞ quando x → 0

Page 86: Sumario´ - labMA/UFRJmcabral/livros/livro-calculo/... · 2019. 6. 12. · Sumario´ 1 Enunciados dos Exercícios 1.1 Exercícios de Limite. 1 1.1.1 Exercícios de Fixação. 1 1.1.2

82 CAPÍTULO 2. RESPOSTAS DOS EXERCÍCIOSou x → +∞. Assim o mínimo é em x0 =√P , om y0 = x0 =

√P e a soma mínima igual a

2√P . O máximo NO existe.Extra 15. (a) MODELAGEM: Seja x o lado doquadrado na base e h a altura. Logo V = hx2.A quantidade de material é propor ional a somadas áreas dos lados A = x2+4xh ( aixa aberta).Como hx =

V

x, queremos minimizar A(x) =

x2 +V

xpara x ∈ (0,+∞).RESOLUÇO: Como A′(x) = 2x − V

x2, oúni o ponto ríti o é x0 = 3

V/2. Note queA(x) → +∞ quando x → 0 ou x → +∞. Assimx0 é um ponto de mínimo (pode-se ver tambémpelo sinal da derivada: função de res e até x0 e res e depois).(b) MODELAGEM: Seja x o lado do qua-drado na base e h a altura. Logo V = hx2.A quantidade de material é propor ional a somadas áreas dos lados A = x2+4xh ( aixa aberta).Como hx =

A− x2

4, queremos maximizar V (x) =

xA− x2

4. Note que se h = 0, x =

√A. Assim

x ∈ [0,√A].RESOLUÇO: Como V ′(x) =

A− 3x2

4, ospontos ríti os são x = ±

A/3. Mas o úni o nointervalo [0,√A] é x0 =

A/3. Como V (0) =V (

√A) = 0 e V (x0) > 0 o máximo é em x0.( ) MODELAGEM: Seja x o lado do quadradona base e h a altura. Logo V = hx2. O ustoé C = 2(x2) + 4xh. Como hx =

V

x, queremosminimizar A(x) = 2x2 +

V

xpara x ∈ (0,+∞).RESOLUÇO: Como A′(x) = 4x − V

x2, oúni o ponto ríti o é x0 = 3

V/4. Note queA(x) → +∞ quando x → 0 ou x → +∞. Assimx0 é um ponto de mínimo (pode-se ver tambémpelo sinal da derivada: função de res e até x0 e res e depois).Extra 16. (a) MODELAGEM: Como a velo idadeé a mesma, o minimizar tempo é o mesmo que mi-nimizar aminho. Suponha que a distân ia de Qaté o espelho é a e de P até o espelho é b. Colo-que a origem no espelho no ponto que é a projeçãoortogonal de Q no espelho. Introduza x omo alo alização do ponto O e seja c o ponto que é aprojeção ortogonal de P no espelho. Por Pitágo-ras a distân ia total per orrida pelo raio em fun-

ção de x é: d(x) =√a2 + x2 +

b+ (c− x)2.Queremos o mínimo om x ∈ R.RESOLUÇO: Note que d(x) → +∞ quandox → ±∞. Assim o mínimo é no ponto om deri-vada zero. Como

d′(x) =x√

a2 + x2− c− x√

b+ (c− x)2.Se d′(x0) = 0 então:

x0√

a2 + x20=

c− x0√

b+ (c− x0)2.Note que isto impli a que sen θi = sen θr. Comoos ângulos são entre 0 e π/2, θi = θr.(b) Como a velo idade é a mesma, o minimi-zar tempo é o mesmo que minimizar aminho.Suponha que a distân ia de Q até a interfa eentre os meios é a e de P até a interfa e é b.Coloque a origem na interfa e no ponto que é aprojeção ortogonal de Q na interfa e. Introduza

x omo a lo alização do ponto O e seja c o pontoque é a projeção ortogonal de P no espelho. PorPitágoras al ulamos a distân ia em ada tre ho:√a2 + x2 e √b+ (c− x)2. O tempo será ob-tido dividindo distân ia pela velo idade. Assim otempo total per orrido pelo raio em função de xé t(x) = √

a2 + x2

v2+

b+ (c− x)2

v1. Queremoso mínimo om x ∈ R.RESOLUÇO: Note que t(x) → +∞ quando

x → ±∞. Assim o mínimo é no ponto om deri-vada zero. Comot′(x) =

x

v2√a2 + x2

− c− x

v1√

b+ (c− x)2.Se t′(x0) = 0 então:

x0

v2√

a2 + x20=

c− x0

v1√

b+ (c− x0)2.Note que isto impli a que sen θ2

v2=

sen θ1v1

.Extra 17. (a) Se y = t, x = ±√t2 + 1. Assima distân ia ao quadrado d(t) = (t − 2)2 + t2 +

1. Como d′(1) = 0, o ponto mais próximo é(±

√2, 1).(b) A distân ia ao quadrado é d(x) = (x −

4)2 + x6. Como d′(x) = 6x5 + 2x − 8. Umadas raízes é x = 1 e a função d′ é res ente (suaderivada é sempre positiva). Assim é a úni a raiz.Assim o ponto é (1, 1).( ) Se x = t, y = ±√8− 4t2, om t ∈

[−√2,√2]. A distân ia ao quadrado é d(t) =

Page 87: Sumario´ - labMA/UFRJmcabral/livros/livro-calculo/... · 2019. 6. 12. · Sumario´ 1 Enunciados dos Exercícios 1.1 Exercícios de Limite. 1 1.1.1 Exercícios de Fixação. 1 1.1.2

2.4. APLICAÇO DE DERIVADA 83(t − 1)2 + 8 − 4t2. Logo d′(t) = −6t − 2. As-sim d′(−1/3) = 0. Agora temos que omparard(−1/3) = 28/3 ≈ 9.33 om d(

√2) = (

√2 −

1)2 ≈ 0.17 e d(−√2) = (−

√2 − 1)2 ≈ 5.28.Logo o ponto mais perto é (

√2, 0).(d) A distân ia ao quadrado é d(x) = (x −

2)2 + x para x > 0. Assim d′(x) = 2x − 3. As-sim d′(3/2). Devemos omparar d(3/2) = 7/4 om d(0) = 4 (d(+∞) = +∞, o outro ex-tremo do intervalo). Assim o ponto mais pertoé (3/2,√

3/2).Extra 18.MODELAGEM: Seja 2x o lado do re-tângulo no diâmetro do semi ír ulo e y o outrolado. A área é A = 2xy. Note que ligando-se o entro do ír ulo a um vérti e do retângulo obte-mos um triângulo retângulo om lados x, y,R.Assim, por Pitágoras, x2 + y2 = R2. Logo,y =

√R2 − x2 para x ∈ [0, R]. Assim que-remos o máximo de A(x) = 2x

√R2 − x2 para

x ∈ [0, R].RESOLUÇO: Como A′(x) = 2R2 − 2x2√R2 − x2

.A raiz positiva será x0 =R√2. Como x20 + y20 =

R2, y0 = x0 =R√2. Note que A(0) = A(R) = 0.Assim o máximo será em x0. Logo as dimensõessão 2x0 = R

√2 e y0 =

R√2.Extra 19.MODELAGEM: Sejam h e r os ladosdo retângulo. Quando giramos o retângulo emtorno do lado de tamanho h obtemos um ilindrode volume V = πr2h. Como P = 2h + 2r,

h = P/2−r. Assim queremos maximizar V (r) =πr2(P/2 − r). Se h = 0, r = P/2. Assim r ∈[0, P/2].RESOLUÇO: Como V ′(r) = πr(P−3r), ospontos ríti os são 0 e P/3, ambos no intervalo.Mas V (0) = V (P/2) = 0. Assim r0 = P/3 é oponto de máximo. Então h0 = P/2− r0 = P/6.Assim r0

h0= 2.Extra 20. (a) MODELAGEM: Seja r o raio e ha altura do ilindro ins rito na esfera. O volumedo ilindro é V = πr2h. Como é função de duasvariáveis, devemos eliminar uma delas.Ligando-se o entro da esfera até um dos pon-tos do ír ulo da base do ilindro observamos otriângulo retângulo om hipotenusa R e atetos

r e h/2. Logo, por Pitágoras, (h/2)2 + r2 = R2,Assim, r2 = R2 − h2/4. Logo V (h) = π(R2 −h2/4)h. Note que h ∈ [0, 2R]. Assim queremos

o máximo de V (h) para h ∈ [0, 2R].RESOLUÇO: Note que V (0) = V (2R) = 0.Como V ′(h) = π(R2 − 3h2/4), o ponto ríti opositivo é h0 = 2R/√3 que perten e ao intervalo(√3 > 1). Este será o ponto de máximo pois afunção é positiva em h0.(b) MODELAGEM: Seja r o raio e h a alturado one ins rito no one. O volume do one é

V =1

3πr2h. Note que omo 1

3π é um onstante,maximizar a função f = r2h é um problema equi-valente. Como é função de duas variáveis, deve-mos eliminar uma delas. Por semelhança de tri-ângulos, H

H − h=

R

r. Logo, h = H(1 − r/R).Assim queremos maximizar f(h) = r2H(1−r/R)para r ∈ [0, R].RESOLUÇO: Como f ′(r) = H/Rr(2R −

3r). As raízes são 0 e 2R/3, ambas no intervalo[0, R]. Mas f(0) = f(R) = 0. Assim o máximoé em r0 = 2R/3.Extra 21.MODELAGEM: Chame de x a distân- ia da base da es ada até a base da er a, de ya distân ia da base do prédio até o ponto ondea es ada en osta no prédio e de d o tamanho daes ada. Por Pitágoras (x + L)2 + y2 = d2. Porsemelhança de triângulos, H/x = y/(x+L). As-sim, y = H(x+L)/x. Utilizando Pitágoras obte-mos que d2(x) = (x+L)2(1+H2/x2). Queremosminimizar d2(x) para x ∈ (0,+∞).RESOLUÇO: É fá il ver que d2(x) → +∞nos extremos deste intervalo. Assim o mínimo éno ponto onde a derivada se anula. Derivandod2(x) obtemos 2(L+ x)(x3 − LH2)

x3. As raízesreais são x0 = −L (des artada pois está fora dointervalo de minimização) e x0 =

3√LH2. Por-tanto a menor distân ia é d(

3√LH2).2.4.4 ⋆Problemas (Taxas Rela iona-das) da p.31Problema 1. Seja x a distân ia entre a aixa e oponto P . Claro que tan θ(t) =

1

x(t). Derivandoobtemos que

θ′

cos2 θ= − x′

x2.Quando x(t) = 1, θ =

π

4. Como x′ = −2 (osinal é negativo pois a aixa está sendo puxada,diminuindo o valor de x), substituindo na relação

Page 88: Sumario´ - labMA/UFRJmcabral/livros/livro-calculo/... · 2019. 6. 12. · Sumario´ 1 Enunciados dos Exercícios 1.1 Exercícios de Limite. 1 1.1.1 Exercícios de Fixação. 1 1.1.2

84 CAPÍTULO 2. RESPOSTAS DOS EXERCÍCIOSa ima, obtemos θ′

cos2(π/4)= −−2

12. Logo, 2θ′ =

2, ou, θ′ = 1m/min.Problema 2. Seja x(t) a posição do arro e y(t) aposição do trem om a a origem na interseção darodovia e os trilhos do trem. Agora por Pitágorasa distân ia d(t) satisfaz d2(t) = x2(t) + y2(t) +102. Pelos dados, x(0) = y(0 = 0. Além disso,x(2) = 2(40) = 80 e y(2) = 2(20) = 40. Noinstante t = 2 a distân ia entre eles por Pitágorasé d(2) = 90.Derivando (e dividindo por 2) obtemos quedd′ = xx′ + yy′. No instante t = 2, omo d =90, x = 80, x′ = 40, y = 40, y′ = 20, obtemosque 90d′(2) = 80(40) + 40(20). Logo d′(2) =400

9m/s.Outra forma (mais ompli ada) sem utilizartaxas rela ionadas: Colo ando a origem na inter-seção, na altura do trilho do trem, om o eixo xna direção do movimento do arro e o eixo y nadireção do movimento do trem, o arro en ontra-se no instante s em c(s) = (40s, 0, 10) e o trem(o nal do último vagão) em t(s) = (0, 20s, 0).A distân ia d(s) =

(40s)2 + (20s)2 + 10. As-sim,d′(s) =

2 · 40s · 40 + 2 · 20s · 202√

(40s)2 + (20s)2 + 10.Cal ulando obtemos que d′(2) =

400

9m/s.Problema 3. Seja x a distân ia da base da es adaaté a parede e y a altura do topo da es ada. PorPitágoras, x2+y2 = L2. Assim, 2xx′+2yy′ = 0.Como x′ = V e quando y = H, x =√L2 −H2,

V√L2 −H2 +Hy′ = 0. Basta resolver para y′,a velo idade do topo: y′ = −(V

√L2 −H2)/H.Problema 4.Por semelhança de triângulos, se ré o raio de água do tanque om h metros deprofundidade, h(t) = r(t)H/R. Assim o volume

v(t) = π/3r2(t)h(t). Assim v′(t) = π/3(2rr′h+r2h′) e h′ = r′H/R. Assim se v′ = V , h = Lentão r = LR/H e r′ = h′R/H.V = π/3(2(LR/H)(h′R/H)L+ (LR/H)2h′) =π(LR/H)2h′. Resolvendo para h′ obtemos: h′ =V H2

πR2L2.Problema 5. Seja O o ponto no solo verti al-mente abaixo do balão. Se h(t) é altura do solo,

x(t) distân ia de O até o arro, e d(t) a distân iabalão- arro, por Pitágoras h(t)2 + x(t)2 = d(t)2.

Logo 2hh′ + 2xx′ = 2dd′. Após 4 segundos:h(4) = 48 + 4 · 3 = 60, x(4) = 0 + 20 · 4 =80. Por Pitágoras, d(4) = 100. Como h′(4) =3 e x′(4) = 20. substituindo na equação (ob-tida por derivação implí ita) obtemos: 2(60)(3)+2(80)(20) = 2(100)d′(4). Logo a variação da dis-tân ia vale d′(4) = 17, 8m/s.Problema 6. Se P = (x0, y0), a equação dareta tangente t que passa em P é y − y0 =−2x0(x − x0). Como y0 = −x20 + 1, a equa-ção de t é: y = 1−x20−2x0(x−x0). O Ponto Qé a interseção de t om o eixo x. Basta olo ary = 0 na equação da reta t para se obter que a oordenada x de Q é: x0 +

1− x202x0

. Note queM = (x0, 0). Assim, MQ =

1− x202x0

. LogoMQ′(t) =

−2x0(t)x0(t)′(2x0(t))− (1− x20)2x

′0(t)

4x0(t)2.Simpli ando,

MQ′(t) =−4x20(t)x

′0(t)− (1− x20)2x

′0(t)

4x0(t)2. To-mando x0 = 1/

√2, obtemos que MQ′(t) =

−3cm/min.Problema 7. Seja d a distân ia da fonte até aparede e r o raio da área iluminada. Como oângulo é de 90o, a o triângulo retângulo ujos atetos são r e d é isós eles. Assim r = d. LogoA(t) = πr(t)2 = πd(t)2. Assim, A′ = 2πdd′.Logo A′ = 2πk(−a) = −2kaπ.Problema 8.Vamos hamar de x o outro lado doretângulo. Por semelhança de triângulos,40− x

40=

y

60. Quando y = 36, resolvendo aequação obtemos que x = 16. Como, −x′

40=

y′

60,quando y′ = 0, 5, x′ = −1/3.Assim a área do retângulo A(t) = x(t)y(t)varia em função do tempo por A′ = x′y + xy′.Logo no instante t quando y′ = 0, 5, x′ = −1/3, y =

36, x = 16, temos que A′(t) = −4cm/s. Logo aárea está diminuindo neste instante.Problema 9. (a) O volume V (r) = 4/3πr3. As-sim V ′(r) = 4πr2. Seja r(t) o raio do balãoem função do tempo. Por hipótese r′(t) = −15para todo t. Seja f(t) = V (r(t)) a variação novolume em função do tempo. Então f ′(t) seráa taxa om o ar estará saindo do balão. Então,f ′(t) = V ′(r(t))r′(t) = −15(4πr2(t)). Assim ser(t) = 9, f ′(t) = −15(4π81) = −4860πm3/s.(b) Es revemos o volume do balão em fun-ção do tempo e do raio R(t) (o raio depende do

Page 89: Sumario´ - labMA/UFRJmcabral/livros/livro-calculo/... · 2019. 6. 12. · Sumario´ 1 Enunciados dos Exercícios 1.1 Exercícios de Limite. 1 1.1.1 Exercícios de Fixação. 1 1.1.2

2.4. APLICAÇO DE DERIVADA 85tempo) por V (t) = 4/3πR3(t). Como V ′(t) =4πR2(t)R′(t). Assim p0 = V ′(t0) = 4πR2

0R′(t0).Como A(t) = 4πR2(t), A′(t) = 8πR(t)R′(t).Como R′(t0) = p0/(4πR

20),

A′(t0) = 8πR0p0/(4πR20) = 2p0/R0.Problema 10. Sejam R(t) o raio da esfera, r(t)o raio e h(t) a altura do one ins rito na esfera.Ligando-se o entro da esfera até um dos pontosdo ír ulo da base do one observamos o triânguloretângulo om hipotenusa R e atetos r e h−R.Logo, por Pitágoras,

(h(t)−R(t))2 + r(t)2 = R(t)2.Agora são dados que R(t) = 1, h(t) = 4/3.Por esta relação obtemos que r(t) = 2√2/3 (ou

r2(t) = 8/9). Derivando e dividindo por 2 obte-mos que(h−R)(h′ −R′) + rr′ = RR′.Como R′ = 0, 9, h′ = 0, 8, R = 1, h = 4/3e r = 2√2/3, resolvendo para r′ obtemos que

r′ = 75√2ou rr′ =

14

15.O volume do one é V (t) =

π

3r(t)2h(t). As-sim a variação do volume do one V (t) é dadopor

V ′(t) =π

3(2r′(t)r(t)h(t) + r2(t)h′(t)).Substituindo os valores a ima (r(t)r′(t) =

14

15e

r2(t) = 8/9) obtemos que V ′(t) =16π

15m3/minProblema 11.A(t) = p2(t)/16, onde A é área e

p perímetro. Logo, A′(t) = p(t)p′(t)/8. Assim,se A(4) = 100, p(4) = 40. Como p′ = 3, A′ =40(3)/8 = 15m/s.Problema 12. Seja x(t) a distân ia da mulher atéo muro e s(t) o omprimento da sombra no muro.Considere a reta saindo do reetor, passando pela abeça da mulher até en ontrar o muro. Igua-lando ∆y

∆xdo intervalo 0 até x e de x até 40obtemos que

1, 80 − s(t)

x=

3− 1, 80

40− x(t)=

1, 2

40− x(t).Logo, (1, 80 − s(t))(40 − x(t)) = 1, 2x(t). Logoquando x(t) = 20 (meio aminho), s(t) = 3/5.Derivando obtemos que

(−s′)(40 − x) + (1, 80 − s)(−x′) = 1, 2x′.

Substituindo x = 20, s = 3/5, x′ = −4 (é nega-tiva pois x diminui quando aminhamos na dire-ção do muro) obtemos que s′ =12

25= 0, 48m/s.Problema 13.Colo ando a origem na asa dopes ador e hamando de x a posição na margemdo rio, temos que a distân ia entre o bar o e a asa é d(x) = √

x2 + 9. Assim, d′ = (xx′)/√x2 + 9.Note que no instante em que a distân ia é 5 entrebar o- asa, |x| = 4 (triângulo retângulo 3,4,5).Orientando a posição para que que negativa an-tes da asa, x′ = 10 e, em (a) x = −4 e em (b)

x = 4. Substituindo obtemos que a velo idade é8Km/h, om sinal negativo em (a) e positivo em(b).Problema 14. (a) Seja a(r) = πr2 a área do ír- ulo. Então a′(r) = 2πr. Assim f(t) = a(r(t)) éa variação da área em função do tempo e f ′(t) =a′(r(t))r′(t) = 2πr(t)r′(t). Logo se r′(t) = k,f ′(t) = 2kπr(t).(b) Seja V (x) = x3 o volume do ubo. EntãoV ′(x) = 3x2. Assim, f(t) = V (x(t)) é a vari-ação do volume em função do tempo e f ′(t) =V ′(x(t))x′(t) = 3x2(t)x′(t). Logo se x′(t) = k,f ′(t) = 3kx2(t) = k/2(6x2(t)), onde 6x2(t) é aárea de superfí ie.( ) Aqui V (r) = kr3 (k = 4/3π). AssimV ′(r) = 3kr2. Assim, f(t) = V (x(t)) é a vari-ação do volume em função do tempo e f ′(t) =V ′(r(t))r′(t) = 3kr2(t)r′(t). Suponha que f ′(t) =C4πr2(t). Então, C4πr2(t) = 3kr2(t)r′(t). Logo,r′(t) =

C4π

3k, uma onstante.2.4.5 ⋆Problemas (Derivação Implí- ita) da p.34Problema 1. (a) y′(1) = −5/38 e a reta tan-gente é y = 5− 5/38(x − 1).(b) Derivando impli itamente obtemos 2x =

−2y + 2y′x(x− y)2

. Substituindo obtemos que y′ = 1.Assim a reta tangente é y = x+ 1.Problema 2.Derivando impli itamente obtemosque y′ =y − x2

y2 − x. Assim a reta tangente seráhorizontal quando y′ = 0, isto é, quando y = x2.Substituindo em x3 + y3 = 3xy, obtemos que

x6 = 2x3, ujas raízes reais são x = 0 e x =3√2. Obtemos o y orrespondente substituindona equação x3 + y3 = 3xy: (0, 0), ( 3

√2, 3

√4).

Page 90: Sumario´ - labMA/UFRJmcabral/livros/livro-calculo/... · 2019. 6. 12. · Sumario´ 1 Enunciados dos Exercícios 1.1 Exercícios de Limite. 1 1.1.1 Exercícios de Fixação. 1 1.1.2

86 CAPÍTULO 2. RESPOSTAS DOS EXERCÍCIOSA reta será verti al quando y′ = ±∞. Assimbasta que o denominador y2 − x se anule, isto é,x = y2. Substituindo em x3 + y3 = 3xy, ob-temos, de forma análoga, y6 = 2y2, ujas raízesreais são y = 0 e y = 3

√2. Obtemos o y orres-pondente substituindo na equação x3+y3 = 3xy:

(0, 0), ( 3√4, 3

√2).Problema 3.A derivada implí ita é 2x− 2yy′ +

1

2√xy

(y + xy′) = 0.(a) Queremos y′(2) = f ′(2). Substituindox = 2 e y = 2, obtemos que4 − 4y′(2) +

1

4(2 + 2y′(2)) = 0. Logo, y′(2) =

f ′(2) =9

7.(b) y − 2 =

9

7(x− 2).( ) g′(x) =f ′(x)x− f(x)

x2. Logo g′(1) =

18/7 − 2

4=

1

7.Problema 4. (a) De res ente, f ′(1) = −1,

f ′′(1) = −19/3.(b) De res ente, f ′(1) = −π/2, f ′′(1) = π.Problema 5.Derivando impli itamente, obtemos4x3 − y − xy′ + 4y3y′ = 0. Os pontos an-didatos ao max/min são pontos onde y′ = 0.Assim obtemos que y = 4x3. Substituindo naequação x4 − xy + y4 = 253 obtemos a equa-ção 256x12 − 3x4 = 253. Por inspeção obtemosas raízes x = ±1. As outras raízes são om-plexas (graças ao Maxima!). Assim em x = 1,y = 4x3 = 4 e em x = −1, y = 4x3 = −4.Para determinar se os pontos são de máximoou mínimo vamos al ular a derivada segunda.Agora derivando impli itamente novamente obte-mos: 12x2 − 2y′ − xy′′ + 12y(y′)2 + 4y3y′′ = 0.Nos pontos onde y′ = 0: 12x2+(4y3−x)y′′ = 0.Em x = 1, y = 4, y′′ = −12/255 < 0: é má-ximo. Em x = −1, y = −4, y′′ = 12/257 > 0:é mínimo. Use um software para plotar a funçãoimplí ita e veri ar esta resposta. Este métodofun iona pois a urva denida pela equação é li-mitada.Problema 6.Como (1, 1) perten e a urva, 1 +a = b. A derivada implí ita é: 2xy + x2y′ +2ayy′ = 0. Logo em x = 1, y = 1, 2 + y′(1) +2ay′(1) = 0 ou (2a+1)y′(1) = −2. Logo y′(1) =−2

2a+ 1. Queremos que seja igual ao oe ienteangular de 4x + 3y = 7, que é −4/3. Assim

y′(1) =−2

2a+ 1= −4/3. Logo, a =

1

4e b =

1 + a =5

4.Problema 7.Primeiro rees revemos a urva omo

exp(y log x) = exp(x log y). Derivando impli i-tamente obtemos que xy(y′ log x+y/x) = yx(log y+xy′/y). Substituindo x = y = k0 obtemos quey′ log k0 + 1 = log k0 + y′. Portanto y′ = 1 e areta tangente é y = x.2.4.6 Desaos da p.35Desao 1.Para todos a resposta é k.Desao 2.Cal ule a série de Taylor de segundaordem em x = 1. Como f ′(x) =

−2

(x− 2)2,

f ′(1) = −2. Como f ′′(x) =4

(x− 2)3, f ′′(1) =

4. Assim, omo a série de Taylor de f(x) deordem 2 é f(1) + f ′(1)h + f ′′(1)h/2, obtemosp(h) = −1−2(h−1)−2(h−1)2 = −2h2+2h−1.Plote om algum software os grá os de f e dopolinmio p.Desao 3. (b)fk(x) =

k!(−1)k

2i

(

1

(x− i)k+1− 1

(x+ i)k+1

).( ) Use (b) e obtenha: x−x3/3+x5/5−x7/7+

x9/9. (d) Como arctan(1) =π

4, substitua x = 1na série de Taylor.Desao 5.O quadrado da distân ia entre o ponto

(s, f(s)) e (a, b) é d(s) = (s− a)2 +(f(s)− b)2.Assim o mínimo será no ponto onde f ′(s0) =0 = 2(s0−a)+2(f(s0)− b)f ′(s0), isto é quandof(s0)− b = −1/f ′(s0)(s0−a). Como f ′(s0) é o oe iente da reta tangente no ponto (s0, f(s0),−1/f ′(s0) é o oe iente da reta normal nesteponto. Assim obteremos a identidade f(s0) −b = −1/f ′(s0)(s0 − a) se, e somente se, (a, b)perten er a reta normal neste ponto. Portanto oponto mais perto de (a, b) no grá o de f é oponto onde a reta normal inter epta (a, b).Desao 6.Queremos minimizar a distân ia aoquadrado: f(x) = (x−x0)

2+(y(x)−y0)2, onde

y(x) é denida impli itamente por ax+ by(x) +c = 0. Derivando impli itamente, a+by′(x) = 0.Como f ′(x) = 2(x − x0) + 2(y(x) − y0)y

′(x),vamos obter w ∈ R tal que f ′(w) = 0. Resol-vendo: (w − x0) + (y(w)− y0)y′(w) = 0. Como

y′ = −a/b e y(x) = −(ax+ c)/b, temos que re-solver. (w−x0)+(−(ax+x)/b−y0)(−a/b) = 0.Com o Maxima obtemos que:w =

−aby0 + b2x0 − ac

b2 + a2.

Page 91: Sumario´ - labMA/UFRJmcabral/livros/livro-calculo/... · 2019. 6. 12. · Sumario´ 1 Enunciados dos Exercícios 1.1 Exercícios de Limite. 1 1.1.1 Exercícios de Fixação. 1 1.1.2

2.4. APLICAÇO DE DERIVADA 87Como (w−x0)+ (y(w)− y0)y′(w) = 0, (y(w)−

y0)y′(w) = −(w − x0) e y′ = −a/b,

(y(w) − y0)2(w − x0)

2(a2/b2).Logo,f(w) = (w − x0)

2(1 + a2/b2).Agora om o Maxima obtemos quef(w) =

(ax0 + by0 + c)2

a2 + b2.Desao 7. Soma das áreas dos trapézios é (1 +

1/x)/2(x − 1) + (1/x + 1/a)/2(a − x). Noteque esta área é maior que log a. Assim queremosminimizar f(x) = (a−1) x2+a2−a2 ax − log a. Cal u-lando f ′(x) = (a−1)x2−a2+a

2 ax2 . Logo f ′(x0) = 0 ex0 > 0 se x0 =

√a.Desao 8. (a) MODELAGEM: Seja r o raio e ha altura do one ins rito na esfera. Área lateraldo one é A = πrl, onde l é o omprimento dalateral do one, que por Pitágoras satisfaz l2 =

h2 + r2. Maximizar A é o mesmo que maximizarA2 = π2r2l2 = π2r2(h2 + r2).Ligando-se o entro da esfera até um dos pon-tos do ír ulo da base do one observamos o tri-ângulo retângulo om hipotenusa R e atetos r eh−R. Logo, por Pitágoras, (h−R)2+ r2 = R2,Assim, r2 = 2hR − h2.Assim queremos maximizarA2(h) = π2(2hR−h2)2hR = 2π2Rh2(2R − h) para h ∈ [0, 2R].RESOLUÇO: A2(h) vale zero nos extremosdo intervalo. A derivada de A2(x) em relação ax é 2π2Rh(4R − 3h). Assim os pontos ríti ossão h = 0 e h = 4R/3. Logo o máximo é emh = 4R/3, o mesmo resultado que obtemos semaximizarmos o volume ao invés da área.(b) MODELAGEM: Seja r o raio e h a alturado ilindro ins rito no one. A área do ilindroé A = 4πr2 + 2πrh. Como é função de duasvariáveis, devemos eliminar uma delas. Note que ortando o one temos uma semelhança de triân-gulos: a alturaH do one está para R assim omoH − h está para r. Assim, H

R=

H − h

r. Logo,

h = H(1 − r/R). Logo queremos maximizarA(r) = 4πr2 + 2πrH(1− r/R) para r ∈ [0, R].RESOLUÇO: Note que A(0) = 0 e queA(R) = 4πR2 é andidato a máximo. ComoA′(r) = 2π/R(HR−r(2H−4R)), o úni o ponto ríti o é r0 =

HR

2H − 4R.

Agora pode-se provar que r0 ∈ [0, R] se, esomente se, H ≥ 4R. Neste aso o máximo seráem r0 e A(r0) =πH2R

2H − 4R( al ulei om Ma-xima, não pre isa al ular).Por outro lado, se H < 4R então r0 6∈ [0, R]e o máximo será em r = R, om área igual a

A(R) = 4πR2.Desao 9.MODELAGEM: Suponha que o raiodo ír ulo é 1. Colo ando a origem no entro do ír ulo queremos partir de θ = 0 hegar em θ =π. Devemos nadar em linha reta de θ = 0 até ϕ edepois orrer na beira do lago até θ = π. O per- urso nadando possui omprimento igual a distân- ia entre (cos 0, sen 0) = (1, 0) e (cosϕ, senϕ).Assim distân ia de nado ao quadrado: (cosϕ −1)2 + (senϕ)2. A distân ia orrendo é o ar o de ír ulo de ângulo π−ϕ. Colo ando omo velo i-dade 1 e 2 para nadar e orrer respe tivamente, otempo t(ϕ) =

(cosϕ− 1)2 + (senϕ)2 + (π −ϕ)/2. Note que ϕ ∈ [0, π].RESOLUÇO: t′(x) = −

√2−2 cos(x)−2 sen(x)

2√

2−2 cos(x).Assim queremos determinar x0 tal que√

2− 2 cos x0 = 2 senx0. Assim, x0 = 0 oux0 = 2π/3. Comparando t(0) = π/2 ≈ 1.57,t(π) = 2 e t(2π/3) = π/6 +

√3 ≈ 2.25. Assimo melhor é orrer diretamente para o outro ladodo lago e levar tempo t(0).Desao 10.Queremos que o ír ulo (x − a)2 +

(y(x) − b)2 = r2 os ule a urva. Derivando im-pli itamente obtemos que (x−a)+ (y− b)y′ = 0e 1 + (y − b)y′′ + (y′)2 = 0. Para que o ír uloos ule, quando x = c: y = η0, y′ = η1, y

′′ = η2.Temos que determinar a, b, r tais que:

(c− a)2 + (η0 − b)2 = r2,(c− a) + (η0 − b)η1 = 0,1 + (η0 − b)η2 + (η1)

2 = 0.Resolvendo, obtemos r (se quiser a e b também).Veja [Ha, p.299 e [Co, p.333 e p.283.Desao 11.É fá il provar que g(x) → +∞ quandox → ±∞. Como g(x) ≥ 0, o problema tem mí-nimo global. Pela linearidade entre um ponto eoutro, o mínimo será em um dos pontos. Se n éimpar, o mínimo o orrerá no ponto do meio, sen for par a função será onstante entre os doispontos do meio.Desao 12.A derivada será a soma de termos dotipo e−1/x2

xk, om k ∈ N. Por LH ada um dostermos tem limite igual a zero.

Page 92: Sumario´ - labMA/UFRJmcabral/livros/livro-calculo/... · 2019. 6. 12. · Sumario´ 1 Enunciados dos Exercícios 1.1 Exercícios de Limite. 1 1.1.1 Exercícios de Fixação. 1 1.1.2

88 CAPÍTULO 2. RESPOSTAS DOS EXERCÍCIOS2.5 Integral2.5.1 Exer. de Fixação da p.37Exer í io 1. (a) Falso. A função pode ser posi-tiva num intervalo e negativo em outro de modoque as áreas se an elam. Exemplo ∫ 2π

0senx dx =

0 mas sen(x) não é zero para todo x.(b) Verdadeiro, pelo Lema 9 da p.149 (mo-notoni idade da integral).( ) Falso. Podemos mudar a integral num nú-mero nito que o valor da integral será mantido.Exer í io 2. (a) Pela Denição 26 da p.149,∫ −1

2f(x) dx = −

∫ 2

−1f(x) dx = −5.(b) Utilizando a linearidade, 5+2(−3) = 5−

6 = −1.( ) Pela Denição 26 da p.149, a integral é 0(mesmos limites de integração).(d) Pelo Lema 9 da p.149, item ( ),∫ 2

−1(· · · ) =

∫ 0

−1(· · · ) +

∫ 2

0(· · · ).Assim,

∫ 2

0f(x) dx =

∫ 2

−1f(x) dx−

∫ 0

−1f(x) dx =

= 5− 7 = −2.(e) Note que g(t) é onstante na integral emds. Assim,∫ 0

−1f(s)g(t) ds = g(t)

∫ 0

−1f(s) ds = g(t)7.Assim,

∫ 2

−1

(∫ 0

−1f(s)g(t) ds

)

dt =

∫ 2

−1(g(t)7) dt

= (−3)7 = −21.(f) Mudar a função em um úni o ponto NOaltera o valor da integral. Assim ∫ 2

−1h(x) dx =

∫ 2

−1f(x) dx = 5.Exer í io 3. (a) F (0) = 0, F (1) = 2 (área doretângulo), F (2) = F (1)+1 = 3, F (3) = F (2)−

1/2 = 5/2.006.o t.2011

(b) F vai res er onde f é positiva, pois aárea vai aumentar. Assim, F res e em (0, 2) edepois de x = 5 e de res e em (2, 5).( ) Máximo lo al em x = 2 pois estava res- endo e passa a de res er e mínimo lo al emx = 5 pois estava de res endo e passa a res- er.Exer í io 4.O aluno X apli ou de forma in or-reta o TFC pois o integrando não é ontínuo em[−2, 2] (a função não está denida em x = 0). Oaluno Y está quase orreto. Como a função nãoestá denida trata-se de uma integral imprópria,que deveria ser es rita omo soma de integrais:∫ 0

−2+

∫ 2

0. Ambas divergem para +∞.Exer í io 5. (a) h(2) = ∫ 2

2(. . .) = 0.(b) Pelo TFC, h′(x) =(5− x)5

x4 + 6. Assim osinal de h′ é igual ao sinal de 5−x. Logo h′(x) >

0 (e h res e) se x < 5 e h de res e para x > 5.( ) somente em x = 5 a derivada é zero.Como h′ é positiva antes e negativa depois, x = 5é máximo lo al.Exer í io 6.Pelo Corolário 10 da p.151,∫ 1

−1h(s) ds = g(1) − g(−1) =

= (Ke+B − C)− (Ke−1 −B − C) =

= K(e− e−1) + 2B.Exer í io 7. (a) a primitiva é x4/2−x3 +5x. Oresultado é 9/2.(b) No intervalo [0, 1], y2 − 1 é negativo.Assim, |y2 − 1| = 1 − y2. Logo, a primitiva éy − y3/3. O resultado é 2/3.( ) Note que a integral é em t. Logo, x é onstante nesta integral. Assim,∫

(3x+ et − 7x sen t) dt = 3xt+ et + 7x cos t.Exer í io 8.Todos integraram orretamente. Pri-mitivas podem diferir por uma onstante. Noteque:−cos2x

2=

sen2x− 1

2=

sen2x

2+ C,

−cos 2x

4= −cos2 x− sen2x

4=

= −1− 2sen2x

4=

sen2x

2+ C.

Page 93: Sumario´ - labMA/UFRJmcabral/livros/livro-calculo/... · 2019. 6. 12. · Sumario´ 1 Enunciados dos Exercícios 1.1 Exercícios de Limite. 1 1.1.1 Exercícios de Fixação. 1 1.1.2

2.5. INTEGRAL 89Exer í io 9.(a) ∫ 5

−∞e−s2 ds = lim

k→−∞

∫ 5

kf(s) ds.(b) ∫ 2

0

dx

log(5− x2)= lim

k→2−

∫ k

0

dx

log(5− x2).( ) ∫ +∞

0

dy

1 + y4= lim

k→+∞

∫ k

0

dy

1 + y4.(d) ∫ 2

1

dx

x10 − 1= lim

k→1+

∫ 2

k

dx

x10 − 1.Exer í io 10. (a) Tome x = t/c. Logo dx =

dt/c. Logo, dt = c dx. Assim, quando t = ac,x = a e quando t = bc, x = b.(b) Tome u = x− c. Logo du = dx. Assim,quando x = a + c, u = a e quando x = b + c,u = b.Exer í io 11. (a) Tome u = K−3x. Então du =

−3dx. Assim devemos integrar ∫ 4√u(−du/3) =

−4/15u5/4. Logo a integral é −4/15(K−3x)5/4.(b) Tome u = x3. Então du = 3x2dx. Assimdevemos integrar ∫ cosu du = senu = sen(x3).( ) Tome u = log t. Então du = dt/t. Assimdevemos integrar ∫ u2du = u3/3 = log3 t/3.(d) Tome u = 3−2x2. Então, du = −4x dx.Assim devemos integrar∫ √

u (−du/4) = −u3/2

6= −(3− 2x2)3/2

6.(e) Tome u = cos θ. Então, du = − sen θ dθ.Assim devemos integrar

− du√u= −2

√u = −2

√cos θ.(f) Tome u = 5 senx. Então, du = 5cos x dx.Assim devemos integrar

eu (du/5) =eu

5=

e5 senx

5.Exer í io 12. (a) Tome u = log x e dv = x.Assim, du = dx/x e v = x2/2. Logo,

x log x dx =x2 log x

2−∫

x2

2

dx

x.Esta última integral a ∫ x

2dx =

x2

4. Logo,obtemos que

x log x dx =x2 log x

2− x2

4.

(b) (Veja exemplo de integral de arcsen nap.159). Tomando u = arctan x e dv = 1 · dx,du =

dx

x2 + 1e v = x. Logo,

arctan x dx = x arctan x−∫

x

x2 + 1dx.Agora vamos resolver a integral tomando z =

x2 + 1, dz = 2x dx. Logo,∫

x

x2 + 1dx =

dz

2z=

log z

2=

log(x2 + 1)

2.Juntando tudo obtemos,

arctan x dx = x arctan x− log(x2 + 1)

2.Exer í io 13. (a) Tome u = 3−2x. Assim du =

−2dx. Logo,∫

(3− 2x)4 dx =

u4(−du

2) = −u5

10=

= −(3− 2x)5

10.Substituindo os limites de integração:

∫ 2

1(3− 2x)4 dx = −(3− 2x)5

10

2

1

=

= −(3− 2(2))5

10+(3− 2(1))5

10=

1

10+

1

10=

1

5.(b) Tome u = −x/4. Assim du = −dx/4.Logo,

eu(−4) du = −4eu = −4e−x/4.Substituindo os limites de integração:∫ +∞

log 3e−x/4 dx = −4e−x/4|+∞

log 3 =

= 4(14√3− e−∞/4) =

44√3.( ) A primitiva é −cos(2θ)

2. Logo,

∫ π

π/4sen(2θ) dθ = −cos(2π)

2+

cos(π/2)

2= −1

2.(d) A primitiva é − 1

2s2. Logo a integral vale

lims→+∞

− 1

2s2+

1

2= 0 +

1

2=

1

2.

Page 94: Sumario´ - labMA/UFRJmcabral/livros/livro-calculo/... · 2019. 6. 12. · Sumario´ 1 Enunciados dos Exercícios 1.1 Exercícios de Limite. 1 1.1.1 Exercícios de Fixação. 1 1.1.2

90 CAPÍTULO 2. RESPOSTAS DOS EXERCÍCIOS2.5.2 Problemas da p.39Problema 1.Primeiro esbo e o grá o:x

y

1 2 3 4 5−1

1

4

Agora al ule as integrais determinando asáreas om sinal.(a) Área do triângulo igual a 4 menos a áreado quadrado igual a 1. Logo a integral é 4−1 = 3.(b) Área do retângulo ( om sinal negativo):−2.( ) Área do trapézio igual a 3 menos a áreado retângulo igual a 2 mais área do triângulo iguala 1/2. Logo a integral é 3− 2 + 1/2 = 3/2.Problema 2. (a) Basta apli ar a (monotoni idadeda integral) do Lema e observar que ∫ b

aM dx =

M

∫ b

adx = M(b− a).(b) Novamente, note que

sen(qualquer oisa) ≥ −1. Como log é res- ente, seu menor valor em [e, 5e] é log e = 1.Assim a função é limitada inferiormente por m =−4. De forma análoga ao item (a), limitamos aintegral por baixo por m(6e−e) = 5em = −20e.Problema 3.Pelo Teorema 5.2 da p.150 (TFC),F ′(x) =

x2 − 1

x2 + 1. O sinal de F ′ será determinadopelo numerador pois o denominador é sempre po-sitivo.(a) F é res ente em x > 1 e x < −1; F éde res ente em (−1, 1).(b) F ′′(x) =

4x

(x2 + 1)2. Assim a on avi-dade é para ima em x > 0 e para baixo em

x < 0.( ) A derivada é zero em ±1. Mas o mínimolo al é em x = 1 pois a on avidade do grá aé para ima neste ponto. O máximo lo al é emx = −1 onde a on avidade é para baixo.Problema 4.Pelo Teorema 5.2 da p.150 (TFC)e pela regra da adeia,

y′(x) =2x cos(x2 − π)

2 + sen(x2 − π)+ cos(x2). Logo,

y′(√π) =

√π + 1. Assim a equação da retatangente é y − log(2) =

√π(x − √

π) ou y =√πx+ log(2) − π.Problema 5. (a) Dena

H(y) =

∫ y

1et

3

dt e G(k) =

∫ k

4cos(1 + s2) ds.Agora pelo TFC, H ′(y) = ey

3 e G′(k) = cos(1+k2). Assim, omo f(y) = G(H(y)), pela regra da adeia, f ′(y) = G′(H(y))H ′(y). Logo, f ′(1) =G′(H(1))H ′(1). ComoH(1) = 0, f ′(1) = cos(1)e.(b) Dena J(x) =

∫ x

8log(t3 + 1) dt. As-sim, g(y) = ∫ 5

yJ(x) dx = −

∫ y

5J(x) dx. Logo,pelo TFC, g′(y) = −J(y) = −

∫ y

8log(t3+1) dt.Assim, g′(8) = 0.Problema 6. (a) A primitiva é x1−p

1− p. Logo aintegral vale 1

p− 1+ lim

N→+∞N1−p

1− p. Para que olimite seja nito (na realidade será zero) o expo-ente de 1−p deve ser negativo. Assim, 1−p < 0,o que impli a que 1 < p. Assim a integral seránita se p > 1 e valerá 1

p− 1.(b) Novamente a primitiva é x1−p

1− p. Logo aintegral vale 1

1− p+lim

k→0

k1−p

1− p. Para que o limiteseja nito (na realidade será zero) o expoente de

1− p deve ser positivo. Assim, 1− p > 0, o queimpli a que p < 1. Assim a integral será nita se0 < p < 1 e valerá 1

1− p.Problema 7. (a) Substitua u =

√k.R: 2 sen(√k) +C.(b) Substitua u = 1− 3x2.R: −√

1− 3x2 +C( ) Deverá ser feita a substituição u = 3x+1.Depois uma integração por partes tomando z = xe dw = sen(u).R :

sen(3x+ 1)− 3x cos(3x+ 1)

9+ C.(d) Tomando u = tan x, obtemos ∫ log u du =

u log u− u.R: tan x log(tan x)− tanx

Page 95: Sumario´ - labMA/UFRJmcabral/livros/livro-calculo/... · 2019. 6. 12. · Sumario´ 1 Enunciados dos Exercícios 1.1 Exercícios de Limite. 1 1.1.1 Exercícios de Fixação. 1 1.1.2

2.5. INTEGRAL 91(e) Substitua u =√t. Depois obterá umaintegral do tipo ∫ u senu du, que deverá ser re-solvida integrando por partes.R: 2 sen(√x)− 2√x cos(

√x) + C.(f) Veja té ni a do Exemplo na página 160(integrar por partes duas vezes seguidas).R: 2/5 e2 x cos(x) + 1/5 e2 x sen(x) +C(g) Tome u = log x. Logo, du = dx/x.Como x = eu, dx = eu du. Portanto temos queintegrar ∫ eu sen(u) du. Veja té ni a do Exem-plo na página 160 (integrar por partes duas vezesseguidas).

R :x (sen (log (x))− cos (log (x)))

2+ C(h) Substitua u = 3

√s. Depois obterá umaintegral do tipo ∫ ueu du, que deverá ser resolvidaintegrando por partes.R: 2 (3

√s− 1) e3

√s

9+ C.(i) Substitua u = ex. Vai obter ∫ du

1 + u2=

arctan u.R: arctan(ex) + C.Problema 8. (a) Primitiva: −e−x2

2. R: 1− 1/e

2.(b) Primitiva: −(3s + 1)e−3s

9. R: 1− 4e−3

9.( ) Primitiva: − 1

2(log x)2. R: 1/2.(d) Primitiva: −2 se−1/2 s − 4 e−1/2 s; R: 4.(e) Primitiva: 2(ex + 1)3/2

3. R: (16−4

√2)/3.(f) Primitiva: log(1 + x2)

2. R: log(2)/2.(g) Primitiva: −2(1 + 1/x)3/2

3. R: 37

24

√2.Problema 9. (a) Separe na integral de 0 até 2 de

x2(2− x) e de 2 até 4 de x2(x− 2). R: 24.(b) Separe na integral de 1/2 até 1 de − log se de 1 até 2 de log s. R: (3 log(2)− 1)/2.( ) Note que ey − 1 > 0 se y > 0. Logoes−1 − 1 > 0 se s − 1 > 0 e aso ontrário seránegativo. Assim al ule −2 até 1 ∫ 1

−21−es−1 ds, uja primitiva é es−1 − s e some om ∫ 2

1es−1 −

1 ds, uja primitiva é s− es−1 − s. R: e+ e−3.Problema 10. (a) Integrando obtemos quey(x) =

4x3/2 + 6√x

3+ C. Como queremos que

y(1) = 10/3 + C = 0, C = −10/3. Assim,y(x) =

4x3/2 + 6√x

3− 10

3.(b) Substitua u = x2 + 1. Vamos obter

y(x) =ex

2+1

2+ C. Como queremos que y(1) =

e2

2+ C = e2, C =

e2

2. Assim, y(x) = ex

2+1

2+

e2

2.Problema 11. (a) A primitiva é 4x3/4

3. R: 32/3.(b) Tome u = log x e faça a substituição. Aprimitiva é − 1

2 log2 x. R. 1/2( ) Tome u = 3 − 2ex e faça a substituição.A primitiva é −3− 2ex

2. R: log 3

2.Problema 12. (a) Note que trata-se de um limitedo tipo 0 vezes innito. Assim, es revendo omoo quo iente da integral por ex2 podemos apli arL'Hospital. Derivando a integral om o TFC ob-temos que o limite é igual ao limite

limx→+∞

log(x9 + 3)

2xex2.Colo ando 1

2xem evidên ia e apli ando L'Hospitalmais uma vez vamos obter o limitelim

x→+∞9x8

(x9 + 3)2xex2.Agora omo exponen ial vai mais rápido para in-nito que polinmio (ou apli ando L'Hospital umas8 vezes mais), on luímos que o limite vale 0.(b) Derivando os dois lados, utilizando o TFC,obtemos que e−xf ′(x) = 3 ou f ′(x) = 3ex. As-sim, integrando, obtemos que f(x) = 3ex + C.Como f(0) = 1 = 3 + C, C = −2. Logo,

f(x) = 3ex − 2.2.5.3 Extras da p.40Extra 1.Aplique LH e o TFC para transformarem limx→+∞

f(x)

1= K.R: O limite é K.Extra 2.Pelo TFC, Si′(x) = sen(x)

x. A derivadaé zero em x = kπ om k ∈ Z∗ (sem o zero, poispelo limite fundamental, Si′(0) = 1). Para saberse é máximo ou mínimo temos que ver o sinal daderivada antes e depois destes pontos. Para x >

Page 96: Sumario´ - labMA/UFRJmcabral/livros/livro-calculo/... · 2019. 6. 12. · Sumario´ 1 Enunciados dos Exercícios 1.1 Exercícios de Limite. 1 1.1.1 Exercícios de Fixação. 1 1.1.2

92 CAPÍTULO 2. RESPOSTAS DOS EXERCÍCIOS0 vamos ter os máximos lo ais em x = 2kπ + πpara k ∈ N. Para x < 0 vamos ter os máximoslo ais em x = 2kπ para k ∈ N.Extra 3. (a) Pelo TFC, f ′(x) = log(ex + x −1)ex

2 . Logo, f ′(1) = e. Como f(1) = 0 (pois∫ 1

1(· · · ) = 0), a reta tangente é y = e(x− 1).(b) Pelo TFC, h′(x) = ex

x2 + 1. Logo, h′(2) =

e2

5. Como h(2) = 7, a reta tangente é y − 7 =

e2

5(x− 2).Extra 4. Sabemos que a a eleração a(t) é igual aderivada da velo idade v(t) que é igual a derivadada posição x(t). Assim, v′ = a, x′ = v. Como

v′(t) = a(t) = Aω2 cos(ωt),v(t) =

a(t) dt =

Aω2 cos(ωt) dt =

= Aω sen(ωt) + C.Como v(0) = 0 = Aω sen(ω0) +C = C, C = 0.Como x′(t) = v(t),x(t) =

v(t) dt =

Aω sen(ωt) dt =

= −A cos(ωt) + C.Como x(0) = −A cos(ω0) + C = 0 = −A + C.Assim, C = A. Logo, x(t) = −A cos(ωt) +A.Extra 5. Seja h(y) =

∫ y

0

sen(t)

t2dt. Note que

f(s) = h(s2)−h(s) (porque?). Pelo TFC, h′(y) =sen(y)

y2. Assim, pela regra da adeia. f ′(s) =

2sh′(s2)− h′(s) =2s sen(s2)

s4− sen(s2)

s4.Extra 6.Denindo h(x) =

∫ x

0sen(t2) dt, que-remos determinar a derivada de g(x) = h(e2x).Pelo Teorema 5.2 da p.150 (TFC), h′(x) = sen(x2).Utilizando a regra da adeia, a derivada de g(x) =

h(e2x) é g′(x) = h′(e2x)(e2x)′ = sen((

e2x)2)2e2x.Logo, g′(x) = sen(e4 x)2e2x.Extra 7.Tome u = 2x + 1. Então du = 2dx.Logo quando x = 0, u = 1; quando x = −1, u =

−1. Logo, ∫ 0

−1f(2x + 1) dx =

∫ 1

−1f(u) du/2.Agora, pelas propriedades da integral,

∫ 1

−1=

∫ 3

−1−∫ 3

1.

Assim ∫ 1

−1f(u) du/2 = (1/2)(7− 3) = 4/2 = 2.Extra 8. (a) Integrando obtemos que

y(θ) =sen(5x)

5+ 3x+ C. Como y(π) = 15π +

C = 5π, C = −10π. Logo, y(θ) =sen(5x)

5+

3x− 10π.(b) Integrando obtemos quey(x) =

log(2x+ 1)

2+C. Como y(0) = 0+C =

3, C = −3. Logo, y(x) = log(2x+ 1)

2− 3.Extra 9.Temos que y′(x) = √

x+ 1 e que y(0) =1. Integrando obtemos quey(x) =

2(x+ 1)3/2

3+ C. Como y(0) = 2/3 +

C = 1, C = 1/3. Logo, y(x) = 2(x+ 1)3/2

3+

1/3.Extra 10. (a) Como x2−3x+2 = (x−1)(x−2),separe em três integrais: ∫ 1

0(x2 − 3x + 2) dx =

5/6, ∫ 2

1(−x2+3x−2) dx = 1/6 e ∫ 4

2(x2−3x+

2) dx = 14/3. R: 5/6 + 1/6 + 14/3 = 17/3.(b) Separe na integral de ∫ 0

−3

√1− x dx =

14/3 e ∫ 3

0

√1 + x dx = 14/3. R: 14/3+14/3 =

28/3.( ) Como x2 − 2x = x(x − 2), separe naintegral de ∫ 0

−2x(x2 − 2x) dx = −28/3 mais

∫ 2

0x(2x− x2) = 4/3dx R: −28/3 + 4/3 = −8.Extra 11. (a) 1/3x3 log(x)− 1/9x3 + C.(b) 2 (x

3

2 log x

3− 2x

3

2

9

)

+ C.( ) x2(2(log x)2 − 2 log x+ 1)

4+ C.(d) Faça a substituição u = cos x + senx,

du = (− sen x+ cosx) dx.R: − log(senx+ cos x)(e) − 1

ex − 1+ C.(f) e3 sen(x)+4

3+ C.(g) sen(e3 + 3) + C.(h) Como ax = exp(x log a), por substitui-ção, ∫ ax dx =ax

log a+ C.

Page 97: Sumario´ - labMA/UFRJmcabral/livros/livro-calculo/... · 2019. 6. 12. · Sumario´ 1 Enunciados dos Exercícios 1.1 Exercícios de Limite. 1 1.1.1 Exercícios de Fixação. 1 1.1.2

2.5. INTEGRAL 93Extra 12. (a) Primitiva −e1/x. R: e2 − e.(b) Primitiva (x2 − 1)ex2

2. R: log(2)− 1.( ) Primitiva 2√x(log x−2). R: 8 log(2)−4.(d) Primitiva 2(sen θ)3/2

3. R: 2/3.(e) Primitiva−2 (1−y)

52

5 + 4 (1−y)32

3 −2√1− yR: 16/15.(f) Primitiva −2 cos(

√x). R: −4.(g) Primitiva log(ex + 4). R: log(7/6).(h) Primitiva √

t2 + 1. R: √2− 1.Extra 13. (a) A primitiva é −1 + log x

x(integra-ção por partes). R: 1.(b) A primitiva é p− 1 + log x

(1− p)xp−1(integraçãopor partes). R: 1/(p − 1)2.( ) A primitiva é 1

4− x. R: 1

2.(d) Por substituição u =

√1− e−x, du =

e−x dx. A primitiva é: 2√1− e−x. R: 2.2.5.4 ⋆Problemas (Integração e Subs-tituição Trigonométri a) da p.42Problema 1. (a) Pela té ni a,cos x cos x = 1/2(cos 2x+ 1).Assim,

cos3 x = 1/2 cos x+ 1/2 cos x cos 2x =

= 1/2 cos x+ 1/4(cos 3x+ cosx) =

= 3/4 cos x+ 1/4 cos 3x.R: 3/4 sen x+ 1/12 sen 3x+ C.(b) Pela té ni a, cos23x = 1/2(cos 6x + 1).Assim, cos23x cos 5x = 1/2 cos 6x cos 5x+1/2 cos 5x.Agora, cos 6x cos 5x = 1/2(cos 11x+cos x). Logo,cos23x cos 5x = 1/4(cos 11x+cos x)+1/2 cos 5x.R: 1/44 sen 11x+1/4 sen x+1/10 sen 5x+C ( ) Pela té ni a do texto,cos x senx = 1/2(sen 2x+ sen 0) = 1/2 sen 2x.R: −cos 2x

4+C.Outra solução: Tome u = senx, du = cos x dx.Assim a integral se transforma em ∫

udu =u2

2.R: sen2x

2+ C.Deixo o leitor veri ar que as primitivas di-ferem por uma onstante. Ainda outra respostapossível é: −cos2x

2+ C.

(d) Como sen2x = 1/2(1− cos 2x), sen4x =1/4(1− 2 cos 2x+cos2 2x). Também temos quecos2 2x = 1/2(cos 4x + 1). Assim, sen4x =−1/2 cos 2x+ 1/8 cos 4x+ 3/8.R: −1/4 sen 2x+1/32 sen 4x+3/8x+C(e) Pela té ni a,cos(4x) sen(3x) = 1/2(sen 7x− senx)R: −1/14 cos 7x+ 1/2 cos x+ C.Problema 2. (a) Fazendo substituição x = sen t,dx = cos t dt obtemos

1

cos2 tdt =

sec2 t dt = tan t.Como x = sen t, tan t = x/√1− x2.R: x√

1− x2+ C.(b) Fazendo substituição x = sen t,

dx = cos t dt:∫

sen2t cos t dt

cos t=

sen2t dt =

= (integral trigonométri a) t

2− sen t cos t

2.Como cos t =

√1− x2 e t = arcsenx:R: arcsen x2

− x√1− x2

2+ C.( ) Completando o quadrado obtemos que

x2 +2x+2 = (x+1)2 + 1. Fazendo u = x+1,du = dx obtemos ∫ du

(u2 + 1)3/2. Fazendo u =

tan t, du = sec2 t dt. Assim, (u2+1)3/2 = sec3 t.Substituindo na integral obtemos∫

sec2 t

sec3 tdt =

1

sec tdt =

cos dt = sen t.Como u = tan t, sen t = u/√u2 + 1. Passandopara x a resposta é x+ 1√

x2 + 2x+ 2.(d) Tome x = sec t. Então dx = sec t tan tdte √

x2 − 1 = tan t. Assim amos om∫

tan2t dt =

(sec2 t− 1) dt = tan t− t (deri-vada de tangente é se ante ao quadrado). Rees- revendo em x:R:√x2 − 1−arctan(√x2 − 1)+C. Ou-tras respostas: √x2 − 1− acos(1/x) ou√

x2 − 1− asec(x).(e) Tome x = sec t. Então dx = sec t tan tdte√x2 − 1 = tan t. Assim amos om ∫ dt

sec t=

cos t dt = sen t. Como cos t = 1/x, sen t =√x2 − 1/x.

Page 98: Sumario´ - labMA/UFRJmcabral/livros/livro-calculo/... · 2019. 6. 12. · Sumario´ 1 Enunciados dos Exercícios 1.1 Exercícios de Limite. 1 1.1.1 Exercícios de Fixação. 1 1.1.2

94 CAPÍTULO 2. RESPOSTAS DOS EXERCÍCIOSR: √x2 − 1

x+ C.(f) Fazendo x = 5 tan t, dx = 5 sec2 t dt. As-sim,√x2 + 25 = 5 sec t. Substituindo na integralobtemos:

5 sec2 t dt

25 tan2 t (5 sec t)=

sec t dt

25 tan2 t=

cos t dt

25 sen2 t.Fazendo u = sen t, du = cos t dt obtemos

cos t dt

25 sen2 t=

du

25u2= − 1

25u= − 1

25 sen t.Como tan t = x/5, sen t = x/

√x2 + 25 e por-tanto: R: −√

x2 + 25

25x+ C.Problema 3. (a) Fazendo substituição x = 2 sen t,

dx = 2cos t dt: ∫ 2 cos t dt

2 cos t=

dt = t =

arcsen(x/2) + C.Poderia ser feita diretamente: ∫ dx√4− x2

=∫

dx

2√

1− (x/2)2. Tomando y = x/2 e sabendoque a derivada de arcsen y é 1

1− y2, obtemos

dy√

1− y2= arcsen y = arcsen(x/2) + C.(b) Neste aso o mais fá il é fazer a substi-tuição u = 1−x2 e du = −2x dx, transformandoem −1

2

du

u1/2= −u1/2 = −

1− x2 + C.Por substituição trigonométri a, fazendo subs-tituição x = sen t, dx = cos t dt:∫

sent cos t

cos tdt =

sent dt =

− cos t = −√

1− x2 + C.Colo ando os limites de integração obtemosR: 1.( ) Tome x = sec t. Então dx = sec t tan tdte√x2 − 1 = tan t. Assim amos om ∫ sec t dt =

log | sec t + tan t| (veja p.44 para esta integral).Tro ando x por t:R: log |x+√x2 − 1|.(d) Fazendo substituição x = sen t, dx =

cos t dt:∫

sen3t cos t

cos3 tdt =

sen3t

cos2 tdt =

=

(1− cos2 t) sen t

cos2 tdt.

Agora tome u = cos t, du = − sen t dt e obte-nha ∫ u2 − 1

u2du = u+

1

u. Logo a integral vale

cos t+1

cos t. Como cos t =

√1− x2 obtemos:R: √1− x2 +

1√1− x2

+ C.(e) Tomando x = 4 senu, √16 − x2 = 4cos u,dx = 4cos u du. Assim, após substituição obte-mos a integral 16∫ cos2 u du Por integral trigo-nométri a, sua primitiva é 4 sen(2u) + 8u. Comox = 4 sen u, os limites de integração são u = π/4(x = 4 sen(π/4) = 2

√2) até u = π/2 (x =

4 sen(π/2) = 4). Cal ulando16

∫ π/2

π/4cos2 u du = 4 sen(2u) + 8u|π/2π/4 = 2π−4.R: 2π − 4.2.5.5 ⋆Problemas (Integração de Fun-ções Ra ionais) da p.43Problema 1.Fatorando denominador: x2 − 1 =

(x+1)(x−1). Cal ulando oe ientes: 2

x2 − 1=

1

x− 1+

−1

x+ 1. Primitiva: −1/(x+1)+1/(x−1).O resultado: log(5/3).R: log(5/3).Problema 2. (a) Fatorando denominador: x2 −

8x+7 = (x−1)(x−7). Cal ulando oe ientes:2x− 3

x2 − 8x+ 7=

1/6

x− 1+

11/6

x− 7.R: 1/6 log |x− 1|+11/6 log |x− 7|+C.(b) Fatorando denominador: x3−x2 = x2(x−

1). Cal ulando oe ientes: 2x− 3

x3 − x2=

1

x+

3

x2+

−1

x− 1. R: − log |x− 1|+ log |x| − 3/x+ C.( ) Fatorando denominador: x3+4x = x(x2+

4). Cal ulando oe ientes:x+ 4

x3 + 4x=

1− x

x2 + 4+

1

x.R: 1/2 arctan(x/2)+log |x|−1/2 log |x2+

4|+ C.(d) Fatorando denominador: x2 + 8x + 7 =(x+ 1)(x + 7). Cal ulando oe ientes:

6

x2 + 8x+ 7=

1

x+ 1+

−1

x+ 7R: log ∣∣∣∣

x+ 1

x+ 7

+ C.

Page 99: Sumario´ - labMA/UFRJmcabral/livros/livro-calculo/... · 2019. 6. 12. · Sumario´ 1 Enunciados dos Exercícios 1.1 Exercícios de Limite. 1 1.1.1 Exercícios de Fixação. 1 1.1.2

2.5. INTEGRAL 95(e) Fatorando denominador: x4−4x2 = x2(x−2)(x+ 2). Cal ulando oe ientes:x2 + 4x− 2

x4 − 4x2=

−1

x+

1/2

x2+

3/8

x− 2+

5/8

x+ 2.R: − log |x| − 1/(2x) + 5/8 log |x− 2|+

3/8 log |x+ 2|+ C.(f) Cal ulando oe ientes:2x2 + x+ 2

(x− 1)(x2 + 4)=

x+ 2

x2 + 4+

1

x− 1.R: 1/2 log |x2+4|+arctan(x/2)+log |x−

1|+ C.Problema 3.Completando o quadrado e olo- ando em evidên ia onstantes, vai apare er umaintegral do tipo ∫ dy

1 + y2= arctan y. Colo- ando os limites (arctan(±∞) = ±π/2) obtemosresposta 2π√

4c− b2.Problema 4. (a) 1

b− alog

(

x− b

x− a

).(b) log(x− a)− log(x)

a2+

1

ax.Problema 5.Dena D = aB− bA e X = ax2+

2bx+ c.(a) A

2alog |X|+ D

2a√∆

log

ax+ b−√∆

ax+ b+√∆

.(b) A

2alog |X|+ D

a√−∆

arctan

(

ax+ b√−∆

).( ) − D

a(ax+ b)+

A

alog |ax+ b|.2.5.6 Desaos da p.43Desao 1. (a) Basta omparar áreas (veja no grá- o de y = f(x) a integral da inversa.(b) Tomando f(x) = arcsenx, g(y) = sen y,

arcsen s ds = x arcsen x+cos(arcsen x)+af(a).Como cos(arcsen x) =√1− x2, a primitiva é

x arcsenx+√1− x2 + C.Tomando f(x) = log x, g(y) = ey, ∫ log s ds =

x log x− elog x + C = x log x− x+C.Desao 2.Derivando os dois lados (TFC), x3ex =(p′(x) + p(x))ex. Can elando ex (que é 6= 0):x3 = p′(x) + p(x). Dai deduzimos que p(x) temgrau 3. Vemos que p(x) = x3 + ax2 + bx + c.Agora p′(x)+p(x) = x3+(3+a)x2+(2a+b)x+b+c = x3+0x2+0x+0. Assim 3+a = 2a+b =b+ c = 0. Assim, a = −3, b = 6, c = −6.R: p(x) = x3 − 3x2 + 6x− 6.

Desao 3.Basta apli ar o TFC. Veja exer í ioanterior para o aso n = 3. Vamos fazer expli- itamente o (b) para n = 2. Derivando (TFC)obtemos (log x)2 = q(log x)+ q′(log x). Para tera igualdade q(y) = y2 + by + c, y2 = y2 + by +c + (2y + b) = y2 + (b + 2)y + b + c. Assim,b = −2, c = 2 e q(y) = y2 − 2y + 2, e por-tanto ∫ (log x)2 dx = xq(log x) = x((log x)2 −2 log x+ 2).Desao 4.Pelo TFC, eax sen bx = eax(A sen bx−Bb sen bx + B cos bx + Ab sen bx). Igualando os oe ientes que multipli am eax sen bx de adalado e que multipli am eax cos bx obtemos o sis-tema ( om in ógnitas A,B):

[

1 −bb 1

] [

AB

]

=

[

10

]Como seu determinante é 1 + b2 > 0, a soluçãoexiste e é úni a.Desao 5. pelo TFC deveríamos ter ex2

= (p′(x)+2xp(x))ex

2 . Assim 1 = p′(x) + 2xp(x). Terminesupondo grau de p igual a 3, 4, et .Desao 6.Note que1

(x2 + 1)m=

1

(x2 + 1)m−1− x2

(x2 + 1)m.O segundo termo da direita pode ser integradopor partes olo ando f(x) = x e

g′(x) =x

(x2 + 1)m. Para detalhes ver [Co, p.228.Note que I1 =

dy

(y2 + 1)= arctan y.Desao 8. (a) Podemos rees rever a integral:

(tan2)ksecn−1x sec x tan x dx =

=

(sec2 − 1)ksecn−1x sec x tan x dxAgora faça a mudança de variáveis u = secx eobtenha ∫ (u2 − 1)kun−1x du = P (u) para al-gum polinmio P . Assim, retomando a variávelx obtemos a resposta.(b) De fato∫

(tan2)ksecnx dx =

(sec2 − 1)ksecnx, dx =

=

P (sec x) dx.Mais expli itamente, Q(y) = (y2 − 1)kyn.

Page 100: Sumario´ - labMA/UFRJmcabral/livros/livro-calculo/... · 2019. 6. 12. · Sumario´ 1 Enunciados dos Exercícios 1.1 Exercícios de Limite. 1 1.1.1 Exercícios de Fixação. 1 1.1.2

96 CAPÍTULO 2. RESPOSTAS DOS EXERCÍCIOSDesao 9.Ver [Co para (a)-( ).(d) Apli ando ( ) obtemos que ∫ dx

1 + cos x=

2

(1 + t2)(1 + (1− t2)/(1 + t2))dt =

dt =

t = tan(x/2).(e) Apli ando ( ) obtemos que ∫ dx

1 + senx=

2

(t+ 1)2dt = − 2

t+ 1= − 2

tan(x/2) + 1.(f) Apli ando ( ) obtemos que ∫ dx

senx=

dt

t= log(tan(x/2)).(g) Note que sec x tan2x =

sen2x

cos3 x. Apli- ando ( ) obtemos que

secx tan2x dx =

8t2

(1− t2)3dt(veja no Maxima a resposta: são muitos termos!).(h) Note que sec 2x tan 3x =

sen(3x)

cos(2x) cos(3x).Agora podemos expandir os termos usando as fór-mulas do seno/ osseno da soma. Por exemplo:

sen(3x) = 3cos2x senx− sen3x. De forma aná-loga (obtive no Maxima omtrigexpand( os(2*x)* os(3*x)):cos(2x) cos(3x) =

= 3 cos x sen4x− 4 cos3 x sen2x+ cos5 x.Denindo Q(c, s) =3c2s− s3

3cs4 − 4c3s2 + c5te-mos que

sec 2x tan 3x dx =

Q(cos x, senx) dx =

=

Q

(

1− t2

1 + t2,

2t

1 + t2

)

2 dt

1 + t2.Desao 10.Veja na Internet artigo da Wikipedia:Sophomore's dream.Desao 11.Dividimos em 2 integrais. A partefá il é:

∫ 1

0e−y2 dy ≤ (1− 0)e−0 = 1. Agora para outraparte, se y > 1 então e−y2 < ye−y2 . Comopodemos integrar expli itamente ye−y2 em 1 até

+∞, a integral onverge.Desao 12. (b) Como sen(x)/x é uma função ontínua perto de zero se for redenida em zero,sen(x)/x é integral em [0, 1]. Para outra parte,

note que | cos(x)/x2| ≤ 1/x2 e esta função éintegrável em [1,+∞].Desao 13.Pelo TFC, f ′(x) = f(x). Pela pro-priedade de f , f(0) = 0. Assim a solução éf(x) = 0 para todo x ∈ R.Desao 14.Use l'Hospital para determinarlim

y→−1f(y) = ln(b)− ln(a) ([Ap p.309).Desao 15.Desao 16.Por ontradição, se f não é nula,será diferente de zero em algum ponto c ∈ [a, b].Sem perda de generalidade, suponha f(c) > 0.Por ontinuidade f > 0 em algum intervalo [x, y] ontendo c. Logo ∫ y

xf(s) ds > 0. Contradição!Desao 19.Pense em representação polar ( oor-denadas r e θ) do número (a, b) ∈ R2.R: 1

rlog

tanx+ θ

2

, r =√a2 + b2.2.6 Apli ações da Integral2.6.1 Exer. de Fixação da p.47Exer í io 1. (a) Uma função é a translação daoutro por 2 unidades. Assim a área é igual a

∫ 5

0(ex + 2− ex) dx =

∫ 5

02 dx = 2 · 5 = 10.(b) Está área é igual ∫ π/2

−π/2cos(x) dx = 2.Exer í io 2. (a) ∫ 2

1(g(x) − f(x)) dx.(b) ∫ 1

−1(f(x)−g(x)) dx+

∫ 2

1(g(x)−f(x)) dx.Exer í io 3. (a) ∫ 1

−1π[f(x)]2 dx.(b) ∫ 2

1π[g(x)]2 dx−

∫ 2

1π[f(x)]2 dx.( ) ∫ 6

3π[g−1(y)]2 dy −

∫ 2

1π[f−1(y)]2 dy.Exer í io 4.∫ 3

−2(g(y) − f(y)) dy.Exer í io 5.Pelo Teorema 29 da p.187 o volumede Ω é ∫ 4

−2A(s) ds.007.o t.2011

Page 101: Sumario´ - labMA/UFRJmcabral/livros/livro-calculo/... · 2019. 6. 12. · Sumario´ 1 Enunciados dos Exercícios 1.1 Exercícios de Limite. 1 1.1.1 Exercícios de Fixação. 1 1.1.2

2.6. APLICAÇÕES DA INTEGRAL 97Exer í io 6.Pela denição de média,K =

1

b− a

∫ b

ag(x) dx.Pela monotoni idade da integral (Lema 9 da p.149), omo g(x) ≤ 5,

∫ b

ag(x) dx ≤

∫ b

a5 dx = 5(b− a).Assim, K ≤ 1

b− a5(b − a) = 5. De formaanáloga, pela monotoni idade da integral, omo

g(x) ≥ −4,∫ b

ag(x) dx ≥

∫ b

a−4 dx = −4(b− a).Assim, K ≥ 1

b− a(−4)(b − a) = −4.2.6.2 Problemas da p.48Problema 1. (a) A interseção o orre quando y =

x2 = x− x2, ou seja, quando 2x2 − x = x(2x−1) = 0. Assim a interseção é em x = 0 e x = 1/2.Logo a área é igual a ∫ 1/2

0(x − x2 − x2) dx =

x2

2− 2x3

3

1/2

0

=1

24(b) A interseção o orre é quando cos x =senx, que o orrerá dentro de um i lo do seno([0, 2π]) em π/4 e π + π/4 = 5π/4. Assim aárea é igual a ∫ 5π/4

π/4(senx− cos x) dx =

√8.Problema 2. (a) A interseção de y = x3 = x+6é em x = 2 e y = 8. A interseção de 2y = x e

y = x+ 6 é em (−4, 2). Assim o esboço é:

x

y

y − x = 6

y − x3 = 0

2y + x = 02−4

2

8

Assim a área é:∫ 0

−4((x+6)−(−x/2)) dx+

∫ 2

0((x+6)−(x3)) dx.Como ∫ 0

−4((x+6)−(−x/2)) dx = 12 e ∫ 2

0((x+

6)− (x3)) dx = 10, a área é 22.(b) Para fa ilitar, o primeiro passo é tro arx om y e resolver o problema: Cal ule a regiãodelimitada por por x2 = 2y + 4 e por x = y − 2.Assim, y =

x2

2− 2 e y = x + 2. A interseçãoo orrerá quando y =

x2

2− 2 = x + 2, isto é se

x = −2 ou se x = 4. Assim a área é igual a∫ 4

−2((x+ 2)− (x2/2− 2)) dx = 18.

x

y

y =x2

2− 2

y = x+ 2

−2 4Resolvendo o problema original (y2 = 2x+ 4e y = x − 2) e integrando em x teríamos quees rever omo soma de duas integrais (verique):∫ 0

−22√2x+ 4 dx+

∫ 6

0(√2x+ 4− (x− 2)) dx =

=16

3+

38

3= 18.( ) Fazendo o esboço observamos que a in-terseção o orrerá em x = 0 e em x = 1. Assim aárea é igual a ∫ 1

0(1−√

x) dx =1

3. Outra possi-bilidade é integrar em y. Como y =√x, x = y2.Assim, a área é ∫ 1

0y2 dy =

1

3.x

y

y =√x

y = 1

x = 01Problema 3. Sua área será determinada por

∫ 4

−4(4+

16− 4x2−4) dx−∫

√2

−√2(6−x2−4) dx.

Page 102: Sumario´ - labMA/UFRJmcabral/livros/livro-calculo/... · 2019. 6. 12. · Sumario´ 1 Enunciados dos Exercícios 1.1 Exercícios de Limite. 1 1.1.1 Exercícios de Fixação. 1 1.1.2

98 CAPÍTULO 2. RESPOSTAS DOS EXERCÍCIOSA primeira integral é igual a metade da área do ír ulo de raio 4: 8π. R: 8π − 8

√2

3.Problema 4.Note que a região é limitada supe-riormente por y = x+1. Assim, rodando no eixo

x o volume será ∫ 1

0π(x+1)2 dx =

3. O sólidoobtido será um tron o de one.Girando em torno do eixo y vamos obter um ilindro de 1 e altura 2 menos o sólido obtidogirando x = y − 1 (já que y = x + 1) para y ∈

[1, 2]. O ilindro possui volume 2π (πr2h, omr = 1 e h = 2). Devemos subtrair ∫ 2

1π(y −

1)2 dy =π

3. Assim o volume é 2π − π

3=

3.Problema 5. (a) Primeiro o esboço.

x

y

y = 3√x

Ω

y = 2

8Sua área é igual a∫ 8

0(2− 3

√x) dx = 4.O volume será al ulado omo a diferença entredois volumes:

π

∫ 8

022 dx−π

∫ 8

0( 3√x)2 dx = 32π−π

96

5=

64π

5.(b) Primeiro o esboço.

x

y

Ωy = e−x

x = 1Sua área é igual a∫ +∞

1e−x dx =

1

e.O volume é igual a

π

∫ +∞

1(e−x)2 dxπ

∫ +∞

1(e−2x) dx =

π

2e2.

Problema 6.Primeiro o esboço:x

y

x = y2x = y

1

1

x = −1

y = −1

(a) Note que y =√x. Como a rotação é emtorno de y = −1, o raio maior é 1+√

x e o menoré 1 + x, ao invés de √x e x se fosse rotação emtorno de y = 0 (eixo x). O volume será dado peladiferença de volumes:

π

∫ 1

0(√x+ 1)2 dx− π

∫ 1

0(x+ 1)2 dx =

17π

6− 7π

3=

π

2.(b) De forma análoga, o raio maior é y+1 e omenor y2+1. O volume será dado pela diferençade volumes:

π

∫ 1

0(y + 1)2 dy − π

∫ 1

0(y2 + 1)2 dy =

3− 28π

15=

15.Problema 7.Note que A = (1, e) pois está na urva y = ex

2 , e portanto, y = e12

= e1 = e.Por outro lado B está parábola. Como y = 0,x = 2. Assim B = (2, 0). Assim a equação dareta L (que passa em A e B) é y = −ex + 2e,ou x = 2 − y/e. A função y = ex

2 intersepta oeixo y em y = 1.O volume será igual ao volume do tron o de one obtido ao girar a reta L para y ∈ [0, e] menoso volume ao girar y = ex2 para y ∈ [1, e] e menoso volume ao girar a parábola para y ∈ [0, 1].Invertendo as funções, omo y = ex

2 , log y =x2, x =

√log y. Assim o integrando será x2 =

log y. Como 4y = (x − 2)2, e na região (vejagura) 2√y = x − 2 ≤ 0, √(x− 2)2 = −(x −

2) = 2 − x. Assim, 2√y = 2 − x. Logo, x =2− 2

√y.

Page 103: Sumario´ - labMA/UFRJmcabral/livros/livro-calculo/... · 2019. 6. 12. · Sumario´ 1 Enunciados dos Exercícios 1.1 Exercícios de Limite. 1 1.1.1 Exercícios de Fixação. 1 1.1.2

2.6. APLICAÇÕES DA INTEGRAL 99Assim o volume é igual aπ

∫ e

0(2− y/e)2 dy−

− π

∫ 1

0(2− 2

√y)2 dy − π

∫ e

1log y dy.Problema 8.A primeira oisa a ser observada éque a resposta NO é o volume da esfera menos ovolume do ilindro de raio a. Isto porque o nal do ilindro retirado pelo furo é arredondado (estána superfí ie da esfera).A esfera é o sólido de revolução de y = f(x) =√

r2 − x2 em torno do eixo x. Como o bura otem raio a (veja gura), o valor x = k para quef(k) = a =

r2 − x20 será k =√r2 − a2.

x

y

aa

y =√r2 − x2

k−kAssim o volume será dado porπ

∫ k

−k(√

r2 − x2)2 dx = 2kπr2 − 2πk3

3,onde k =

√r2 − a2.Problema 9.Como y = ±

√r2 − x2, o lado doquadrado para ada x é 2

√r2 − x2. A área de ada orte A(x) = 4(r2 − x2). Assim, o volumeé

∫ r

−r4(r2 − x2) dx =

16

3r3.Problema 10.Faça a gura e observe que a in-terseção é em (1, 1) e (0, 0). O lado do qua-drado para ada x é x−x2. A área de ada orte

A(x) = (x− x2)2. Assim, o volume é∫ 1

0(x− x2)2 dx =

1

30.Problema 11. (a) k2

3.(b) 2

π.

2.6.3 Extras da p.50Extra 1. (a) Começamos om o esboço:x

y

y = 1/x

y = x2

1

x = 2x = −1

y = −2Assim a área será:∫ 1

−1(x2 − (−2)) dx +

∫ 2

1(1/x− (−2)) dx =

14

3+ log(2) + 2 =

20

3+ log 2.(b) Note que as raízes da equação do segundograu são 0 e 2π. O esboço é:

x

y

y = 6πx− 3x2

y = cos(x)− 12π

Assim a área é∫ 2π

0(6πx− 3x2 − (cos(x)− 1)) dx = 4π3 + 2π.( ) A interseção o orrerá se y = x2 = 8−x2,isto é, quando 2x2 = 8, em x = ±2.Assim a área é ∫ 2

−2((8− x2)− x2) dx =

64

3.

x

y y = x2

y = 1− x2Extra 2. (a) Começamos pelo esboço.

Page 104: Sumario´ - labMA/UFRJmcabral/livros/livro-calculo/... · 2019. 6. 12. · Sumario´ 1 Enunciados dos Exercícios 1.1 Exercícios de Limite. 1 1.1.1 Exercícios de Fixação. 1 1.1.2

100 CAPÍTULO 2. RESPOSTAS DOS EXERCÍCIOSx

yy = x3 − x

y = sen(πx)

1−1

Assim a área será, por simetria, o dobro daárea para x ∈ [0, 1], ou seja,2

∫ 1

0(sen(πx)− (x3 − x)) dx =

π + 8

2π=

1

2+

4

π.(b) Começamos pelo esboço.

x

yy = x3 − 3x2 + 2x

y = 3x− x2 − 21−1 2

Assim al ulamos a área somando duas inte-grais:∫ 1

−1(x3 − 3x2 + 2x− (3x− x2 − 2)) dx+

∫ 2

1(3x− x2 − 2− (x3 − 3x2 + 2x)) dx =

8

3+

5

12=

37

12.Extra 3.Note queWn = π

∫ 1

0(xn)2 dx =

π

2n + 1eVn = π

∫ 1

0(1 − (y1/n)2) dx =

n+ 2. Logo,

limn→+∞

Vn

Wn= lim

n→+∞2(2n + 1)

n+ 2= 4.

Extra 4.f(ab) =

∫ ab

1

dx

x=

∫ a

1

dx

x+

∫ ab

a

dx

x.Tome u = x/a. Assim, du = dx/a. Mudandovariável vamos obter ∫ b

1du/u = f(b).Extra 5. (a) duas parábolas, uma om on avi-dade para baixo, outra para ima. Ambas se in-terse tam, e possuem omo raízes ±√

c.(b) ∫ √c

−√c(c−x2−2x2+2c) dx = 4c3/2 = 25.Logo, c = 4.Extra 6. (a) O volume será dado por

π

∫ +∞

e

log(x)− 1

x2dx = − log x

x

+∞

e

e.(b) O volume será dado por:

∫ e2

1log2 x dx =

= x(log2 x− 2 log x+ 2)∣

e2

1= 2e2 − 2Extra 7. (a) Quando x = 1, y = 1/9. Noteque x =

1/y − 5 na região. Assim, o volumerodando em x será dado porπ

∫ 2

0

dx

(x2 + 5)2.O volume rodando em y é

π

∫ 1/9

0(2)2 dy + π

∫ 1/5

1/9(1/y − 5) dy.(b) A interseção é em (0, 0) e (4, 2). Ambosvolumes serão determinados subtraindo volumes.O volume rodando em x será dado por

π

∫ 4

0x dx− π

∫ 4

0

x2

4dx.Invertendo as relações obtemos x = y2 e x = 2y.O volume rodando em y será dado por

π

∫ 2

04y2 dy − π

∫ 2

0y4 dy.( ) Note que embora as raízes de (6−x)2 = xsejam x = 4 e x = 9, a úni a interseção (soluçõesde 6 − x =

√x) é em x = 4, y = 2 (porque

Page 105: Sumario´ - labMA/UFRJmcabral/livros/livro-calculo/... · 2019. 6. 12. · Sumario´ 1 Enunciados dos Exercícios 1.1 Exercícios de Limite. 1 1.1.1 Exercícios de Fixação. 1 1.1.2

2.6. APLICAÇÕES DA INTEGRAL 101des artei x = 9?). A reta y = 6− x intersepta oeixo x em x = 6O volume rodando em x será dado porπ

∫ 4

0x dx+ π

∫ 6

4(6− x)2dx.Invertendo as relações obtemos que x = 6 − y e

x = y2. O volume rodando em y será dado porπ

∫ 2

0(6− y)2 dy − π

∫ 2

0y4dy.Extra 8.Note que y(x) = ±2

√x. Logo, para ada x, o lado do quadrado é 4√x. A área de ada orte A(x) = 16x. Assim, o volume é

∫ 9

016x dx = 648.Extra 9.Veja artigo Cavalieri's_prin iple naWikipedia.Extra 10.De forma análoga a um exer í io ante-rior onde determinamos o volume de uma esfera om um furo. Por Pitágoras, r2 = a2 + (r− h)2.Rodando a gura em 90 graus, pensando naesfera omo o sólido de revolução de y = f(x) =√

r2 − x2 em torno do eixo x e denindo k =r − h, o volume da alota seráπ

∫ r

k(√

r2 − x2)2 dx =π

3(2r3 − 3kr2 + k3).Substituindo k = r− h, obtemos que o volume é

π(h2r − h3/3). Com mais alguma manipulaçãotambém obtemos que o volume é πh

6(3a2 + h2).Outra solução é utilizando somente o prin í-pio de Cavalieri e a ralação entre volume de one, ilindro e esfera. É solução elementar, que podeser feita no Ensino médio.Extra 11.O volume total é

π

∫ 2

0(x+ 1) dx = 4π.O volume até x = a é

π

∫ a

0(x+ 1) dx = π

a2 + 2a

2.Igualando a2 + 2a

2=

1

24, obtemos que a =

√5−

1 (a solução no intervalo [0, 1], pois a outra−√5−

1 está fora.

Extra 12.Observe que y(x) = ±b√

1− (x/a)2 om x ∈ [−a, a]. Assim o volume éπ

∫ a

−ab2(1− (x/a)2) dx =

4πab2

3.Extra 13.Por frações par iais, dx

x(x2 + 1)=

1

x−

x

x2 + 1. Assim ∫ dx

x(x2 + 1)= log x− log(x2 + 1)

2.Colo ando limites de integração obtemos resposta.R: π log(2)/2.2.6.4 ⋆Problemas (Comprimento deCurvas no Plano) da p.52Problema 1. (a) Cal ulando 1+[f ′(x)]2 =

x2

x2 − 1.A integral que determina o omprimento possuiprimitiva √

x2 − 1.R: √3.(b) Cal ulando, 1 + [f ′(x)]2 =16

16− x2. Aintegral que determina o omprimente possui pri-mitiva 4 arcsen(x/4).R: 2π.( ) Vamos ter que al ular ∫ √

1 + x2

xdx.Fazendo substituição hiperbóli a, obtemos a pri-mitiva √

1 + x2 − arcsenh(1/x).R: √5−√2+arcsenh(1)− arcsenh(1/2).(d) Como g′(x) = tan x, al ulamos

1 + tan2 x dx =

sec dx =

log(sec x+ tanx).Substituindo os limites de integração obtemos:R: log(2 +√2

2−√2

).Problema 3.Prove que se f(x) = ax + b parax ∈ [C,D] (f é uma poligonal neste tre ho) en-tão o omprimento do grá o neste intervalo éigual a√(D − C)2 + (aD − aC)2 ( omprimentodado por Pitágoras).2.6.5 ⋆Problemas (Área de Super-fí ie de Sólido de Revolução) dap.52Problema 1. (a) π

27(103/2 − 1).

Page 106: Sumario´ - labMA/UFRJmcabral/livros/livro-calculo/... · 2019. 6. 12. · Sumario´ 1 Enunciados dos Exercícios 1.1 Exercícios de Limite. 1 1.1.1 Exercícios de Fixação. 1 1.1.2

102 CAPÍTULO 2. RESPOSTAS DOS EXERCÍCIOS(b) π

32(18

√5− log(2 +

√5)).( ) π(√2 + log(1 +

√2)).2.6.6 Desaos da p.52Desao 1. Suponha que os ilindros possuem omoeixos os eixos x e y. Agora para ada plano z = s,que é paralelo aos eixos, a interseção é um qua-drado de lado 2L. Por Pitágoras, r2 = L2 + s2,ou, L(s) = √

r2 − s2. A área de ada quadradoé A(s) = (2L(s))2 = 4(r2−s2). Assim o volumeé∫ r

−r4(r2 − s2) ds =

16

3r3.Pro ure na internet Interse tion of Two Cylinders.Desao 2. Introduzindo oordenadas e olo andoum ír ulo na origem e outro em (0, L), obtemosque

x2 + y2 = R2 e x2 + (y − L)2 = r2.Vamos denotar por (x0, y0) uma solução positivadeste sistema. Suponha, sem perda de generali-dade, que r < R. Geometri amente (faça umagura) é laro que para que o orra uma interse-ção,L− r ≤ R ≤ L+ r.Resolvendo para y0 subtraindo as duas equaçõesobtemos que

y20 − (y0 − L)2 = R2 − r2.Assim,y0 =

R2 − r2 + L2

2L.Como r < R, y0 ≥ 0. Agora podemos denir

x0 =√

R2 − y20.Vamos al ular a área da lúnula omo a dife-rença entre duas integrais. Para isto denimosyR(x) =

R2 − x2 e yr(x) = L+√

r2 − x2,funções que delimitam a lúnula. Assim a sua áreaé igual a∫ x0

−x0

(yr(x)− yR(x)) dx.Por simetria a área pode ser al ulada por2

∫ x0

0(yr(x)− yR(x)) dx.

Agora pre isamos da primitiva de √a2 − x2:

a2 − x2 dx =x

2

a2 − x2+a2

2arcsen(x/a).Apli ando esta fórmula e substituindo√R2 − x20por y0 e √r2 − x20 por |y0 − L| obtemos que aárea é

x0(2L+ |y0 − L| − y0)+

+ r2 arcsen(x0/r)−R2 arcsen(x0/R).Embora formalmente orreta, temos que ve-ri ar que a as funções arcsen estão bem deni-das. Para isto temos que veri ar que (porque?)0 ≤ x0 ≤ R e 0 ≤ x0 ≤ r.De fato, omo x0 é solução positiva e satisfazx20 + y20 = R2, x20 ≤ R2. Logo 0 ≤ x0 ≤ R.Como também satisfaz x20 + (y0 − L)2 = r2, deforma análoga, 0 ≤ x0 ≤ r.Desao 3. Isto o orre pois f(xi+1)f (xi)

2→ f(x)quando ∆xi → 0.